You are on page 1of 228

72.3.

7 Unit 7:
We are going to give you some suggestions in this section on how to deal with portions of Unit-7. We
would be focusing on the following questions which generally one has in his/her mind when one thinks
about the easiest way to prepare for Unit-7.

Is this topic important keeping in mind CSIR-NET is in 2 months?

Which books to follow?

How many questions can I expect from this topic?

How to prepare in less time?

Topic-wise suggestion:

Is this topic important?

Another huge Unit of the CSIR-NET exam. But no need to stress out for this. This Unit includes
the basics of all physiological phenomenon in Animals. You can expect around 4-5 questions
every year in Part-C from this Section. Part-B questions from this Unit are basically rare, but 1-
2 questions can be expected.

How shall I start?

All the topics from this Unit are independent and can be followed up as individual chapters. But
it is recommended to start with the topics related to Nervous System or Endocrine System. This
way you can get a better understanding of the overall working mechanisms of all the organs and
organ systems. This would make it easier to go through the rest of the topics and sub-topics of
this Unit.
Which books to follow?
You can follow any standard Book for this Unit. For Additional reference, I will suggest the following
books:

Principles of Anatomy and Physiology:


o This book encompasses all the important required to prepare this Unit. The book
has maintained in the text the superb balance between structure and function and
continue to emphasize the correlations between normal physiology and
pathophysiology, normal anatomy and pathology, and homeostasis and homeostatic
imbalances
Textbook of Medical Physiology by Arthur C Guyton:
o This book is very thorough regarding every topic. So only if you need in depth
suggestions or clarifications, then go for this book. Otherwise follow any other book
of your choice which you find convenient.

Time for this Unit:

The Unit contains topics which are very familiar to us. Thus even after being exhaustive, this Unit
can be dealt with if you spend an hour each day for the next 60 days.

Prioritize the subtopics:

All the chapters in this Unit can be dealt separately as individual sections. But for a better
understanding, you can start with Nervous system or Endocrine System portions.

Suggestions:

For this Unit, you need to go through all the sections thoroughly.
Not much of specific suggestions are required to do the same in this Unit.
Topic-wise suggestion:
Blood and circulation:

In this section, the most important topics you can expect questions from are the major
pathways of cells, intrinsic and extrinsic factors involved during Blood Clotting.
Composition of the different types of cells in the Circulatory System is also very important.

Cardiovascular System:
Significant topics of this section include different volume capacities and
mechanism of heart beat and action.
Respiratory system:
I would suggest you to put a little stress on the topics related to the lung volumes and
physiology of the Respiratory System.
Nervous system:
Thorough knowledge of Action Potential of neurons is very important to score from
this section. You should also go through the topics and sub topics involving types of
neurons and neurotransmitters.
Sense organs:
Most of the topics in this section are fact-based and direct questions can be expected
from this part. So just go through thoroughly and answer specifically.
Excretory system:
Questions from this section are to be dealt carefully. Mostly questions appear from the
portions involving the topics of Net filtration rate, Glomerular filtration rate.
Thermoregulation:
Commonly questions from this section are very rare. Mostly in part-C you can expect.
Going through the topics of this section is enough to crack any question from the same.
Stress and adaptation:
Go through in general these topics.
Digestive system:
This is a very basic topic. Thorough knowledge regarding the different enzymes involved
and their region of action along with production is recommended
BLOOD AND CIRCULATION

1. Increased blood flow to muscles during exercise is due to arteriolar vasodilation caused by:
a. local factors: decreased PO2 , Increased PCO2 , decreased pH, K+, adenosine
b. activation of parasympathetic cholinergic fibers
c. activation of alpha 1 adrenergic receptors
d. activation of beta 1 adrenergic receptors
e. activation of sympathetic cholinergic fibers

2. The number of oxygen molecules carried by one Hb molecule


a. 1
b. 2
c. 4
d. 8

3. Arneth count is counting of


a. lymphocytes
b. lobes of eosinophils
c. lobes of neutrophils
d. reticulocyte.

4. Majority of clotting factors are produced in


a. Liver
b. Kidney
c. Heart
d. Brain

5. Endothelial cells synthesise


a. Fibrinogen
b. factor 8
c. factor 10
d. factor 12.

6. Iron is stored in the body in the following except


a. spleen
b. R.E. system
c. gall bladder
d. bone marrow.

7. Action of plasmin is
a. to remove calcium
b. antithrombin action
c. to stimulate heparin
d. to degenerate fibrin

8. Osmotic pressure of plasma is mainly maintained by


a. Albumin
b. alpha globulin
c. beta globulin
d. gamma globulin.

9. Which is the most rare blood group


a. A Rh+
b. AB Rh+.
c. AB Rh-.
d. B Rh-

10. Hematocrit of 45% means that in the sample of blood analysed


a. 45% Hb is in the plasma
b. 45% of total blood volume is made up of plasma
c. 45% of Hb is in the RBC
d. 45% of the total blood volume is made up of RBC's and WBC's.

11. The normal Albumin/Globulin ratio in blood is


a. 1:2
b. 2:1
c. 1:3
d. 3:1.

12. Which of the following statements concerning the monocyte is incorrect


a. more common in blood than eosinophils and basophils
b. produced in the adult by the bone marrow and lymph nodes
c. unlike neutrophil does not accumulate outside circulation in area of inflammation
d. not classified as a granulocyte.

13. The normal non fasting blood ketone level is


a. 0.1 - 0.5 mg%
b. 0.5- 2 mg%
c. 2- 10 mg %
d. 100 - 500 mg%.

14. Choose one correct answer.


a. Cardiac muscle requires beat-to-beat Ca influx to trigger contraction.
b. A DHP blocker will increase the strength of contraction in heart muscle.
c. SR Ca release channels are called DHP receptors.
d. None are correct.

15. Choose one correct answer.


a. Digitalis decreases the strength of contraction of heart muscle.
b. Na-Ca exchange requires ATP directly and normally acts to pump Ca into heart cells.
c. Blockade of SR Ca pumps will increase the rate of relaxation of heart and skeletal muscle.
d. None are correct.

16. Positive bathmotropic effect on heart is produced by


a. stimulation of vagus nerve
b. stimulation of sympathetic nerves
c. atropine
d. sectioning of vagus.

17. Mary's law denotes relationship between heart and


a. contractility and conductivity
b. rate and contraction
c. rate and BP
d. contraction and BP.

18. Which of the following conducting systems has the slowest conducting velocity
a. Sino Atrial Node
b. atrial muscle
c. purkinje fibres
d. Atrioventricular node.

19. In heart, within physiological limits the force of contraction is directly proportional to the
a. pacemaker activity
b. a-v nodal delay
c. initial length of the cardiac muscle.
d. respiratory rate.

20. The diacrotic notch on aortic pressure curve is caused by


a. closure of mitral valve
b. closure of tricuspid valve
c. closure of atrial valve
d. closure of pulmonary valve.

21. The PR interval of ECG corresponds to


a. ventricular repolarization
b. ventricular repolarization
c. atrial repolarization and conduction through AV node
d. repolarization of AV node and bundle of His.

22. Increased vagal tone causes


a. Hypertension
b. Tachycardia
c. Bradycardia
d. increase in cardiac output.

23. Which of the following is not increased during exercise.


a. Stroke volume
b. total peripheral resistance
c. systolic BP
d. heart rate.

24. Which of the following takes longest time to return to normal after 1L of blood is removed
from a normal individual
a. number of RBC's in peripheral blood
b. plasma volume
c. renin secretion
d. Blood pressure.

25. In a patient with mitral stenosis one would expect to hear


a. continuous murmur
b. a systolic murmur loudest over the base of heart
c. a diastolic murmur loudest over the apex of heart
d. a diastolic murmur loudest over the base of heart.

26. The 's' wave in ECG is below isoelectric line because of


a. repolarization of ventricles
b. change in direction of the impulse when the base of the ventricles are getting depolarised
c. depolarisation of apex of heart
d. repolarisation of apex of heart

27. Which of the following is least likely to cause hypertension


a. chronically increased secretion of adrenal medulla
b. treatment with OCP
c. chronically increased secretion of thyroid gland
d. Chronically increased secretion by zona glomerulosa of adrenal cortex.

28. Lymph flow from the foot is


a. increased when an individual rises from the supine to standing position
b. increased by massaging the foot
c. increased when capillary permeability is decreased
d. decreased by exercise

29. The pressure in the radial artery is determined by


a. the degree of constriction of brachial vein
b. the rate of discharge in sympathetic nerve fibres to the arm
c. pressure in the hepatic portal vein
d. pressure in the brachial vein

30. The 'T' wave in ECG is above the isoelectric line because of
a. depolarisation of ventricles
b. depolarisation of bundle of His
c. change in the direction of repolarisation from the wave of depolarization of the ventricles
d. repolarisation of purkinje fibres

31. In Caissons disease pain in the joints and muscles is due to


a. formation of N2 bubbles
b. formation of CO2 bubbles
c. due to fatigue
d. due to increase in barometric pressure

32. Which of the following will require the smallest increase in oxygen consumption by the heart?
a. Generating an increase in left ventricular pressure.
b. Severe hypertension.
c. Generating an increase in stroke volume
d. Aortic stenosis.

33. Which of the following vessels has the largest effect on total peripheral resistance?
a. Arteries.
b. Arterioles.
c. Veins.
d. Venules.
e. Capillaries.

34. If you decrease a blood vessel's radius in half, by what fraction does the blood flow change?
a. 1/2
b. 1/4
c. 1/8
d. 1/16
e. 1/32

35. Which of the following is INCORRECT concerning turbulent blood flow?


a. The resistance to turbulent flow is greater than laminar flow.
b. Above a critical velocity, blood flow becomes turbulent.
c. Turbulence can create vibrations that can be detected as murmurs.
d. Turbulence is found normally in the aorta and in narrowed vessels.
e. Turbulent flow rate is proportional to the cube root of the driving pressure.

36. Which of the following blood vessels has the greatest compliance?
a. Arteries.
b. Veins.
c. Arterioles.
d. Capillaries.

37. Which of the following pairs is INCORRECT?


a. Nitric oxide (NO): vasodilator
b. Endothelin: vasoconstrictor
c. Nitric Oxide (NO): promotes cell growth
d. Endothelin: potent positive inotropic effect

38. Normal Bleeding time is


a. 35 sec
b. 3-5 minute
c. 57 minute
d. 11-13 sec

39. Normal Serum cholesterol is


a. 60-180 unit/L
b. 150- 250 mg/dl
c. 30- 120 mg/dl
d. 150- 250 unit/L
40. Normal Serum creatine is
a. 0.6 1.5 mg/dl
b. 0.2 0.6 mg/dl
c. 15 40 mg/dl
d. 8.5 10.5 mg/dl

41. After moderate exercise, following changes occur in blood flow:


a. Increase in heart rate,
b. Increase in muscle blood flow,
c. Decrease in peripheral resistance,
d. All of the above.

42. Fetal oxygenation depends on all except


a. HbF,
b. Blood flow through placenta,
c. Fetal respiration,
d. Fetal heart rate.

43. In the Blood antibodies are found in


a. Serum albumin
b. Serum globulin
c. Erythrocyte cell membrane
d. Serum fibrin
44. Proaccelerin Blood Clotting factor is
a. 4
b. 5
c. 6
d. 7

45. Normal Haemoglobin count in male is


a. 12-14gm/100ml
b. 14-16 gm/100ml
c. 16-18 gm/100ml
d. 18-20 gm/100ml

46. A mature human RBC has an average diameter of about


a. 2.5 micron
b. 5.5 micron
c. 7.5 micron
d. 10.5 micron

47. Each haemoglobin molecule can combine with how many molecules of Oxygen.
a. 4
b. 3
c. 2
d. 1
48. In health, the albumin globin (A/G) ratio is about
a. 0.5
b. 1.2
c. 1.7
d. 2.3

49. Blood group were discovered by


a. Romanowsky
b. Landsteiner
c. Hopkins & Funk
d. McMurray

50. The enzyme Serum alkaline phosphatase is produced by


a. Bone
b. Liver
c. Placenta
d. All

51. Haemophilia is due to deficiency of


a. Factor VII
b. Factor VIII
c. Factor IX
d. Factor X

52. Haemophilia B is due to deficiency of


a. Factor VII
b. Factor VIII 18
c. Factor IX
d. Factor X

53. Which of the following vitamin is essential for rapid wound healing
a. Vit. A
b. Vit. C
c. Vit. E
d. Vit. D

54. In normal person, blood is prevented from clotting by all except


a. Plasminogen,
b. Normal blood flow,
c. natural anti clotting factor,
d. Smooth endothelial cells.

55. Blood helps to maintain homeostasis by


a. transporting materials between the tissue fluid and the external environment
b. ridding the body of wastes
c. breaking down nutrients
d. coordinating metabolic reactions

56. The average blood volume for an adult is about


a. 45 liters
b. 450 ml
c. 5 gallons
d. 5 liters

57. In which situation does the person described first have a higher blood volume?
a. a person with 30% body fat or a person with 10% body fat if they are the same size
b. a person weighing 200 lb. or a person weighing 100 lb., if both have the same % body fat
c. a dehydrated person or a person with normal body water content, assuming they have
the same weight and % body fat
d. a female or a male, assuming same body weight and percent body fat

58. Each body function on the left is correctly matched with the corresponding function of the
blood on the right EXCEPT
a. respiration - transports oxygen and carbon dioxide
b. immune defense - platelet factors initiate clotting
c. acid-base balance - buffers acids and bases
d. thermoregulation - allows heat to escape from the body at the skin

59. Which statement concerning blood viscosity is correct?


a. The viscosity of blood is 4.5 to 5.5 higher than the viscosity of water.
b. Blood viscosity is due to the presence of the plasma proteins and erythrocytes.
c. The higher the blood viscosity the harder the heart has to work to move blood through
the vessels.
d. Anemia increases blood viscosity.

60. Low blood osmolarity


a. causes the blood to absorb excess tissue fluid as it passes through the capillaries
b. could lead to increased blood volume and increased blood pressure
c. allows too much fluid to remain in the tissues and leads to edema
d. could be caused by increased plasma protein or sodium levels

61. What percentage of body weight is blood?


a. 91%
b. 55%
c. 8%
d. 45%

62. Other than water, the most common component of plasma is


a. chloride
b. urea
c. protein
d. sodium

63. Which problem could NOT be caused by a deficiency of plasma proteins?


a. edema
b. reduced ability to transport oxygen
c. reduced ability to clot
d. reduced ability to transport molecules such as lipids

64. The antibodies that circulate in the plasma are made by


a. the liver, which makes all of the plasma proteins
b. the kidney, which monitors plasma composition
c. B lymphocytes, which are part of the immune system
d. the spleen, which uses them to tag red blood cells for destruction

65. If you have some blood from which the formed elements have been removed, how can you
tell if it is plasma or serum?
a. plasma contains hemoglobin; serum does not
b. serum is yellow; plasma has no color
c. serum contains antibodies; plasma does not
d. plasma contains clotting proteins; serum does not

66. Blood colloid osmotic pressure


a. is the component of total blood osmotic pressure due to the presence of proteins in the
plasma
b. would increase if blood sodium levels increase
c. would increase if the number of RBCs increases
d. would increase in a person who has liver damage

67. In which choice are the major groups of plasma proteins listed in order from highest to lowest
percent?
a. fibrinogen, globulins, albumins
b. globulins, fibrinogen, albumins
c. albumins, globulins, fibrinogen
d. albumins, fibrinogen, globulins

68. In which choice is the plasma protein correctly matched with one of its functions?
a. albumin - antibodies involved in immune response
b. fibrinogen - attacks and destroys pathogens
c. globulins - transport minerals, lipids, hormones and vitamins
d. globulins - colloid osmotic pressure

69. The benefit of the biconcave shape of red blood cells is that it
a. decreases the distance between the cell membrane and hemoglobin molecules,
facilitating oxygen uptake
b. allows the cells to hold more hemoglobin
c. gives more room for the nucleus and organelles
d. keeps them from leaking out through the walls of capillaries

70. Blood cells all develop from a common stem cell in bone marrow, but early in development
they split into separate lineages. Which three of these cells develop from the myeloblast
lineage?
a. basophil, eosinophil, neutrophil
b. lymphocyte, monocyte, erythrocyte
c. thrombocyte, erythrocyte, lymphocyte
d. monocyte, erythrocyte, neutrophil

71. Blood test results for an adult male are returned and the total red blood cell count is 2,500,000
cells per mm3. This person
a. has a normal total red blood cell count
b. has an abnormally low total red blood cell count
c. has an abnormally high total red blood cell count
d. shows that he has been exercising more than usual or has spent some time at high
altitudes

72. As red blood cells get older, they


a. are less likely to become damaged
b. become more fragile
c. are less likely to rupture
d. are less likely to be phagocytized in the spleen or liver

73. Damaged red blood cells are phagocytized by macrophages in the spleen or liver and their
components are recycled, EXCEPT for the
a. iron atom from the heme group, which is excreted in bile
b. globin peptide chains, which are sent to the kidneys for excretion
c. globin peptide chains, which are converted into bilirubin
d. heme group (minus the iron atom), which is converted into a green pigment called
biliverdin

74. What would you suspect to find if someone has high blood erythropoietin levels?
a. blood oxygen deficiency
b. high blood oxygen levels
c. a high red blood cell count
d. a high hemoglobin level

75. Which hormone stimulates red blood cell formation?


a. thrombin
b. transferrin
c. bilirubin
d. erythropoietin

76. Which condition would stimulate erythropoietin secretion?


a. high red blood cell count
b. low blood oxygen level
c. high hemoglobin levels
d. increased blood flow through kidneys

77. The lower the total amount of iron stored in the body,
a. the higher the rate of red blood cell formation.
b. the higher the absorption of ingested iron in the intestine.
c. the lower the level of transferrin in the blood.
d. the higher the level of ferritin in the liver.
78. Which blood component is NOT correctly matched with its function?
a. erythrocytes - transport oxygen
b. leukocytes - protect against disease
c. platelets - phagocytize bacteria
d. plasma proteins - maintain blood osmotic pressure; involved in clotting

79. In which choice are the formed elements arranged in order of increasing size?
a. red blood cell, neutrophil, platelet
b. platelet, red blood cell, neutrophil
c. neutrophil, platelet, red blood cell
d. platelet, neutrophil, red blood cell

80. Which cell is NOT classified as a granulocyte?


a. basophil
b. eosinophil
c. lymphocyte
d. neutrophil

81. In adults, red blood cells are made in one place, spend most of their lifespan in another, and
most are finally destroyed in yet another place. Which choice lists these locations in
chronological order?
a. bone marrow; blood vessels; spleen
b. bone marrow; spleen; blood vessels
c. spleen; blood vessels; spleen
d. blood vessels; spleen; bone marrow

82. Which leukocyte is NOT correctly matched with its description?


a. eosinophil - bilobed nucleus, deep red granules
b. lymphocyte - smallest leukocytes, round nucleus, thin rim of cytoplasm
c. neutrophil - lobed nucleus with two to five segments, light purple granules
d. basophil - typical kidney-shaped nucleus, largest white blood cell

83. Which type of leukocyte is correctly matched with its function?


a. basophils - release histamine in areas of damaged tissue
b. neutrophils - leave the blood vessels and become macrophages
c. lymphocytes - phagocytize bacteria
d. neutrophils - defend against parasitic worm infections

84. Which is NOT correct about RBC recycling?


a. amino acids can be recycled into new proteins
b. the entire heme molecule is eliminated by the liver
c. iron is removed before heme is converted to bilirubin
d. liver dysfunction can cause accumulation of bilirubin in the body

85. The carrier protein that transports absorbed iron through the blood is
a. thrombopoietin
b. hemoglobin
c. erythropoietin
d. transferrin

86. Joe had to have both of his kidneys removed and is now kept alive only by dialysis machines
that remove waste molecules from his blood and adjust electrolyte levels. Because Joe has no
remaining kidney tissue, he may also need
a. clotting proteins
b. vitamin B12 to stimulate RBC production
c. erythropoietin to stimulate RBC production
d. a bone marrow transplant

87. Without the RBC enzyme carbonic anhydrase


a. the blood would not be able to carry oxygen
b. the blood would not be able to coagulate
c. the blood would not be able to convert carbon dioxide into bicarbonate
d. we would not be able to remove and break down heme

88. What is the role of the nitric oxide (NO) carried by hemoglobin?
a. NO stimulates the formation of more red blood cells
b. NO relaxes blood vessels and decreases blood pressure
c. NO helps the hemoglobin bind to oxygen
d. NO prevents the hemoglobin from picking up carbon monoxide

89. Which process attracts leukocytes to damaged tissue?


a. Chemotaxis
b. Hemostasis
c. Hemopoiesis
d. agglutination

90. If someone has no megakaryocytes, he/she will


a. not be able to make white blood cells
b. not be able to make red blood cells
c. have a reduced ability to prevent blood loss
d. not be able to phagocytize bacteria

91. An overdose of erythropoietin would cause


a. anemia
b. polycythemia
c. low blood viscosity
d. low blood pressure

92. All of these describe normal erythrocytes EXCEPT


a. biconcave disk
b. contain no nucleus
c. contain mitochondria
d. contain hemoglobin

93. Which choice represents an abnormal value?


a. total RBC count - 5.1 million / mL
b. total WBC count - 1500 / mL
c. total platelet count - 250,000 / mL
d. hemoglobin - 15 g / dL

94. Immature erythrocytes that contain a network of endoplasmic reticulum and normally make
up about 1 - 3% of circulating blood are called
a. granulocytes
b. basophilic erythroblasts
c. proerythroblasts
d. reticulocytes

95. Several proteins are involved in the absorption, transport and storage of iron. Each protein is
correctly matched with its function EXCEPT
a. apoferritin - a storage protein in the liver
b. ferritin - an iron-containing protein found in the erythrocytes
c. gastroferritin - binds to iron in the stomach and transports it to the small intestine
d. transferrin - transports iron through the blood

96. Which statement is correct?


a. Thrombopoiesis occurs mainly in the lungs.
b. Thrombopoietin is a hormone that stimulates platelet production.
c. Platelets have a large nucleus containing many sets of chromosomes.
d. Megakaryocytes divide by mitosis to produce platelets.

97. In which choice are the leukocytes correctly arranged in order from most common to least
common?
a. monocytes, lymphocytes, neutrophils, eosinophils, basophils
b. lymphocytes, neutrophils, monocytes, eosinophils, basophils
c. neutrophils, monocytes, lymphocytes, basophils, eosinophils
d. neutrophils, lymphocytes, monocytes, eosinophils, basophils

98. Once released from the bone marrow into the circulation, the average life span of a red blood
cell is about
a. 72 hours
b. one week
c. 4 months
d. 2 years

99. One molecule of hemoglobin


a. contains two alpha globin proteins and two beta globin proteins.
b. contains one heme group.
c. contains 2 atoms of iron.
d. can carry 12 molecules of oxygen.

100. In a red blood cell, carbon dioxide is attached to the


a. iron of the Hemoglobin
b. amino acids of the Globin
c. plasma membrane
d. heme of the hemoglobin

101. Erythropoietin is produced by


a. red blood cells
b. red bone marrow
c. liver
d. kidneys

102. Red blood cells do not use any of the oxygen that they carry, because
a. oxygen is bound to heme and cannot be removed
b. they lack mitochondria to use oxygen for aerobic respiration
c. they do not need ATP
d. they use carbon dioxide instead of oxygen for energy production

103. In which choice are the proteins listed in the order in which they are activated?
a. prothrombin activator, thrombin, fibrin
b. fibrin, thrombin, prothrombin activator
c. thrombin, fibrin, prothrombin activator
d. thrombin, prothrombin activator, fibrin

104. Platelet plug formation is initiated when platelets come into contact with
a. undamaged endothelial cells
b. fibrinogen
c. collagen fibers in the connective tissue beneath the endothelium
d. antithrombin

105. Which statement about hemostasis is NOT correct?


a. Positive feedback rapidly increases the amount of activated clotting factors.
b. The intrinsic mechanism occurs in response to tissue damage.
c. During clotting, blood cells and platelets are trapped in a mesh of protein fibers.
d. The extrinsic mechanism is initiated by tissue factor (thromboplastin), released by
damaged tissues.

106. Repair of damaged blood vessel walls is stimulated by


a. tissue thromboplastin
b. platelet-derived growth factor
c. plasminogen
d. serum

107. After preliminary testing, a man who had had a stroke was given tissue plasminogen
activator (tPA) at the hospital. The reason for this treatment is that tPA
a. activates an enzyme that dissolves clots
b. prevents the formation of clots by removing calcium from the blood
c. prevents the formation of thrombin
d. initiates repair in damaged blood vessels
108. A blood clot that forms in an undamaged vessel is called a(n) _______, and if it breaks
loose and travels through the bloodstream it is called a(n) __________.
a. hematoma; thrombosis
b. thrombus; hematoma
c. thrombus; embolus
d. embolus; thrombus

109. A pulmonary embolism is a blood clot that blocks blood flow in a vessel
a. supplying part of a lung
b. in the heart
c. in the brain
d. in the kidney

110. The production of prostacyclin by endothelial cells


a. means that blood clots do not normally form in healthy blood vessels
b. attracts platelets
c. exposes the collagen below the endothelial cells
d. reduces the synthesis of clotting proteins by the liver

111. Which substance is NOT correctly matched to its function?


a. antithrombin - inactivates thrombin
b. prostacyclin - prevents platelet adhesion
c. heparin - interferes with the formation of prothrombin activator
d. plasminogen - initiates extrinsic mechanism

112. Which of these statements is correct?


a. Thrombin catalyzes the conversion of fibrinogen to fibrin.
b. Ca++ is not required for coagulation.
c. Tissue factor is always required for activation of factor X.
d. The extrinsic mechanism causes coagulation when there is no tissue damage.

113. Which molecule is a potent stimulator of coagulation?


a. Prostacyclin
b. tissue factor or thromboplastin
c. plasmin
d. tissue plasminogen activator

114. During platelet plug formation, platelets release these substances, which cause more
platelets to adhere to the plug.
a. prostacyclin and NO
b. erythropoietin and bilirubin
c. heparin and plasmin
d. ADP and thromboxanes

115. What happens when fibrinogen is converted to fibrin?


a. a transfusion reaction
b. iron is stored in the liver
c. a clot is formed
d. a platelet plug is formed

116. Vascular spasm


a. closes off small blood vessels when they are damaged, mainly in response to chemicals
released from the damaged cells
b. occurs in blood vessels when the pressure is too high
c. occurs when blood oxygen levels are low
d. occurs when a blood vessel dilates in response to pain

117. Which of these blood components is NOT involved in clotting?


a. Platelets
b. Fibrinogen
c. Calcium
d. potassium

118. Each of these prevents unwanted coagulation EXCEPT


a. tissue thromboplastin
b. antithrombin
c. heparin
d. prostacyclin

119. Removal of this substance from blood would prevent clotting


a. Transferring
b. Iron
c. Antibodies
d. calcium

120. A patient lacks fat absorbing mechanism in the intestines. This person is most likely going
to suffer severe bleeding problems because
a. fat droplets help to keep platelets together during clotting
b. vitamin K will not be absorbed and this will lead to deficiency of clotting factors
c. fatty acids are essential for the synthesis of tissue factors
d. Vitamin C will not be absorbed, which is essential for synthesis of clotting factors.

121. A person's blood type is determined by


a. mixing his blood with the blood of another person of known blood typeif there is no
reaction they are the same type
b. looking at red blood cells under a microscope to see what kind of antigens are on their
membranes
c. checking to see which plasma antibodies a person hashe won't have antibodies against
his own blood type
d. mixing the blood with different antibodies to see which ones cause the red blood cells to
agglutinate

122. What kind of risk exists when a woman who is Rh+ carries an Rh- fetus?
a. There is usually no risk during the first pregnancy, but it can harm the fetus during a
subsequent pregnancy if the mother is not treated
b. It always poses a serious risk to the fetus, even in the first pregnancy
c. Only in rare cases is there a risk to the fetus during the first pregnancy
d. There is never a risk to the fetus in any pregnancy

123. Antibodies to blood type A or B antigens are


a. formed when antigen A is present on the red blood cell membranes
b. formed when antigen A is absent from the red blood cell membranes
c. formed during fetal development
d. small enough to cross the placenta

124. A blood transfusion reaction will


a. occur when plasma antibodies contact red blood cells carrying the antigen with which
they react
b. only occur if both red blood cells and plasma are transfused
c. always occur if someone is given type O blood
d. not cause any damage to the red blood cells

125. A person who is Rh negative might form anti-Rh antibodies if she


a. receives a transfusion of Rh positive blood
b. receives a transfusion of Rh negative blood
c. becomes pregnant with a fetus that is Rh negative
d. is given a shot of the drug RhoGAM

126. Erythroblastosis fetalis (hemolytic disease of the newborn) occurs when


a. a premature Rh positive baby is given a transfusion of Rh negative blood
b. anti-Rh antibodies from the maternal circulation cross the placenta and agglutinate the
fetal red blood cells
c. the fetus cannot make normal red blood cells
d. the fetus' white blood cells are destroyed by antibodies from the maternal circulation.

127. Agglutination, which occurs during transfusion reactions, is caused by binding between
a. hemoglobin and the kidney tubule cells
b. clotting proteins and the enzymes that activate them
c. platelets and endothelial cells
d. RBC membrane antigens and plasma antibodies

128. A person with type O+ blood would


a. make anti-Rh antibodies if exposed to the Rh antigen
b. have neither anti-A nor anti-B antibodies in his plasma
c. have Rh but not A or B antigens on his RBCs
d. be able to receive type A+, B+, or AB+ in a transfusion if necessary

129. Blood types are determined


a. genetically by glycoproteins and glycolipids on the RBC membrane
b. genetically by antibodies in the plasma
c. by the different forms of hemoglobin present in the RBCs
d. by exposure to intestinal bacteria during infancy

130. Which of the following is true for an individual's with type A blood?
a. The person can donate blood to type O
b. The individuals plasma contains anti-A antibodie
c. The individual can donate blood to type AB
d. The individual must be Rh+

131. Which of the following is true for an individual with anti-A antibodies in the plasma?
a. The individual is blood type A
b. The individual is blood type B
c. The individual is blood type AB
d. The individual can be either blood type B or AB

132. Type O is considered to be universal


a. donor because it has neither anti-A antibodies nor anti-B antibodies circulating in the
plasma
b. donor because it has neither antigen A nor antigen B on its RBCs
c. recipient because it has neither anti-A antibodies nor anti-B antibodies circulating in the
plasma
d. recipient because it has neither antigen A nor antigen B on its RBCs

133. A dietary deficiency of iron could cause


a. reduced ability of the blood to coagulate
b. a low platelet count
c. insufficient white blood cell production
d. a low red blood cell count

134. For a person whose hematocrit is 45%,


a. white blood cell volume would be 45% of total blood volume
b. red blood cell volume would be 55% of total blood volume
c. plasma volume would be 45% of total blood volume
d. plasma volume would be 55% of total blood volume

135. For a person whose hematocrit is 45%, in which choice below are the components of
blood correctly ranked in order of decreasing percent of total volume?
a. white blood cells, plasma, red blood cells
b. red blood cells, white blood cells, plasma
c. plasma, red blood cells, white blood cells
d. plasma, white blood cells, red blood cells

136. Which of these diseases involves a problem with the red blood cells?
a. Leukocytosis
b. Hemophilia
c. Leukemia
d. sickle-cell anemia
137. A person with a vitamin B12 or folic acid deficiency would probably also have
a. Anemia
b. Polycythemia
c. Leukemia
d. leukocytosis

138. Which describes anemia?


a. a deficiency of white blood cells
b. a high level of hemoglobin
c. a reduction in oxygen-carrying capacity of the blood
d. an excessive number of red blood cells

139. Aplastic anemia is caused by


a. a defective gene for hemoglobin
b. inability to absorb vitamin B12
c. exposure to toxic chemicals or radiation
d. iron deficiency

140. The percentage of whole blood volume occupied by the red blood cells is the
a. specific gravity
b. differential cell count
c. hematocrit
d. viscosity

141. Based on lab results that show a person has a total white blood cell count of 7,000 WBC
/ mm3 blood, you would conclude that this person
a. has hemophilia
b. exhibits leucopenia
c. has leukocytosis
d. has a normal white blood cell count

142. Based on the results of a differential WBC count shown in the table, this person has
Basophils 1%
Eosinophils 10%
Neutrophils 56%
Lymphocytes 25%
Monocytes 8%
a. mononucleosis
b. leucopenia
c. allergies or a parasitic worm infection
d. an acute bacterial infection

143. The buffy coat, a layer that appears when blood is centrifuged for determining the
hematocrit, is made of
a. plasma only
b. plasma proteins only
c. plasma proteins and red blood cells
d. white blood cells and platelets
144. Which of these conditions would most likely cause an increase in the number of
reticulocytes in the blood?
a. blood loss
b. bacterial infection
c. dehydration
d. hemophilia

145. An abnormally high number of normal white blood cells is called


a. Leukemia
b. Leukocytosis
c. Leucopenia
d. leukopoiesis

146. Hemophilia is caused by


a. hypersecretion of erythropoietin
b. a deficiency of any clotting factor
c. overproduction of platelets
d. abnormal hemoglobin

147. Blood disorder characterized by the rupture of red blood cells on infection by protozoa is
a. Septicemia
b. Malaria
c. infectious mononucleosis
d. AIDS

148. Thoracic lymph contains:


a. Clotting factors
b. Higher protein content than plasma
c. Similar composition to ISF
d. Rarely contains fat

149. The total osmotic pressure of plasma is:


a. 25 mmHg
b. 285 mOsm/l (or ?308mOsm/l)
c. 5900 mmHg
d. 300 kPa
e. None of the above

150. Total plasma osmolality can be calculated via:


a. Van Halens equation
b. Starling equation
c. P = nRT
d. (multiplying 19.2mmHg/mOsm/L by body Osm)
e. None of the above

151. Which of the following will increase plasma potassium concentration


a. Beta adrenergic receptor AGONIST
b. Insulin
c. Aldosterone
d. None of the above

152. Osmotic pressure in plasma is usually 1.6 mosmol/L more than ISF. This is because of
a. Plasma Proteins
b. Plasma Oxygen Tension
c. Plasma creatinine

153. Arterial gases including pH 7.46 bicarbonate 31mmol/l PCO2 46mmHg indicate:
a. Metabolic alkalosis with respiratory compensation
b. Respiratory alkalosis
c. Respiratory acidosis with compensation
d. Metabolic acidosis with respiratory compensation
e. Mixed metabolic and respiratory alkalosis

154. Which of the following decrease platelet aggregation & cause vasodilatation?
a. PGE2
b. PGF2alpha
c. TBXA2
d. PGD2
e. PGI2

155. Which is associated with inhibition of platelet aggregation?


a. Prostaglandin I
b. Prostaglandin E
c. Prostaglandin F
d. Prostaglandin F1

156. Which ONE of the following causes bronchodilatation?


a. PGE2
b. PGF2 alpha
c. TBXA2
d. LTB4
e. LTD4

157. In a patient receiving 24 units of blood over 2 hours, the complication most likely to be
seen would be:
a. Hypercalcaemia
b. Increased oxygen uptake in the lungs
c. Coagulopathy
d. Hypokalaemia

158. Problems of massive transfusion most commonly include:


a. Metabolic alkalosis
b. Hyperkalaemia
c. Coagulopathy due to hypocalcaemia
d. Hypokalaemia

159. The effect which is LEAST likely to occur shortly after transfusion of 25U of whole blood
a. Hypocalcaemia
b. Dilutional coagulopathy
c. Metabolic alkalosis
d. Increased affinity of Hb for O2
e. Hyperkalaemia

160. Which immunoglobulin (?MW 69,000) would exist as a monomer in tears, saliva & mucus
secretions?
a. IgA
b. IgG
c. IgM
d. IgE
e. IgD

161. Erythropoietin is a glycoprotein which:


a. Stimulates red and white cell production
b. Is broken down in the kidney
c. Has a half life of days
d. Levels inversely proportional to haematocrit
e. none of the above

162. Erythropoietin:
a. Red cell maturation 24 to 72 hours
b. Inactivated by Kupffer cells
c. Metabolised in liver
d. Half-life is 5 ?mins/hours

163. Antithrombin III affects (inactivates) which coagulation factor?


a. XIIa (?XIa)
b. Xa
c. IIa
d. IXa
e. All of the above

164. Vitamin K (neutralizes):


a. Factor 5
b. Heparin
c. Antithrombin 3
d. Plasminogen
e. None of the above

165. Desmopressin:
a. Increases factor 8 levels/activity
b. Anti-heparin effect
c. Has pressor activity
166. Post-translational modification occurs with:
a. Factor V
b. Von Willebrand factor
c. Factor XII
d. Protein C

167. Post-translational modification:


a. Removal of introns
b. Modification of amino acid residues in proteins
c. Self-splicing
d. tRNA involved

168. Haemoglobin breakdown:


a. Fe is excreted by the kidney
b. Haem is broken down to biliverdin
c. Haem is converted to bilirubin and transported to liver bound to albumin

169. Platelet activation will NOT occur without:


a. Ca+2
b. Vessel wall damage
c. Von Willebrand factor
d. Fibrinogen
e. Serotonin ?Factor VIII

170. Glycoprotein CD4 is expressed on:


a. Cytotoxic T cells
b. Suppressor T cells
c. Helper T cells
d. Plasma cells

171. Immunoglobulin G (IgG) has:


a. 4 heavy chains
b. 4 light chains
c. 2 heavy & 2 light chains
d. Variable heavy & light chains
e. None of the above

172. Platelet activation requires:


a. Vessel wall damage
b. Ca++-
c. Cyclooxygenase
d. vonWillebrand factor
e. Prostaglandins

173. Cytokines are:


a. Low molecular weight proteins
b. Enzymes
c. Autacoids
d. Immunoglobulins
e. Interleukins

174. Complement activation requires


a. Antigen antibody complex
b. Opsonisation of bacteria
c. Helper T cells
d. Previous exposure to antigen
e. Plasma proteins

175. Fixed macrophages in lungs & liver:


a. Originate in the bone marrow and migrate to their site of action as megakaryocytes
b. Kill bacteria in phagosomes by lymphokines
c. Are activated by cytokines secreted by activated T cells
d. Part of humoral immunity

176. HLA antigens are found on:


a. All leucocytes
b. B cells
c. T cells
d. All nucleated cells

177. For a T cell to react to (recognise) a foreign antigen:


a. Previous exposure
b. Presentation of antigen by Antigen presenting cells
c. Active T helper cells

178. Thrombin inhibits


a. factor Xa
b. tPA
c. protein C
d. platelets
e. none of the above

179. Lymphocytes
a. Dont remain in the lymph system
b. Are formed in the bone marrow in adults
c. Formed from neonatal precursor cells
d. Produced by tissues derived from foetal bone marrow

180. Rejection of an allograft is due to:


a. Non specific immunity
b. Supressor T cells
c. Helper T cells
d. Cytotoxic T cells
e. HLA cytotoxic reaction
181. Haemoglobin contains:
a. One protoporphorin ring and 4 ferrous ions
b. Four protoporphorin ring and one ferrous ion
c. Four protoporphorin rings and four ferrous ions
d. One protoporphorin ring and one ferrous ion

182. Blood viscosity:


a. Is independent of the white cell count
b. Falls as haematocrit rises
c. Is independent of vessel diameter
d. Falls as flow rate rises
e. Is independent of fibrinogen concentration

183. ABG's (Arterial blood gas)in healthy young man with pneumothorax:
a. pO2=50, pCO2=25
b. pO2=50, pCO2=46
c. pO2=90, pCO2=25
d. pO2=90, pCO2=46

184. ABG pH 7.48, PCO2 24 (or 26), HCO3 19 BE 15 is consistent with:


a. Mixed metabolic and respiratory acidosis
b. Acute respiratory alkalosis
c. Metabolic acidosis with compensated respiratory alkalosis
d. Chronic respiratory disease
e. Mountain climber after several weeks at altitude
f. Hyperventilating consistent with acclimatization to altitude

185. Bicarbonate system is (?the most) important buffer system because:


a. Has a pKa close to physiological pH
b. CO2 can be exchanged in lungs and HCO3 excreted in the kidneys
c. HCO3
d. Can be regulated by lung & kidney

186. ABGs: pH 7.35, pCO2 60 mmHg, pO2 40 mmHg. These blood gas results are consistent
with:
a. Atelectasis
b. Morphine induced respiratory depression (OR: Acute morphine overdose)
c. Diabetic ketoacidosis
d. . Patient with COAD

187. Buffering of a bicarbonate infusion:


a. 60 to 70% occurs intracellularly
b. Exchanged for Cl- across the red cell membrane
c. Compensated for by increased respiratory rate.
d. Intracellular proteins

188. Phosphate buffer system is an effective buffer intracellularly and in renal tubules because:
a. Its pKa is close to the operating pH
b. High concentration in distal tubule
c. High concentration intracellularly
d. All of the above

189. Arterial gases including pH 7.46 bicarbonate 31mmol/l PCO2 46mmHg indicate:
a. Metabolic alkalosis with respiratory compensation
b. Respiratory alkalosis
c. Respiratory acidosis with compensation
d. Metabolic acidosis with respiratory compensation
e. Mixed metabolic and respiratory alkalosis.

190. Which contains Oxygenated blood


a. Pulmonary artery
b. Right ventricle
c. Left Ventricle
d. Venules

PART-C

191. During the Spanish conquest of the Inca Empire at the high altitude in Peru, many soldiers
fell sick. It was found that the sickness was due to low partial pressure of O2 in the atmosphere
at that altitude. To determine the reason, blood was collected from those patients. The
circulating erythropoietin (EPO) level were estimated and the O2-dissociation curve of
haemoglobin were drawn and compared with the same in native people as depicted below

Which of the following combinations is logically correct?


a. A and C
b. A and D
c. B and C
d. B and D

192. Which of the following are not required for clot formation? (1) vitamin K, (2) calcium, (3)
prostacyclin, (4) plasmin, (5) fibrinogen.
a. 1, 2, and 5
b. 3, 4, and 5
c. 4 and 5
d. 1, 2, and 3
e. 3 and 4

193. A person with type A Rh- blood can receive a blood transfusion from which of the
following types? (1) A Rh+, (2) B Rh-, (3) AB Rh-, (4) O Rh-, (5) A Rh-.
a. 1 only
b. 3 only
c. 4 only
d. 4 and 5
e. 1 and 5

194. A person with type B positive blood receives a transfusion of type AB positive blood. What
will happen?
a. The recipients antibodies will react with the donors red blood cells.
b. The donors antigens will destroy the recipients antibodies.
c. The donors antibodies will react with and destroy all of the recipients red blood cells.
d. The recipients blood type will change from Rh_ to Rh_.
e. These blood types are compatible, and the transfusion will be accepted.

195. Blood platelets assist in arresting bleeding by all of the following except.
A. Releasing factors promoting blood clotting.
B. Adhering together to form plugs when exposed to collagen.
C. Liberating high concentrations of calcium.
D. Releasing factors causing vasoconstriction.
E. Inhibiting fibrinolysis by blocking the conversion of plasminogen to plasmin.

a. E only,
b. B and C,
c. D and E
d. C and E

196. A person who was recently been on broad spectrum antibiotics for a recurrent urinary
bladder infection. While slicing vegetables, he cuts himself and had difficulty stopping the
bleeding. How could the antibiotics have played a role in her bleeding?
a. Antibiotics have removed the normal flora of large intestine which were producing
vitamin K.
b. Antibiotics do not allow the absorption of vitamin K.
c. Antibiotics for complex with Vitamin K therefore unavailable for clotting.
d. Antibiotics are directly inhibiting the clotting factor.

197. Which of the following are true concerning capillary exchange?


1. Large, lipid-insoluble molecules cross capillary walls by transcytosis.
2. The blood hydrostatic pressure promotes reabsorption of fluid into the capillaries.
3. If the pressures that promote filtration are greater than the pressures that promote
reabsorption, fluid will move out of a capillary and into interstitial spaces.
4. A negative net filtration pressure results in reabsorption of fluid from interstitial spaces
into a capillary.
5. The difference in osmotic pressure across a capillary wall is due primarily to red blood
cells.
a. 1, 3, and 4
b. 1, 2, 3, 4, and 5
c. 1, 2, 3, and 4
d. 3 and 4
e. 2, 4, and 5

198. Which of the following would not increase vascular resistance?


1. vasodilation,
2. polycythemia,
3. obesity,
4. dehydration,
5. anemia.
a. 1 and 2
b. 1, 3, and 4
c. 1 and 5
d. 1, 4, and 5
e. 1 only

199. Capillary exchange is enhanced by


(1) the slow rate of flow through the capillaries,
(2) a small cross-sectional area,
(3) the thinness of capillary walls,
(4) the respiratory pump,
(5) extensive branching, which increases the surface area.
a. 1, 2, 3, 4, and 5
b. 1, 2, 3, and 5
c. 1 and 3
d. 3 and 5
e. 1, 3, and 5

200. Systemic vascular resistance depends on which of the following factors?


(1) blood viscosity,
(2) total blood vessel length,
(3) size of the lumen,
(4) type of blood vessel,
(5) oxygen concentration of the blood.
a. 1, 2, and 3
b. 2, 3, and 4
c. 3, 4, and 5
d. 1, 3, and 5
e. 2, 4, and 5
ANSWER KEYS:

1.a

2.c

3.c

4.a

5.b

6.c

7.d

8.a

9.c

10.d

11.b

12.c

13.c

14.a

15.d

16.b

17.c

18.c

19.c

20.c

21.c

22.c
23.b

24.a

25.c

26.b

27.c

28.b

29.b

30.c

31.a

32.c

33.b

34.e

35.e

36.b

37.c

38.b

39.b

40.b

41.c

42.c

43.c

44.a

45.c

46.c

47.a
48.c

49.b

50.d

51.b

52.c

53.b

54.a

55.a

56.d

57.b

58.b

59.d

60.c

61.c

62.c

63.b

64.c

65.d

66.b

67.b

68.c

69.a

70.a

71.b

72.b
73.d

74.a

75.d

76.b

77.b

78.c

79.b

80.c

81.a

82.d

83.a

84.b

85.d

86.c

87.c

88.b

89.a

90.c

91.b

92.c

93.b

94.d

95.b

96.b

97.d
98.c

99.a

100.b

101.d

102.b

103.a

104.c

105.b

106.b

107.a

108.c

109.a

110.a

111.d

112.a

113.b

114.d

115.c

116.a

117.d

118.a

119.d

120.b

121.d

122.d
123.b

124.a

125.a

126.b

127.d

128.c

129.a

130.c

131.b

132.b

133.d

134.d

135.c

136.d

137.a

138.c

139.c

140.c

141.d

142.c

143.d

144.a

145.b

146.b

147.b
148.a

149.e

150.e

151.d

152.a

153.a

154.e

155.b

156.a

157.c

158.b

159.c

160.a

161.e

162.c

163.e

164.e

165.a

166.d

167.b

168.b

169.a

170.c

171.c

172.b
173.a

174.e

175.c

176.d

177.b

178.e

179.c

180.d

181.c

182.d

183.a

184.f

185.b

186.d

187.b

188.d

189.a

190.c

Part C

191.b

192.e

193.d

194.a

195.d
196.a

197.a

198.c

199.e

200.a
CARDIOVASCULAR SYSTEM

1. The increased cardiac output during exercise is caused by all of the following, except:
a. stimulation of beta 1-adrenergic receptors
b. increase in venous return
c. increased blood pressure
d. increased stroke volume and heart rate
e. inhibition of the parasympathetic tone

2. Based on experimental observations, the only factor(s) in arterial blood that could account for
increased ventilation at or near VO2max is(are):
a. PO2
b. PCO2
c. pH
d. all of the above
e. (b) and (c) only

3. Which of the following is NOT a part of the specialized conduction system of the heart?
a. Cells of the SA node.
b. Cells of the AV node.
c. Working myocardial cells.
d. His bundle.
e. Purkinje fibers.

4. Proto diastolic phase time duration in cardiac cycle is -(Ventricular diastole)


a. 0.4 sec.
b. 0.10 sec.
c. 0.04 sec.
d. 0.15 sec.

5. Total Filling phase duration in ventricular diastole is


a. 0.25 sec.
b. 0.10 sec.
c. 0.04 sec.
d. 0.4 sec.

6. Duration of 1st Heart sound is


a. 0.9- 0.14 sec.
b. 0.9- 0.16 sec.
c. 0.9- 0.18 sec.
d. 0.15 sec.

7. Total Ejection phase duration in ventricular systole is


a. 0.25 sec.
b. 0.10 sec.
c. 0.04 sec.
d. 0.15 sec.

8. Time duration of 2nd Heart sound is -


a. 0.10- 0.14 sec.
b. 0.9- 0.16 sec.
c. 0.10- 0.12 sec.
d. 0.15 sec.

9. After a meal, the heart rate usually


a. Rises
b. Falls
c. Remains unlatered
d. None

10. The heart rate is least in which posture


a. Standing
b. Sitting
c. Recumbent
d. None

11. Natural pace makes of the heart is


a. S.A. Node
b. A.V. Node
c. Bundle of His
d. Purkinje

12. PR interval is:


a. 0.12- 0.16 seconds
b. 0.3 0.4 sec.
c. 0.6 0.8 sec.
d. 0.12 4 sec.

13. Which of the following is INCORRECT concerning cardiac muscle?


a. Cardiac cells are smaller than skeletal muscle cells.
b. Electrical communication between cardiac cells is maintained via gap junctions which are
specialized portions of the intercalated discs.
c. Mechanical attachment of cardiac cells is at the intercalated disc.
d. The spread of excitation through the heart muscle is 3-dimensional.
e. Transverse tubules are larger in skeletal muscle than in heart muscle allowing more diffusion of
Ca2+ into the interior of the cell.

14. Which of the following is usually the dominant pacemaker and fires the fastest?
a. SA node.
b. AV node
c. His bundle.
d. Purkinje fibers

15. Which of the following is a TRUE statement concerning pacemaker potential?


a. Specialized cells depolarize during phase 4, but ventricular and atrial muscle cells do not.
b. Specialized cells depolarize during phase 3, but ventricular and atrial muscle cells do not.
c. Specialized cells repolarize during phase 4, but ventricular and atrial muscle cells do not.
d. Specialized cells repolarize during phase 3, but ventricular and atrial muscle cells do not.
e. None are correct

16. Which of the following is the property of a cardiac cell to initiate and fire an action potential on its
own without external stimulation?
a. Selectivity
b. Spontaneity.
c. Automaticity.
d. Conductance.

17. Which of the following does NOT show rapid initial depolarization at the start of an action potential?
a. SA node.
b. Atrial muscle.
c. Purkinje fibers.
d. Ventricular muscle.
e. Bundle of His.

18. Which of the following is INCORRECT concerning diastolic depolarization at the SA node?
a. It results from a decrease in IK.
b. Results from the activation of If carried mainly by Na ions.
c. Its rate is decreased by sympathetic stimulation.
d. Activating IKACh decreases its rate.
e. Its rate is decreased by decreasing ICa

19. If the heart rate increases, which of the following would be a result?
a. ERP would increase while APD would decrease.
b. ERP would decrease while APD would increase.
c. Both ERP and APD would increase.
d. Both ERP and APD would decrease.
20. Which of the following is INCORRECT concerning local circuit flow in the heart?
a. It flows from regions with high membrane potential to regions with low potential.
b. The cell with the more negative potential is called the current source, while the one with the more
positive potential is called the current sink.
c. The circuit flows from the current source to the current sink.
d. The circuit flows from the active cell to the resting cells in its vicinity.

21. Which of the following will increase the length constant involved in the local circuit current flow of
the heart?
a. Increase in membrane resistance.
b. Increase in extracellular resistance.
c. Increase in intracellular resistance.
d. None of the above.

22. Which of the following is FALSE concerning the safety factor in the heart?
a. It is how much the depolarizing local circuit current exceeds what is required to elicit a propagated
action potential.
b. It depends on the characteristics of the current source and current sink.
c. It is higher in cells activated by I Na
d. It is higher in cells activated by I Ca

23. Which of the following would result in an increase in conduction velocity of an impulse through the
heart?
a. Increase Cm
b. Decrease cell diameter
c. Increase the threshold voltage (i.e. make it less negative)
d. Decrease the extracellular resistance

24. Which of the following will decrease the Vmax and sodium current in a myocardial cell?
a. A premature heart beat.
b. Late membrane depolarization.
c. Low extracellular potassium levels.
d. More than normal activated Na channels

25. Which of the following does NOT contribute to the slow conduction found in the SA and AV nodes?
a. Low AP height.
b. Small cell size.
c. High Vmax.
d. Few nexal connection between cells

26. Which of the following is NOT a method of Ca removal following contraction of heart muscle?
a. Na-Ca exchange transports Ca out of the cell.
b. ATP-dependent sarcolemmal Ca pumps remove Ca from the cell.
c. Ca passively diffuses out of the cell.
d. ATP-dependent SR Ca pumps move Ca from the cytoplasm into the SR.

27. Which of the following agents helps to increase the Ca storage capacity of the SR in the heart?
a. dihydropyridines
b. ryanodine
c. calsequestrin.
d. acetylcholine

28. In which of the following does EADs (early afterdepolarizations ) occur more frequently because of
their long action potential duration?
a. Atrial fibers.
b. SA node
c. AV node
d. Purkinje fibers.

29. Which of the following is FALSE concerning the regulation of heart muscle performance?
a. Recruiting more numbers of motor units will increase the strength of contraction.
b. Increases in end-diastolic volume will generate a larger stroke volume during systole.
c. The amount and duration of Ca released from the SR can alter contractility.
d. Positive inotropic agents will increase contractility.

30. Which of the following pairs is INCORRECT?


a. P wave: atrial depolarization
b. QRS complex: ventricular depolarization
c. T wave: ventricular repolarization
d. QT interval: Measure of duration of atrial action potential

31. Which of the following pairs in INCORRECT concerning the Einthoven triangle?
a. Lead I: RA/LA
b. Lead II: RA/LL
c. Lead III: LA/LL
d. All of the pairs are correct.

32. Which of the following is NOT affected by the preload in the heart muscle?
a. End systolic volume
b. End diastolic volume
c. Stroke Volume
d. Ejection fraction.
e. Cardiac output.
33. Which of the following is INCORRECT concerning isotonic contraction in the heart?
a. One occurs during the ejection phase of the cardiac cycle.
b. Is typically the first step in heart muscle contraction followed by an isometric contraction.
c. It involves a change in muscle length against a constant load.
d. An increased afterload will slow the velocity of muscle shortening.

34. Which of the following is NOT an effect of a positive inotropic agent on the heart?
a. It increases stroke volume.
b. It increases the initial velocity of muscle shortening at all loads.
c. It increases the rate of blood ejection from the heart.
d. It decreases the rate of rise in ventricular blood pressure during systole.

35. Which of the following will NOT increase aortic systolic blood pressure?
a. Decrease in arterial compliance
b. Decrease in aortic distensibility.
c. Increase in stroke volume.
d. Decrease in ejection velocity.

36. About perfusion of heart not true is


a. It is 84ml/min,
b. Coronary flow at rest in 250ml/min.
c. At rest heart extracts 70-80% of O2 form each unit of blood given to it,
d. Coronary vessels are compressed during systole

37. Which of the following is a correct formula for the mean arterial blood pressure?
a. MAP = CO X SV
b. MAP = CO X HR
c. MAP = SV X HR X TPR
d. MAP = HR X TPR

38. Which of the following pairs is INCORRECT concerning arteriolar tone?


a. Myogenic control: vascular smooth muscle responds to changes in pressure.
b. Metabolic control: accumulation of waste products stimulates vasoconstriction.
c. Neural control: vasoconstriction activated by sympathetic nerves.
d. Hormonal control: catecholamines affect the adrenergic receptors of the vascular smooth muscle.

39. Which pair is INCORRECT concerning the microcirculation at the capillary level?
a. Tight junctions: brain
b. Gaps large enough to exchange plasma proteins: kidneys
c. Intercellular junctions / pores: exchange of water and lipid-insoluble substances.
d. Pinocytosis: cytoplasmic vesicles exchanging fluid and solutes.
e. Intracellular fenestrations: exchange of large molecules.

40. If a patient is suffering from swollen hands, feet, and face, all of the following could be a cause,
except?
a. Large decrease in arterial blood pressure.
b. Lymphatic blockage.
c. A decrease in plasma proteins.
d. Increase in capillary permeability to proteins.
e. Vasodilation.

41. Which of the following is NOT a mechanism to promote venous return and reduce blood pooling?
a. One way venous valves.
b. Parasympathetic stimulation.
c. Skeletal muscle pump.
d. Thoraco-abdominal pump
e. Pumping of the heart.

42. Which of the following in INCORRECT concerning a patient with ventricular tachycardia?
a. It occurs when the heart rate drops to dangerously low levels.
b. Cardiac output can falls to dangerous levels.
c. Arterial blood pressure falls to dangerous levels.
d. It involves impaired ventricular filling.

43. Which of the following would NOT occur following a major blood loss?
a. Vagal tone to the SA node would be decreased which would then increase the heart rate.
b. An increase in sympathetic output in arterioles would increase the total peripheral resistance,
except in the brain and heart.
c. Sympathetic output to the atria and ventricles would increase resulting in an increase in stroke
volume.
d. Sympathetic output to the veins would increase, resulting in increased venous pressure, venous
return, and cardiac output.
e. Stroke volume would increase while heart rate decreased. Together this would increase the
cardiac output.

44. Effects of hypokalaemia:


a. Short PR interval B. Ventricular extrasystoles
b. Elevated ST segments
c. Long QRS interval
d. Long QT interval
e. Q waves

45. Hypokalaemia:
a. Hyperpolarises membrane
b. Peaked T waves
c. Prolonged QT
d. VEBs
e. ST elevation

46. Hypokalaemia:
a. Hyperpolarizes the membrane
b. Shortens the QRS
c. Shortens the PR interval
d. Depresses the ST segment
e. Prolongs the QT interval

47. Hypokalemia
a. ST segment changes
b. P wave flattening
c. shortened QT
d. No Q wave

48. For two solutions separated by a semi-permeable membrane (Solution A: saline solution AND solution
B: H2O): Which of the following statements is true?
a. A hydrostatic pressure applied to A will stop osmotic pressure
b. There will be bulk flow from A to B
c. The fluid level in B will go up
d. No change

49. Rapid (ingestion/infusion) of 2 litres of normal saline causes:


a. Increased ECF, increased ICF, decreased [Na+]
b. Increased ECF, unchanged ICF, increased [Na+]
c. Unchanged ECF, increased ICF, increased [Na+]
d. Increased ECF, unchanged ICF, unchanged [Na

50. Hyperkalaemia:
a. Causes a prolonged QT interval
b. Prolongs the QRS duration
c. Causes ST segment elevation
d. Potentiates digoxin toxicity

51. ECG changes in hyperkalaemia include:


a. ST depression
b. T wave inversion
c. P wave flattening
d. D. Sinus tachycardia
e. E. Causes loss of P wave

52. In the circulatory system, which is correct?


a. Right ventricle, Semi-lunar valve, Pulmonary artery, Lungs
b. Left ventricle, Semi-lunar valve, Pulmonary artery, Lungs
c. Right ventricle, Semi-lunar valve, Pulmonary Vein, Lungs
d. Left ventricle, Semi-lunar valve, Pulmonary Vein, Lungs

53. If individual has Heart rate of 60, Stroke rate of 150ml Cardiac output is
a. 900ml
b. 9000ml
c. 2.5ml
d. 25ml

54. The highest blood pressure is found in the


a. Venules
b. Aorta
c. Arteries
d. Veins

55. Which ONE of the following statements about intravenous crystalloid solutions is TRUE?
a. Rapid infusion of (?one litre) Hartmann's may cause lactic acidosis
b. Hartmann's 300-308 mosm/kg
c. Hartmann's pH 7.35-7.45
d. N/saline osmol 300-308 mosm/kg
e. 0.9% sodium chloride has a pH 6.5-7.5
f. One litre of Hartmanns solution contains 150 mmol of Na+

56. In a normal cardiac cycle:


a. RA systole precedes LA systole
b. RV ejection precedes LV ejection in expiration
c. RV contraction precedes LV contraction in inspiration
d. Pulmonary valve closes before aortic in expiration.

57. Normal jugular venous pressure c waves occur:


a. Just prior to atrial systole
b. Just after atrial systole
c. During ventricular systole
d. During expiration

58. The c wave in the JVP corresponds most closely with:


a. Peak aortic flow
b. Isovolumetric contraction
c. Isovolumetric relaxation
d. Closure of aortic valve
e. Closure of mitral valve

59. In a normal heart at rest the LV end-systolic volume is:


a. 10 to 30 ml
b. 50 to 70 mls
c. 120 to 150 ml
d. 80 - 100 ml

60. Left ventricular end-diastolic volume is:


a. 10-30 mls
b. 30-50 mls
c. 50-70 mls
d. 70-100 mls
e. 100-130 mls

61. In moderate exercise, the LV end-systolic volume is:


a. 10 mls
b. 30
c. 70 mls
d. 120 mls
e. 140 mls

62. The best site to measure mixed venous pO2 is:


a. SVC
b. RA
c. Pulmonary artery
d. Pulmonary vein

63. Changes with raised intracerebral pressure (ICP):


a. BP increase, HR decrease, RR decrease
b. BP increase, HR increase, RR decrease
c. BP decrease, HR increase, RR increase
d. BP increase, HR decrease, RR increase
e. No change in BP or HR

64. With increased heart rate:


a. Myocardial oxygen demand increases
b. Ratio of systole to diastole increases
c. Vascular filling is unchanged
d. Prolonged AP
e. Decrease in diastolic filling
f. Decrease in coronary perfusion
g. None of the above

65. In exercising muscle, the major increase in blood flow is due to:
a. Sympathetic vasodilatation
b. Metabolic vasodilatation
c. Muscle pumping

66. Which circulation has predominant metabolic control?


a. Renal
b. Liver
c. Lung
d. Splanchnic

67. Local metabolic control is most important in determining flow to:


a. Skin
b. Lung
c. Skeletal muscle
d. Kidney
e. Liver

68. Myocardial ischaemia in shock is due mainly to:


a. Decreased coronary artery pressure
b. Increased myocardial O2 demand
c. Decreased myocardial O2 supply

69. The atrial component of ventricular filling


a. 5%
b. 10%
c. 30%
d. 50%
e. 80%

70. In a 70 kg man 2 metres tall with right atrial pressure of 2 mmHg & aortic root pressure 100 mmHg,
the pressure in the dorsum of the foot is:
a. 0 mmHg
b. 2 mmHg
c. 5 mmHg
d. 30 mmHg
e. >50 mmHg

71. When moving from a supine to an erect position:


a. Mean arterial pressure increases
b. Skin perfusion immediately decreases
c. Decreased renin-angiotensin II
d. Cardiac output increases
e. Increased ADH secretion
f. TPR increases

72. The lowest intrinsic discharge activity resides in the:


a. SA node
b. AV node
c. Bundle branches
d. Purkinje fibres
e. Ventricular fibres

73. Slowest conduction (velocity) occurs in:


a. Atrium
b. AV Node
c. Bundle of His
d. Purkinje Fibres
e. Ventricular muscle

74. The hepatic artery : portal vein blood flow ratio is:
a. 1 : 10
b. 3:1
c. 2:1
d. 1:6
e. 1:3

75. Which ONE of the following causes vasodilatation:


a. TXA2
b. Serotonin (5HT)
c. Endothelin
d. Neuropeptide Y
e. Angiotensin II
f. VIP

76. Which of the following is not a vasodilator?


a. cGRP
b. VIP
c. Neuropeptide Y
d. Bradykinin
e. Acetylcholine

77. Which ONE of the following causes vasoconstriction:


a. Serotonin
b. Prostacyclin
c. Neuropeptide Y
d. Substance P

78. Which ONE of the following is true?


a. Neuropeptide Y secreted by vagus
b. CGRP present in afferent nerves
c. Pancreatic polypeptide is secreted by nerve that ends at pancreas.
d. Substance P is a pain killer

79. Each of the following cause vasoconstriction except:


a. Bradykinin
b. Carotid occlusion
c. Hypovolaemia
d. Valsalva manoeuvre

80. In running 100 metres, the increased oxygen requirements of tissues is met by:
a. Increased cardiac output
b. Increased 2,3DPG
c. Increased erythropoietin
d. Rise in CO2 partial pressure, activating peripheral chemoreceptors No the main benefit is right
shift of the ODC
e. Increased oxygen tension

81. Pressure difference when lying supine is greatest between:


a. Anterior tibial artery and vein
b. Pulmonary artery and vein
c. Femoral vein and right atrium
d. Renal afferent arteriole & renal vein

82. Femoral vein pressure decreased most in standing person by:


a. Taking a step forward
b. Systemic arteriolar constriction
c. Systemic arteriolar vasodilatation
d. Apnoea

83. The highest oxygen extraction is found in the:


a. Carotid body
b. Heart
c. Kidney
d. Brain

84. In order of oxygen extraction from highest to lowest:


a. Heart > Brain > Kidney
b. Kidney > Brain > Heart
c. Kidney > Heart > Brain
d. Brain > Kidney > Heart
e. Heart > Kidney > Brain

85. The LAST part of the heart to depolarise is:


a. Base of the left ventricle
b. Base of the right ventricle
c. The apex of the epicardium
d. The endocardium of the right ventricle

86. The fastest conduction velocity is found in:


a. SA node
b. Atrial muscle
c. AV-node
d. Bundle of His E. Ventricular conduction system/Purkinje system

87. Isovolumetric contraction is closest to:


a. c wave
b. a wave
c. v wave
d. x descent
e. y descent

88. The Fick principle states that:


a. Oxygen uptake as gas is equal to the arterio-venous oxygen difference in flow through the lungs
b. Arterio-venous oxygen difference in the brain multiplied by flow equals oxygen uptake
c. None of the above
89. With a mixed venous oxygen content of 110ml/l and an arterial oxygen content of
150ml/l and oxygen uptake of 280ml/min cardiac output is
a. 5 litres/min
b. 6 litres/min
c. 7 litres/min
d. 8 litres/min
e. 9 litres/min

90. Blood flow at rest is most for


a. Brain
b. Liver
c. Kidney
d. Heart
e. Skin
f. Skeletal muscle

91. Oxygen consumption (in mls/100g/min) is highest for


a. Muscle
b. Brain
c. Kidney
d. Liver
e. Heart

92. During strong (severe) exercise, oxygen consumption is greatest in:


a. Brain
b. Heart
c. Skeletal muscle
d. Liver
e. Kidney
f. Skin

93. The effects on plasma volume of 500 ml blood loss are neutralized within:
a. 1-2 hours
b. 8-10 hours
c. 24 hours
d. 1 week
e. 1 month

94. Venoconstriction occurs EXCEPT during:


a. Lying down
b. Valsalva manoeuvre
c. Carotid sinus compression

95. Coronary blood flow is:


a. Dominant in left coronary artery in 60% of people
b. Better supply to subendocardium in systole
c. Better supply to subendocardium in diastole
d. Better supply to left ventricle in systole
e. Left > right during systole
f. Supply to subepicardium > in LV than RV during systole

96. Compensatory mechanisms in a patient with coarctation of the (descending) thoracic aorta:
a. Lower sympathetic tone in the lower half of the body
b. Decreased total peripheral resistance
c. Increased BP in upper body

97. During a cardiac cycle, the first part of the ventricles to contract is:
a. Apex of left ventricle
b. Base of left ventricle
c. Septum
d. Epicardium at base of left ventricle
e. Right ventricle

98. Beta adrenergic receptors:


a. Described by Lundqvist/Lofgren in 1936/1943
b. At least 3 subtypes are now known

99. When the aortic valve closes, the pressure in the right ventricle is:
a. 0 mmHg
b. 15 mmHg
c. 30 mmHg
d. 50 mmHg
e. 120 mmHg

100. The velocity of blood flow is greatest in:


a. Capillaries
b. Pulmonary vein during diastole
c. Small arteries
d. Inferior vena cava

101. In a 70kg trained athlete at rest:


a. Cardiac output 7 lpm
b. Left ventricular end-systolic volume 60mls
c. Stroke volume 70mls
d. Oxygen consumption 350mls/min
e. a-v O2 extraction 5mlsO2/100mls blood

102. If CO constant & ODC unchanged& O2 consumption constant, Mixed venous oxygen tension
decreased with:
a. Cyanide toxicity
b. Anaemia
c. Decreased temperature
d. Increased CO2

103. Arterial baroreceptor afferents


a. Reach spinal cord via sympathetic nerves
b. Utilise glycine as a neurotransmitter
c. Primary synapse in C1 area of the medulla
d. Activate GABA inhibitory interneurons
e. Excite autonomic efferents in the anterolateral horn

104. Which ONE of the following is true:


a. Right atrial systole and left atrial systole occur at same time
b. Pulmonary valve closes before aortic in inspiration
c. c wave of atrial pressure trace occurs at time of peak aortic pressure
d. RV ejection precedes LV ejection

105. In an average, healthy 70kg male with standing erect with mean arterial BP of 100mmHg:
a. Cerebral venous pressure is approximately 10mmHg
b. Mean arterial pressure at head level is 70mmHg
c. Venous pressure in foot is approximately ?70/?100mmHg
d. Cerebral perfusion pressure 70mmHg

106. During isovolumetric contraction of the ventricles:


a. Aortic blood flow is reversed
b. Coronary blood flow increases
c. The pulmonary valve is not yet shut
d. Aortic pressure is falling
e. When both ventricles reach the same pressure their respective outflow valves open

107. Isovolumetric contraction is associated with:


a. Immediate increase in heart rate due to cardiac sympathetics
b. Cardiac output increased/unchanged
c. Increased systolic BP and decreased diastolic BP
d. Does no work
e. Decrease stroke volume

108. Cerebral blood flow is increased by:


a. Decrease in CSF pressure of 5 mmHg
b. An increase in MAP of
c. Significantly increased by an increase of pCO2 of 5mmHg
d. Plasma glucose > 10 mmol/l
e. Increased regional (or global) neural activity

109. Baroreceptors located in all EXCEPT:


a. Carotid sinus
b. Carotid body
c. Right atrium
d. Aortic arch
e. Large veins

110. The volume of blood is greatest in:


a. Systemic Capillaries
b. Large veins
c. Small arteries
d. The liver
e. The lung

111. Hydrostatic pressure increases in:


a. Arteriolar constriction
b. Venous constriction
c. Capillary dilatation

112. Configuration of an ECG recording:


a. 25 mm / sec, 0.5 mV /cm
b. 25 mm/sec, 1mV /cm
c. 50mm/sec 0.5 mV /cm
d. 50mm/sec 1mv / cm
e. none

113. During exercise in an untrained person, increased cardiac output is mainly due to:
a. Increased heart rate
b. Increased stroke volume
c. Increased venous return

114. Long term control of tissue blood flow includes:


a. Adenosine
b. Nitric oxide
c. Change in tissue vascularity
d. Oxygen tension at the precapillary sphincter
e. something else also short term

115. Peak left ventricular (LV) volume corresponds with (or correlates best with):
a. a wave
b. v wave
c. c wave
d. x descent
e. y descent

116. Cardiac muscle is different from skeletal muscle because:


a. Fast Na Channels
b. Slow Ca Channels
c. Presence of actin and myosin
d. Lower RMP

117. Widened pulse pressure in all except:


a. More rapid ventricular ejection
b. Increased aortic compliance
c. Increased diastolic pressure

118. Adrenaline in VF arrest


a. Increases contractility
b. 'Coarsens' fine VF
c. improves coronary perfusion

119. In a young woman who loses 20% of her blood volume:


a. Decreased diastolic BP
b. Increased serum ADH
c. Increased pulmonary vascular resistance
d. Decreased cerebral blood flow
e. Increased urinary sodium concentration

PART-C

120. Which letter indicates the point in the cardiac cycle that the mitral valve opens?

a. A
b. B
c. C
d. D
e. F

121. Which letter in the image represents the pressure reaction in the left atrium as the mitral valve
bulges during ventricular contraction followed by the stretching of the atrium due to tension of the
AV ring?
a. L
b. K
c. F
d. D

122. Which letter in the image represents the isovolumetric contraction of the left ventricle in the
heart?

a. F
b. B
c. H
d. D

123. Which of the following is a CORRECT order of blood flow through the heart?

1 - Right atrium
2 - Left atrium
3 - Right ventricle
4 - Left ventricle
5 - Vena cava
6 - Pulmonary artery
7 - Pulmonary vein
8 - Tricuspid AV valve
9 - Bicuspid (mitral) AV valve
10 - Pulmonary valve
11 - Aortic valve
12 - Aorta

a. 5, 1, 9, 3, 10, 6, 7, 2, 8, 4, 11, 12
b. 5, 1, 8, 3, 10, 7, 6, 2, 9, 4, 11, 12
c. 5, 1, 8, 3, 10, 6, 7, 2, 9, 4, 11, 12
d. 5, 1, 8, 3, 11, 6, 7, 2, 9, 4, 10, 12

124. For a normal heart, the time taken for arterial systole and diastole are As and Ad seconds,
respectively, while the same for ventricular systole and diastole are Vs and Vd. Which one of the
following equations is correct?
a. As+Ad=Vs+Vd
b. As+Ad >Vs+Vd
c. As+Ad-Vs+vd <0
d. As+Ad < Vs+Vd

125. An isolated carotid sinus was prepared so that the pressure may be regulated by a pump and the
resulting discharge in single carotid sinus nerve fibre could be recorded. The following are the possible
observations.
A. No discharge when carotid sinus perfusion pressure was below 30 mm Hg.
B. Linear increase in discharge frequency when carotid sinus perfusion pressure was gradually
increased from 70 to 110mm Hg.
C. Increase in discharge frequency was more prominent in greater pulsatile changes of carotid sinus
pressure keeping the mean pressure identical in all cases.
D. Increase in discharge was more prominent in the falling phase of pulsatile change of carotid sinus
pressure than in the rising phase.
Which one of the following is correct?
a. A, B and C
b. A and C
c. B and D
d. D only.

126. An organism having heart for circulation excretes through green glands. It has several ganglia and
tactile organs on its body and its larval form is very different than its adult form. This organism is most
likely to respire by:
A) Exchanging oxygen and carbon dioxide through an extensive tracheal system.
B) Gaseous exchange over thinner areas of cuticle or by gills
C) An efficient tracheal system that delivers oxygen directly to the tissues
D) A double transport system, where the circulating fluid contains a dissolved respiratory pigment.
Choose the correct option.
a. A and C
b. Only D
c. Only B
d. B and D
127. Suppose you measure the diameter of the major arteries in a number of species and find that
more active animals have wider aortas. A colleague suggests to you that this would significantly
decrease vascular resistance, thus there will be greater blood flow and oxygen delivery. In reality, it
wouldnt change the resistance of the circulatory system much. The reason would be:
a. Resistance increases drastically as the radius gets smaller. Because arteries are so much wider,
they contribute very little to the total resistance of the system.
b. Resistance increases as the radius gets wider. Because arteries are narrower, they contribute
very little to the total resistance of the system.
c. Resistance to blood flow is not related to the diameter of the arteries. This would contribute
very least to the circulatory system.
d. In reality the the radius of all the arteries are the same. Therefore it will not change the
resistance to the circulatory system.

128. Which of the following are true concerning action potentials and contraction in the myocardium?
(1) The refractory period in a cardiac muscle fiber is very brief.
(2) The binding of Ca2_ to troponin allows the interaction of actin and myosin filaments, resulting
in contraction.
(3) Repolarization occurs when the voltage-gated K_ channels open and calcium channels are
closing.
(4) Opening of voltage-gated fast Na_ channels results in depolarization.
(5) Opening of voltagegated slow Ca2_ channels results in a period of maintained depolarization,
known as the plateau.
a. 1, 3, and 5
b. 2, 3, and 4
c. 2 and 5
d. 3, 4, and 5
e. 2, 3, 4, and 5

129. Michael was brought into the emergency room suffering from a gunshot wound. He is bleeding
profusely and exhibits the following: systolic blood pressure is 40 mmHg; weak pulse of 200 beats per
minute; cool, pale, and clammy skin. Michael is not producing urine but is asking for water. He is
confused and disoriented. What is his diagnosis and why is he not urinating?
a. Hypovolemic shock and there is increase secretion of aldosterone and ADH.
b. Hypervolemic shock and there is increased secretion of aldosterone and ADH
c. Hypovolemic shock and decreased secretion of aldosterone and ADH
d. Hypervolemic shock and decreased secretion of aldosterone and ADH
ANSWER KEYS:

1.c

2.c

3.c

4.c

5.d

6.c

7.a

8.c

9.a

10.c

11.a

12.a

13.e

14.a

15.a

16.c

17.a

18.c

19.d

20.b

21.a

22.d

23.d

24.a
25.c

26.c

27.c

28.d

29.a

30.d

31.d

32.a

33.b

34.d

35.d

36.a

37.c

38.b

39.b

40.a

41.b

42.a

43.e

44.a

45.a

46.a

47.a

48.a

49.b
50.b

51.c

52.a

53.b

54.b

55.d

56.a

57.c

58.a

59.b

60.e

61.c

62.c

63.a

64.b

65.b

66.d

67.c

68.c

69.c

70.d

71.f

72.e

73.b

74.e
75.f

76.c

77.c

78.b

79.a

80.a

81.a

82.b

83.b

84.a

85.b

86.d

87.a

88.b

89.c

90.c

91.e

92.c

93.c

94.b

95.c

96.c

97.c

98.b

99.b
100.b

101.e

102.b

103.d

104.a

105.d

106.d

107.d

108.c

109.b

110.b

111.b

112.b

113.c

114.c

115.c

116.b

117.b

118.c

119.b

Part C

120.d
121.b

122.a

123.c

124.a

125.a

126.c

127.a

128.e

129.a
RESPIRATORY SYSTEM

1. During a tidal expiration,


a. the diaphragm contracts
b. the abdominal muscles contract
c. the scalenes contract
d. the internal intercostals contract
e. there is no muscle contraction

2. PIP > PA
a. during a normal expiration
b. during a forced expiration
c. during a normal inspiration
d. during a forced inspiration
e. never

3. The compliance of an alveolus increases when


a. the pressure increases
b. the radius decreases
c. the amount of surfactant increases
d. the amount of surfactant decreases
e. the surface tension increases

4. A person with a fibrotic lung disease will have


a. increased lung compliance
b. FEV1/FVC < 0.8
c. decreased total lung capacity
d. increased residual volume
e. increased FRC

5. During a severe asthmatic attack, Joes airways have been reduced to their normal radius. By
how much will Joe have to increase the pressure gradient in order to achieve a normal amount of
air flow?
a. 2 times
b. 4 times
c. 8 times
d. 16 times
e. No change it is not possible to change the pressure gradient

6. A man fills his lungs as much as he can, then exhales until his functional residual capacity remains.
Which of the following is true as he exhales?
a. the flow rate of air increases with increased effort
b. the flow rate becomes independent of effort
c. the flow rate is independent of the radius of the airway
d. the typical volume expelled will be 1.2 L
e. abdominal muscle contraction is required to expel this air

7. Which of the following will be true of a previously normal person whose alveolar wall thickens to
twice the normal thickness?
a. PAO2 decreases to half its normal value
b. Oxygenation of blood at rest becomes diffusion-limited
c. Oxygenation of blood during intense exercise becomes diffusion-limited
d. The diffusion rate of CO2 from the blood to the lungs will decrease to half its normal value
e. Both (c) and (d)

8. Jill hikes to the top of a mountain, where Patm = 650 mm Hg. At this location, what will be the PO2 in
her bronchi at the end of a tidal inspiration?
a. 136.5 mm Hg
b. 126.6 mm Hg
c. 89.5 mm Hg
d. 86.6 mm Hg
e. 47.6 mm Hg

9. A man has a tidal volume of 0.5 L and a respiration rate of 10 breaths/minute. He starts breathing
through a 200 ml snorkel. What should be his total minute ventilation in order to maintain the same
alveolar PO2 ?
a. 5 L/min
b. 5.5 L/min
c. 6 L/min
d. 6.5 L/min
e. 7 L/min

10. Which of the following decreases from the top to the bottom of the lung in a person standing
upright?
a. ventilation
b. perfusion
c. V/Q
d. All of the above
e. None of the above

11. Bronchodilation will result from:


a. sympathetic activation
b. a decrease in PO2
c. an increase in PCO2
d. 2 adrenergic receptor activation
e. all of the above

12. Under which of the following conditions is the Hb-O2 dissociation curve shifted to the left?
a. increased temperature
b. presence of 2,3DPG
c. anemia
d. fetal hemoglobin
e. decreased pH

13. At PO2 = 40 mm Hg, normal pH and temperature, Hb is _______ saturated with O2.
a. 0%
b. 25%
c. 50%
d. 75%
e. 98%

14. Which of the following reflects a condition of respiratory acidosis?


a. [HCO3-] = 25 mM, PACO2 = 60 mm Hg
b. [HCO3-] = 12 mM, PACO2 = 36 mm Hg
c. [HCO3-] = 40 mM, PACO2 = 34 mm Hg
d. [HCO3-] = 25 mM, PACO2 = 34 mm Hg
e. [HCO3-] = 24 mM, PACO2 = 40 mm Hg

15. In an anesthetized animal, a cut in the brainstem, above the pons, will result in:
a. a normal breathing rhythm, but irregular depth of breathing
b. deep, prolonged inspirations
c. short, shallow inspirations
d. normal breathing
e. no breathing

16. Which of the following is true of the HCO3-/Cl- transporter?


a. It transports Cl- out of red blood cells in the lungs
b. It transports HCO3- into red blood cells in the tissues
c. It is defective in patients with cystic fibrosis
d. It is located in the apical membrane of gastric parietal cells
e. It is located in the basolateral membrane of pancreatic duct cells

17. The main stimulus (stimuli) for erythropoietin secretion by the kidneys is (are):
a. decreased blood pH
b. increased arterial PCO2
c. decreased arterial PO2
d. (a) and (c)
e. (b) and (c)

18. A drug that inhibits the activity of carbonic anhydrase would:


a. interfere with the formation of carbaminohemoglobin
b. interfere with oxygen binding to hemoglobin
c. cause an increase in venous blood pH
d. decrease the amount of carbon dioxide dissolved in plasma
e. all of the above

19. Emphysema is considered an obstructive lung disease because:


a. vital capacity is decreased
b. dynamic compression of airways is abnormally high
c. lung compliance is increased
d. residual volume is increased
e. all of the above

20. A wounded soldier is in circulatory shock because of bleeding. He is treated with intravenous
plasma, until his blood pressure is stabilized, but his hematocrit is 20%. Which of the following
laboratory findings do you expect after the plasma transfusion:
a. decreased arterial PO2
b. decreased venous PO2
c. decreased arterial oxygen content
d. (b) and (c) only
e. (a), (b), and (c)

21. An increase in PCO2 and a decrease in PO2 in the lungs will cause:
a. Bronchodilation and Vasodilation
b. Bronchodilation and Vasoconstriction
c. Bronchoconstriction and Vasoconstriction.
d. Bronchoconstriction and Vasodilation
e. None of the above.

22. Which of the values below would you expect to find in a sample of air from the trachea of a normal
subject at sea level at functional residual capacity (FRC)?
a. PCO2 = 40, PO2 = 100 mm Hg
b. PCO2 = 0, PO2 = 150 mm Hg
c. PCO2 = 40, PO2 = 40 mm Hg
d. PCO2 = 46, PO2 = 40 mm Hg
e. PCO2 = 47, PO2 = 160 mm Hg
23. The alveolar PO2 calculated with the alveolar gas equation in a normal subject with normal
metabolism will be lowest when the main substrates for energy production are:
a. carbohydrates
b. proteins
c. fats
d. the calculated PAO2 will be the same regardless of energy source
e. only(b) and (c)

24. All of the following histamine effects are mediated by H2- receptors EXCEPT:
a. Vasodilatation
b. Bronchoconstriction
c. Gastric acid secretion
d. Tachycardia

25. Normal value of FEV1 in an adult is


a. 95%
b. 80%
c. 65%
d. 50%

26. The most important gas maintaining alveolar ventilation is


a. oxygen
b. hydrogen
c. carbon dioxide
d. N2.

27. Hyperbaric oxygen is useful in all except


a. congenital heart disease
b. gas gangrene
c. CO poisoning
d. N2 toxicity.

28. Administration of O2 is of value in all except


a. cytotoxic hypoxia
b. stagnant hypoxia
c. anaemic hypoxia
d. histotoxic hypoxia.

29. As one ascends to higher than 3000meters above sea level changes in alveolar pO2 and pCO2 are
as follows
a. decrease in pO2, increase in pCO2
b. decrease in pO2, decrease in pCO2
c. increase in both pO2 and pCO2
d. Increase in pO2, decrease in pCO2.

30. Surfactant is secreted by


a. type 1 pneumatocytes
b. type 2 pneumatocytes
c. goblet cells
d. pulmonary vessels.

31. Which of the following effects is not observed during prolonged stay is space
a. decrease in blood volume
b. decrease in muscle strength
c. increase in red cell mass
d. loss of bone mass
.
32. Which of the following discharge spontaneously during quiet breathing
a. stretch receptors in lung
b. motor neurons in respiratory muscles
c. dorsal respiratory group of neurons
d. ventral respiratory group of neurons

33. Pneumatic center functions primarily to


a. limit inspiration
b. prolong expiration
c. decrease rate
d. discharge inspiratory action potentials

34. Which of the following is the effect of negative G on the eye


a. temporary blinding with redout
b. blackout of vision within few seconds
c. no effect
d. redout and blackout

35. Airway resistance


a. increases in asthama
b. decreases in emphysema
c. increases in paraplegic patients
d. Does not affect work of breathing.

36. Decrease on pCO2, decrease in H+ and increased pO2 causes


a. Hyperventilation
b. Hypoventilation
c. Hypercapnoea
d. Hypoxia

37. Herring-Breur inflation reflex in human being


a. decreases the rate of respiration
b. is not activated until the tidal volume increases above 1.5 lit
c. is an important factor in normal control of ventilation
d. is activated only when tidal volume is les than 1 lit.

38. Total vital capacity is decreased but timed vital capacity is normal in
a. bronchial asthama
b. scoliosis
c. chronic bronchitis
d. all the above.

39. The intrapleural pressure at the end of deep inspiration is


a. - 4mm Hg
b. + 4 mm Hg
c. - 6mm hg
d. + 6 mm Hg

40. Which of the following are incorrectly paired


a. pancreatic alpha amylase-starch
b. elastase-tissue rich in elastin
c. renin-coagulated milk
d. erythropeptidase-polypeptides

41. Which of the following is NOT a function of the lungs?


a. Metabolism.
b. Serves as a reservoir of blood for the left ventricle.
c. It is a filter to protect the systemic vasculature.
d. Facilitates the exchange of O2 and CO2 between air and blood.
e. All of the above are true.

42. Which of the following is in the correct path of CO2 from the tissue to the atmosphere?
a. Reaction with H2O to make H2CO3, dissociation to H+ and HCO3-, H+ combines with imidazole
side chain of hemoglobin, carried back to lungs as HHb+ and HCO3-, reverse reaction forms
CO2.
b. O2 is metabolized to CO2, reaction with H2O to make H2CO3, H2CO3 combines with imidazole
side chain of hemoglobin, H2CO3, Hb+ is carried back to the lungs, reverse reaction forms CO2.
c. Reaction with H2O to make H2CO3, dissociation to H+ and HCO3-, HCO3- combines with
imidazole side chain of hemoglobin, carried back to the lungs as HCO3-Hb+ and H+, reverse
reaction forms CO2.
d. O2 is metabolized to CO2, reaction with H2O to make H2CO3, dissociation to H+ and HCO3-,
carried back to lungs in this form, reverse reaction forms CO2.

43. Which of the following is NOT effector of respiration?


a. Heart.
b. Diaphragm.
c. Intercostals.
d. Trapezius.

44. Which of the following is the first branching of the bronchial tree that has gas exchanging
capabilities?
a. Terminal bronchioles.
b. Respiratory bronchioles.
c. Alveoli
d. Segmental bronchi
e. Alveolar ducts.

45. Which of the ollowing could NOT be part of an acinus?


a. Alveolar sacs
b. Alveolar ducts
c. Terminal bronchioles
d. Respiratory bronchiole.

46. If you increased the left atrial pressure from 5 mmHg to 15 mmHg, what effect would that have on
pulmonary circulation?
a. It would force blood the opposite direction.
b. It would increase the speed at which blood moves through the pulmonary circulation.
c. No change.
d. Blood flow would almost or completely stop.

47. Which of the following concerning average lung volumes and capacities of a person at rest is TRUE?
a. TLC>VC>TV>FRC
b. TLC>FRC>VC>TV
c. TLC>VC>FRC>TV
d. TLC>FRC>TV>VC

48. Which of the following is NOT a normal occurance with increasing age?
a. Vital capacity of the lung decreases.
b. Residual volume increases.
c. Functional residual capacity increases.
d. Inspiratory capacity decreases.
e. Expiratory reserve volume increases.

49. Which of the following spirometry measurements has the greatest sensitivity for detecting early air
flow obstruction?
a. FVC
b. FEV1
c. FFE
d. FEF25-75.

50. Which of the following does NOT happen during inspiration?


a. The ribs move upward.
b. The diaphragm lifts up.
c. The antero-posterior dimensions of the chest are increased.
d. The tranverse dimensions of the thorax are increased.
e. The scalene and sternocleidomastoid muscles can be recruited for inspiration.

51. During inspiration, how does alveolar pressure compare to atmospheric pressure?
a. Alveolar pressure is greater than atmospheric.
b. Alveolar pressure is less than atmospheric.
c. Alveolar pressure is the same as atmospheric.
d. Alveolar pressure is one of the few pressures where the reference pressure is not atmospheric.

52. Which of the following represents the pressure difference that acts to distend the lungs?
a. Alveolar pressure
b. Airway opening pressure
c. Transthoracic pressure
d. Transpulmonary pressure
e. Esophageal pressure.

53. If a patient had a progressive lung disease that required an ever increasing pressure to fill the same
volume of lung, how would the lung's compliance be affected?
a. It would increase it.
b. It would stay the same.
c. It would decrease it.
d. These variables do not affect lung compliance.

54. An asthma sufferer finds she has to breathe at twice her normal rate. How does that affect her
dynamic compliance?
a. It stays the same.
b. It decreases.
c. It increases.
d. Static compliance, not dynamic, is the variable affected by asthma.

55. According to the Law of Laplace, air should flow from the smaller alveoli to the larger, collapsing
them. In the lungs, several factors counter that tendency, and stabilize the alveolar structures. Which
of the following is NOT one of them?
a. Surfactant lowers surface tension to a greater degree when it is on a smaller surface area, allowing
the smaller alveoli to stay open.
b. Mechanical stability is given by surrounding alveoli.
c. Transpulmonary pressure is lower for smaller alveoli, allowing them to stabilize in comparison to
the bigger ones.
d. Surface tension at the gas-liquid interface increases as alveolar surface area increases.

56. Which of the following is FALSE concerning the production and role of lung surfactant?
a. It is part of a lipoprotein called dipalmitoyl phosphatidyl-choline.
b. It is synthesized by alveolar type II cells.
c. As the alveolar surface area decreases during the compression curve, the surfactant decreases
the surface tension at a constant rate.
d. When surfactant density is decreased during expansion, surface tension initially rises rapidly, then
slows down until it reaches the starting point.

57. Which of the following is NOT true concerning respiratory distress syndrome in premature infants?
a. Their ability to synthesize DPPC is limited.
b. Higher pressures are required to ventilate the lungs.
c. Lung compliance is low.
d. Positive pressure respirators are often used to assist them in breathing.
e. Alveoli tend to over expand and sometimes burst at the end of inspiration.

58. Which of the following is NOT true at FRC?


a. It is about 75% TLC.
b. The elastic recoil of the chest wall is outward.
c. The elastic recoil of the lung is inward.
d. The relaxation pressure of the lung and chest wall combined is at atmospheric pressure.

59. If the lung were punctured, which of the following would happen?
a. The lung would collapse on the side of the puncture.
b. Both the lung and the chest wall would collapse on the side of the puncture.
c. The relaxation pressure of the chest wall would increase until it surpassed the atmospheric.
d. The relaxation pressure of the chest wall would increase, but stop before it reached atmospheric
pressure.

60. Which of the following is FALSE concerning the airflow in the lungs?
a. During inspiration and expiration, the flow in the trachea and larger bronchi is turbulent.
b. Towards the middle of the bronchial tree, the flow is turbulent at the branches and laminar in
between.
c. Near the end of the bronchial tree, the flow is laminar.
d. The acini have very small radii which significantly increases the total air flow resistance of the
bronchial tree.

61. Which of the following is FALSE concerning airway resistance?


a. Up to 50% is in the nose.
b. The maximum resistance in the bronchial tree occurs at the fourth generation.
c. In the later generations, the radii are smaller, increasing the total resistance at each successive
generation.
d. Airway resistance can be increased by loss of tissue elasticity and contraction of bronchial smooth
muscles.
62. Which of the following is FALSE concerning the effect of effort on airflow and volume during
inspiration and expiration?
a. During inspiration, greater effort always results in greater flow.
b. Peak expiratory flow occurs at the beginning of expiration.
c. At low and moderate lung volumes, the greater the effort above threshold, the greater the
airflow in expiration.
d. Portions of the expiration curve are effort independent.

63. If the equal pressure point during expiration is in the lobar bronchi, which of the following is TRUE?
a. Expiratory flow would be effort dependent.
b. Expiratory flow would be effort independent.
c. The bronchi beyond the equal pressure point would compress.
d. This situation would only occur during medium and low lung volumes.

64. Which of the following does NOT apply to the alveoli at the base of the lungs?
a. They are less elastic than the alveoli at the apex.
b. The pleural pressure is lower.
c. At FRC they are less inflated than the alveoli at the apex.
d. They are closed at RV.
e. They have a greater volume change than alveoli at the apex during inspiration from FRC.

65. Which of the following is FALSE concerning the closing volume for the lung?
a. Comes between Phase 3 and Phase 4 on the single breath N2 washout curve.
b. Marks the point where the alveoli at the apex close.
c. Marks a sudden increase in nitrogen concentration in the expelled breath.
d. Marks when the overinflated, poorly ventilated alveoli at the apex expel their air with high N2
concentrations.

66. Which of the following is TRUE if a patient breathes slower than normal with increased tidal volumes?
a. More resistive work is done.
b. The total work done decreases.
c. More elastic work is done.
d. Compliance is decreased.

67. Which of the following is INCORRECT concerning the efficiency of breathing and the oxygen
consumption of the respiratory muscles?
a. Efficiency is defined as the ratio of mechanical work done to move air to the amount of
metabolic energy used by the respiratory muscles.
b. The respiratory system uses less than 3% of the body's total oxygen consumption at rest.
c. Respiratory muscles are more efficient than large muscle groups.
d. Emphysema increases the oxygen requirement for respiratory muscles.
e. Hyperventilation can increase the oxygen consumption of respiratory muscles to 30%.

68. If the FIO2 is .21, the FEO2 is .16, the VT is 0.5 L, and the frequency of breathing is 12. What is the
VO2? The equations are VO2=VI * (FIO2 - FEO2) and VI = VT * f.
a. 3.0 L/min
b. 0.75 L/min
c. -0.75 L/min.
d. 0.3 L/min.

69. In what situation would the gas exchange ratio be decreased compared to the respiratory quotient?
a. During slowed breathing.
b. Holding your breath.
c. During hyperventilation.
d. The two are always equivalent.

70. A 140 lb woman would have approximately how much dead space in her lungs?
a. 140 ml.
b. 70 ml.
c. 280 ml.
d. 35 ml.

71. How do you calculate how much inspired air actually ventilates the alveoli during one minute?
a. Subtract the volume of dead space from the tidal volume.
b. Subtract both the dead space volume that was already in the lungs plus the dead space of the
inspired air that won't reach the alveoli from the tidal volume.
c. Subtract the volume of dead space from the tidal volume and multiply it by the number of
breaths per minute.
d. It is equal to the tidal volume times the frequency of breathing.

72. Which of the following is NOT a function of dead space?


a. Warms expired air to body temperature.
b. Saturates inspired air with water vapor.
c. Removes bacteria and other particulate matter.
d. Conducts the warmed air to the respiratory membranes.

73. Calculate pO2 for a person at sea level for R = 0.82 and pCO2 = 40 Torr.
a. 110 Torr.
b. 95 Torr
c. 80 Torr
d. 101 Torr.

74. To which of the following is alveolar pCO2 directly proportional?


a. Rate of CO2 production and alveolar ventilation.
b. Rate of CO2 production and rate of O2 consumption.
c. Alveolar ventilation and rate of O2 consumption.
d. Alveolar ventilation, rate of O2 consumption, and rate of CO2 production.

75. If a patient's blood carries 10 grams of Hb per deciliter, what is the O2 carrying capacity of his blood?
a. 18 milliliters per deciliter.
b. 20 milliliters per deciliter.
c. 10 milliliters per deciliter.
d. 13 millliliters per deciliter.

76. Which of the following definitions is FALSE?


a. O2 content of blood is the actual amount of O2 in one deciliter of blood.
b. O2 saturation of blood is the ratio of O2 content to its O2 capacity.
c. The O2 uptake curve of blood is the functional relationship between O2 content and pO2.
d. The O2 content of blood depends completely on the amount of Hb in the blood.

77. Which of the following statements about Hb is FALSE?


a. A higher P50 than normal means that the O2 binds less tightly to Hb.
b. An increase in 2,3-DPG shifts the O2 uptake curve to the left.
c. An increase in PCO2 causes a right shift of the O2 uptake curve.
d. An decrease in pH increases P50.
e. An increase in temperature shifts the O2 uptake curve to the right.

78. Which of the following is NOT a form by which CO2 can be transported in the blood?
a. As bicarbonate
b. Dissolved in the blood.
c. Bound to the amino end groups in proteins.
d. Bound to the imidazole ring of glutamate.

79. Which of the following in FALSE concerning CO2 uptake?


a. If PO2 = PCO2, then there will be more total CO2 in the blood.
b. Oxygenation moves the CO2 uptake curve downward.
c. The CO2 uptake curve is generated by comparing the total CO2 per unit volume of blood, and the
PCO2.
d. Deoxygenated blood carries less CO2 than oxygenated.

80. Which of the following is INCORRECT concerning the O2 / CO2 movement and processing through the
lungs and tissues?
a. Binding of O2 to Hb changes its configuration so that CO2 and H+ ions are more likely to
dissociate.
b. When CO2 diffused into the alveoli, the PaCO2 is lowered.
c. Carbonic acid is an intermediate in the reaction combining H+ with HCO3- to form H2O and CO2.
d. Arterial blood flows to the tissues where H+ ions combine with HCO3- to form H2O and CO2.

81. Which of the following can cause stagnant hypoxia?


a. COPD
b. Shock or heart failure.
c. Cyanide poisoning.
d. Carbon monoxide poisoning.

82. If you block the blood supply to an alveolus, which of the following would NOT occur as a result?
a. The ventilation perfusion ratio would be 0.
b. The pO2 would be greater than normal.
c. The pCO2 would be 0.
d. All of the above are true.

83. Which of the following is FALSE concerning the ventilation and perfusion of different regions of the
lung?
a. Alveoli at the top of the lung have a smaller dynamic compliance.
b. The Hb moving through the base of the lung is less saturated than that at the apex of the lung.
c. pO2 at the apex of the lung is higher than that at the base of the lung.
d. Regional variation in ventilation-perfusion is more efficient for oxygenating blood than is uniform
ventilation-perfusion.
e. Variation of the ventilation/perfusion ratio in the lungs only becomes significant when lung
function begins to degrade.

84. Which of the following is FALSE concerning the relationships of the variables in diffusion of O2 across
a membrane?
a. Doubling the thickness of the membrane would cut the total flow of O2 in half.
b. Doubling the area of the membrane would double the total flow of O2 .
c. If you increased the alveolar concentration of O2, you would increase the total flow of O2 across
the alveolar membrane.
d. The lower the diffusion coefficient, the higher the total flow.
e. Increasing the arterial concentration of O2 would decrease the total flow of O2.

85. If the blood moved slower than normal through the alveolar capillaries, which of the following would
have an increased uptake?
a. Carbon dioxide.
b. Carbon monoxide.
c. Oxygen
d. None of the above.

86. Which of the following is FALSE concerning diffusion in the lungs?


a. CO is used to measure diffusing capacity because its uptake is diffusion limited.
b. Actual diffusion time includes time required for an O2 molecule to diffuse from the alveolus,
through the membrane, the plasma, and into a RBC.
c. Reaction time is the time it takes the O2 molecule to react with Hb.
d. The diffusion time is greater than the reaction time of an O2 molecule.

87. Which of the following pairs is INCORRECT concerning central nervous systems and a factor they
respond to by affecting respiration?
a. Cerebellum: Mechanoreceptor input
b. Limbic system: emotional states
c. Cerebral cortex: voluntary control
d. Cerebral motor cortex: exercise.

88. Which of the following will NOT increase the minute ventilation?
a. An increase in arterial pH.
b. An increase in arterial partial pressure of carbon dioxide.
c. Increase in alveolar pressure of carbon dioxide.
d. Exercise.
e. Hypoxia.

89. Which of the following is the primary regulating variable of the central chemoreceptors?
a. pO2.
b. pCO2
c. arterial pH
d. Input from stretch receptors.

90. In which situation would the response to hypoxia be limited?


a. In a patient with obstructed airways.
b. When hypoxia is accompanied by hypercapnea.
c. In the hypoxia induced by high altitude.
d. During hypoventilation.

91. Which of the following is FALSE concerning chemoreceptor input to the respiratory centers
a. CSF is a poor buffer and a drop in pCO2 produces a large change in pH initiating a change in
respiration.
b. The body can adjust to chronic hypercapnea by using an active HCO3- transport process in the
choroid plexus.
c. The carotid and aortic bodies detect increases in pCO2 and pH, and decreases in pO2
d. 75% of ventilatory response is regulated by chemoreceptors in the CSF and 25% by the carotid
and aortic bodies.
e. Central chemoreceptors tend to respond slowly over time, while carotid bodies react quickly to
immediate needs.
92. Which of the following pairs is NOT a pulmonary mechanoreceptor paired to a possible stimulus?
a. Stretch receptor: inflation
b. Irritant receptor: inhaled dust
c. Juxtacapillary receptors: decreases interstitial fluid volume in alveolar walls.
d. Bronchial C receptors: large inflations

93. Amount of volume of air in lung after normal expiration is


a. 2200ml,

b. 4900ml,

c. 1000ml,

d. 2700ml.

94. When Spinal cord is secured at the level of brainstem, the effect on respiration will be
a. Caeses,

b. Normal,

c. Irregular,

d. Apneic

95. Which one of the vitamin is useful in treatment of Bronchial Asthma


a. Vit.. A
b. Vit. k
c. Vit. B6
d. Vit. E

96. Which of the following is a normal characteristic of lung?


a. 3,000,000 alveoli
b. Alveolar diameter 3 mm
c. External surface area: 10 m2
d. Alveolar surface area: 5 to 10 m2
e. None of the above

97. A young man collapses one lung. His ABGs on room air would be:
a. pO2 80, pCO2 50 mmHg
b. pO2 50, pCO2 80 mmHg
c. pO2 50, pCO2 50 mmHg
d. None of these is ideal

98. The ABGs in a healthy young 70kg male with one collapsed lung are:
a. paO2 50 mmHg, pCO2 25 mmHg
b. paO2 95 mmHg, pCO2 40 mmHg
c. paO2 60 mmHg, pCO2 45 mmHg
d. paO2 60 mmHg, pCO2 25 mmHg

99. Pulmonary vascular resistance:


a. Is minimal at FRC
b. Increases/?decreases with increase in lung volume
c. Increases with elevated CVP

100. Pulmonary vascular resistance is increased in :


a. Increase in pulmonary arterial pressure
b. Hypocarbia
c. Alkalosis
d. Increased left atrial pressure
e. none

101. The greatest increase in (physiological) dead space would be expected with:
a. Pulmonary embolism
b. Atelectasis (or: collapse of one lung)
c. Pneumothorax
d. Bronchoconstriction
e. Obesity
102. As we go from the top of the erect lung to the bottom:
a. Water vapour pressure remains constant
b. pN2 remains constant
c. pCO2 at apex is higher than at the base
d. pO2 at base is lower than at the apex
e. Difference in V/Q ?
f. Ventilation goes up as go up lung
g. Compliance is more at base than apex

103. Distribution of ventilation & perfusion:


a. Gradient of change in ventilation is greater than that for perfusion
b. Ventilation increases as go up the lung
c. Perfusion increases as go up the lung
d. V:Q ratio at apex is greater than at base
e. None of the above

104. Oxygen unloading:


a. Increases with increased paCO2
b. Decreases with increase in temperature
c. Decreases with increase in 2,3 DPG

105. Alveolar dead space:


a. Is less than physiological dead space
b. Is decreased with mechanical ventilation
c. Is increased with hypotension

106. Alveolar dead space is increased with:


a. Pleural effusion
b. CCF
c. Pneumothorax
d. Hypotension
e. None of the above

107. If dead space is one third of the tidal volume and arterial pCO2 is 45 mmHg, what is the mixed
expired pCO2?
a. 20 mmHg
b. 25 mmHg
c. 30 mmHg
d. 45 mmHg

108. With constant FIO2, CO and VO2, an increase in mixed venous O2 content would be seen with:
a. Hypothermia
b. Increased paCO2
c. Decreased 2,3 DPG
d. Alkalosis
e. None of the above

109. With altitude:


a. Increased 2,3 DPG
b. Increased oxygen unloading in peripheries
c. Increased oxygen uptake in the lungs

110. With acute acclimitisation to altitude:


a. Hypoventilation
b. Decreased cardiac output
c. Pulmonary oedema
d. Polycythaemia
e. Increase in 2,3 DPG

111. Central chemoreceptors:


a. Bathed in CSF
b. Respond to increase in CSF pH
c. Bathed in ECF
d. In medullary respiratory centre

112. The peripheral chemoreceptors:


a. Have a nonlinear response to paO2 changes
b. Have an intact response at 1MAC
c. Respond to a fall in PaCO2
d. Respond to alkalaemia
e. Are in the carotid sinus

113. Peripheral chemoreceptors:


a. In the carotid sinus
b. Have glomus cells
c. Low A-V difference
d. Innervated by glossopharyngeal nerve
e. Blood flow of 200mls/G/min

114. The peripheral chemoreceptors:


a. Have a nonlinear response to paO2 changes
b. Have an intact response at 1MAC
c. Respond to a fall in PaCO2
d. Respond to alkalaemia
e. Are in the carotid sinus
f. Low metabolic rate

115. Surfactant:
a. Causes hysteresis (Or: Is the ONLY cause of hysteresis)
b. Is produced by type 1 pneumocytes
c. Is commonly deficient in term neonates
d. Acts like detergent in water
e. Reduces the amount of negative intrapleural pressure
f. Production is slow
g. Increases pulmonary compliance
116. In quiet breathing, exhalation is:
a. Passive due to elastic tissue alone
b. Passive due to surface tension in the alveoli and elastic tissue recoil
c. Active due to intercostal contraction

117. The normal arterio-venous difference for CO2 is:


a. 2ml/100ml
b. 4ml/100ml
c. 6ml/100ml
d. 10ml/100ml

118. The lung:


a. Removes/inactivates serotonin (5HT)
b. Activates bradykinin
c. Converts angiotensin II to I
d. Inactivates aldosterone
e. Takes up noradrenaline

119. Contribution to the increase in CO2 carriage as blood passes from artery into vein:
Carbamino HCO3 Dissolved

a. 5% 90% 5%
b. 30% 60% 10%

120. Increased physiological dead space with:


a. Age
b. Anaesthesia
c. Supine position
d. All of the above

121. Increased physiological dead space with:


a. Decreases with age
b. Anaesthesia
c. Supine position
d. Calculated from Bohr equation using end-tidal CO2
e. Calculated from endtidal CO2 and arterial CO2
f. Decreases with increase in anatomical dead space
g. Increases with PEEP

122. Physiological dead space increases with


a. Pulmonary hypertension
b. Hypotension
c. Atelectasis
d. Pleural effusion
e. None of the above

123. Shunt can be calculated by knowing:


a. Cardiac output
b. Arterial oxygen content
c. Mixed venous oxygen content
d. End pulm. capillary oxygen content
e. All of the above

124. Alveolar pressure:


a. Is always negative throughout normal quiet breathing
b. Is zero (atmospheric pr) during pause between inspiration and expiration
c. Is greater than 5-6 cm H2O during quiet expiration
d. Is less than 5-6 cms H2O during quiet inspiration

125. Patient with chronic airflow limitation:


a. Independent of elastic recoil
b. Will have increased total lung capacity
c. Has increased static compliance.
d. Show decreased elastic recoil

126. One lung anaesthesia:


a. High FIO2 will completely correct paO2
b. CPAP will completely correct paO2
c. Supine position will give better VQ matching
d. Associated with hypercarbia

127. The partial pressure of oxygen in dry air at sea level:


a. 163 mmHg
b. 159 mmHg
c. 149 mmHg
d. 100 mmHg

128. Work of breathing (as % of total VO2) in normal healthy adult::


a. 1%
b. 5%
c. 10%
d. 20%
129. PEEP causes:
a. Variable effect on FRC
b. Reduced lung compliance
c. Decrease in lung water
d. Reduces airway resistance

130. PEEP:
a. Has a variable effect on FRC
b. Reduced lung compliance
c. Reduces lung water
d. Reduces airway resistance

131. At an atmospheric pressure of 247 mmHg, what is the moist inspired p02?
a. 200 mmHg
b. 2 mmHg
c. 40 mmHg
d. 50 mmHg

132. Type II pneumocytes


a. Develop from type I pneumocytes
b. Are macrophages
c. Are very flat and practically devoid of organelles
d. Metabolise surfactant

133. Type I pneumocytes


a. Give rise to Type II pneumocytes
b. Are flat & minimal organelles
c. Bind surfactant (? receptors) on their brush border

134. Control (inspiratory) of the diaphragm originates in:


a. Pneumotactic centre
b. Apneustic centre in pons
c. Dorsal medullary (neurons of) respiratory centre
d. Ventral medullary (neurons of) respiratory centre

135. For a normal Hb-O2 dissociation curve, the most correct relationship is:
a. PaO2 340mmHg, SaO2 99%
b. PaO2 132mmHg, SaO2 98%
c. PaO2 68mmHg SaO2
d. PaO2 60mmHg, SaO2 91%
e. None of the above
136. Oxygen toxicity:
a. Is caused by superoxide dismutase (OR: Increased by increased SOD)
b. Causes CNS toxicity at over 100kPa
c. Is caused by absorption atelectasis
d. Is due to formation of superoxide radicals
e. Prolonged ventilation at 50kPa causes pulmonary toxicity
f. Causes lipid peroxidation

137. Pulmonary stretch receptors:


a. Are only stimulated by maintained stretch
b. Show (slow) adaptation
c. Cause an immediate decrease in tidal volume.
d. Cause an immediate increase in tidal volume

138. The peripheral chemoreceptors are located:


a. Carotid sinus
b. Carotid bodies
c. The vasomotor centre.
d. diencephelon

139. Mixed Venous Blood


a. Higher haematocrit than arterial
b. SaO2 48%
c. Higher pH than arterial Blood
d. Best sampling site RA
e. pO2 lower than coronary sinus blood
f. Coronary sinus O2 saturation of 30%.

140. Carbon dioxide carriage:


a. 10% dissolved
b. 30% carbamino
c. 85% bicarbonate
d. 60% bicarbonate
e. Unaffected by pO2

141. Factors that favour formation of carbamino-haemoglobin include:


a. Carbonic anhydrase
b. A decrease in oxygen tension
c. An increase in oxygen tension
d. A decrease in pH
e. None of the above
142. In the lung, airway resistance
a. Mainly in small airways
b. Varies with change in lung volume
c. Increased by stimulation of adrenergic receptors
d. Can be measured by flow rate divided by pressure difference between mouth and alveolus
e. Increased by breathing helium-oxygen mixture

143. The effect of decreasing airway diameter has the following effect on airway resistance:
a. 1/8
b.
c.
d. 4 times
e. 16 times

144. Gas composition of air?


PO2; PCO2 ;PN2 P ;other gases

a. 20.98 O.4
b. 20.98 0.4
c. 21 0.04
d. 20.98 0.04 78.58 0.42
e. 20.98 0.04 78.2 0.98

145. Closing capacity


a. Increases with age
b. 10% vital capacity
c. Decreases with age.
d. Not dependent on age

146. Measurement of Functional residual Capacity (FRC):


a. Helium dilution does not measure unventilated spaces on chest
b. Body plethysomography inaccurate if high FIO2 used
c. Helium used to decrease airflow viscosity

147. The absolute humidity of air saturated at 37C:


a. 760 mmHg
b. 47 mmHg
c. 100%
d. 44g/m3
e. 17mg/m3

148. What are the respiratory organs in insects?


a. Gills
b. Trachea
c. Lungs
d. Book Lungs
e. Skin

149. In the snails, the aquatic respiration is effected by


a. Gills
b. Lungs
c. Ctenidium
d. Airtubes
e. Skin

150. In earth worms and leeches, gaseous exchange takes place through
a. Nostrils
b. Skin
c. Gills
d. Lungs
e. Air tubes

PART-C

151. Which of the following would cause oxygen to dissociate more readily from hemoglobin? (1) low
PO2, (2) an increase in H_ in blood, (3) hypercapnia, (4) hypothermia, (5) low levels of BPG (2,3-
bisphosphoglycerate).
a. 1 and 2
b. 2, 3, and 4
c. 1, 2, 3, and 5
d. 1, 3, and 5
e. 1, 2, and 3

152. Which of the following statements are correct? (1) Normal exhalation during quiet breathing is an
active process involving intensive muscle contraction. (2) Passive exhalation results from elastic recoil
of the chest wall and lungs. (3) Air flow during breathing is due to a pressure gradient between the
lungs and the atmospheric air. (4) During normal breathing, the pressure between the two pleural
layers (intrapleural pressure) is always subatmospheric. (5) Surface tension of alveolar fluid facilitates
inhalation.
a. 1, 2, and 3
b. 2, 3, and 4
c. 3, 4, and 5
d. 1, 3, and 5
e. 2, 3, and 5
153. Which of the following factors affect the rate of external respiration? (1) partial pressure
differences of the gases, (2) surface area for gas exchange, (3) diffusion distance, (4) solubility and
molecular weight of the gases, (5) presence of bisphosphoglycerate (BPG).
a. 1, 2, and 3
b. 2, 4, and 5
c. 1, 2, 4, and 5
d. 1, 2, 3, and 4
e. 2, 3, 4, and 5

154. A 1-year-old boy is brought to the emergency department because he is having difficulty
breathing after drinking water that contains nitrate. He is cyanotic. Blood drawn for laboratory studies
is chocolate-colored. His hemoglobin most likely has which of the following properties?
a. Decreased glycosylation
b. Increased binding of 2,3-bisphosphoglycerate
c. Increased proportion of Fe3+
d. Presence of Beta subunits only
e. Sickle cell mutation

155. The figure shows the oxygen binding curves for hemoglobin (Hb) and Myoglobin(Mb)

Identify the correct curves for Hb and Mb

a. P: Mb; Q:Hb
b. Q:Mb; P:Hb
c. R:Mb; Q:Hb
d. S:Mb; R:Hb

156. A 10-month-old boy with hemoglobin SS disease is brought to the emergency department by his
parents because of severe pain in his feet for 2 hours. He appears acutely ill and is crying. Vital signs
are normal except for a pulse of 160/min. Physical examination shows splenomegaly and edematous,
tender feet. Laboratory studies show a hemoglobin concentration of 7 g/dL and a reticulocyte count
of 12%. A peripheral blood smear shows target cells, poikilocytes, and sickled erythrocytes. Which of
the following processes most likely caused the sickled erythrocytes in this patient?
a. Decreased interaction of hemoglobin S with spectrin upon deoxygenation
b. Inability of abnormal beta-chains to form tetramers upon deoxygenation
c. Inability of abnormal beta-chains to form tetramers upon oxygenation.
d. Increased interaction of hemoglobin S with spectrin upon oxygenation
e. Spontaneous polymerization of hemoglobin S upon deoxygenation
f. Spontaneous polymerization of hemoglobin S upon oxygenation
destroy

157. When we sprint, pumping air in and out of our lungs can become a substantial energy cost. For
fish, moving water across their gills when they are sprinting and is no more difficult. The possible
reason would be
a. Fishes have unidirectional flow.
b. Gills are not as developed as lungs, they have simple anatomy.
c. Fishes use the dissolved air.
d. Fishes are small in size than human, therefore volume of air required is very less.

158. Even after air has been in our lungs for a while, it never becomes fully deoxygenated. In fact, the
oxygen pressure doesnt drop much below 100mmHg. How come the blood cannot extract all the
oxygen. Why?
a. The structural change in the hemoglobin do not allow the complete extraction of oxygen
b. Alveoli of the lungs use some amount of oxygen.
c. The plasma oxygen tension reaches equilibrium with the oxygen tension of air.
d. Oxygen is used for cellular respiration by the lungs, so blood plasma cannot utilize the oxygen.

159. Match the following


A deficiency of oxygen at the tissue level Ventilation-perfusion coupling

Above-normal partial pressure of carbon-di-oxide Hypercapnia

Normal quiet breathing Hypoxia

Hypoxia-induced vasoconstriction to divert Eupnea,


pulmonary blood from poorly ventilated to well-
ventilated regions of the lungs

a. A-3, B-4; C-1; D-2


b. A-4, B-3; C-1; D-2
c. A-3, B-1; C-4; D-2
d. A-2, B-4; C-1; D-3
160. Which of the following causes is resulted in this shift of hemoglobin-oxygen curve?

A. increase 2,3-BPG
B. Increase CO2
C. Decrease pH
D. Increased temperature
a. Only A
b. Only C
c. A, B and D
d. B and D only
ANSWER KEYS:

1.e

2.e

3.c

4.c

5.d

6.a

7.e

8.b

9.e

10.c

11.e

12.d

13.d

14.a

15.d

16.a

17.c

18.c

19.b

20.d

21.b

22.a
23.c

24.b

25.b

26.c

27.d

28.d

29.b

30.a

31.c

32.c

33.a

34.a

35.a

36.b

37.b

38.b

39.a

40.d

41.e

42.a

43.d

44.c

45.c

46.d

47.c
48.e

49.d

50.b

51.b

52.d

53.c

54.b

55.c

56.c

57.e

58.a

59.d

60.d

61.c

62.c

63.a

64.a

65.b

66.c

67.c

68.d

69.c

70.a

71.c

72.a
73.d

74.b

75.d

76.d

77.b

78.d

79.d

80.d

81.b

82.a

83.d

84.d

85.b

86.d

87.a

88.a

89.b

90.c

91.c

92.c

93.a

94.d

95.c

96.e

97.d
98.d

99.a

100.e

101.a

102.a

103.d

104.a

105.a

106.b

107.c

108.e

109.a

110.e

111.c

112.a

113.c

114.a

115.g

116.b

117.b

118.a

119.b

120.a

121.g

122.b
123.e

124.b

125.d

126.d

127.b

128.a

129.d

130.d

131.c

132.d

133.b

134.c

135.d

136.d

137.b

138.b

139.a

140.c

141.e

142.b

143.e

144.e

145.a

146.a

147.d
148.b

149.c

150.b

PART C

151.e

152.b

153.d

154.c

155.a

156.e

157.a

158.c

159.a

160.c
NERVOUS SYSTEM

1. The substance that contributes maximally to the osmolality inside the cell is
a. Protein.
b. Phosphate
c. Urea
d. Potassium.

2. The resting membrane potential of a cell


a. is dependent on the permeability of the cell membrane to K+ being greater to Na+
b. falls to zero if Na+/K+ ATPase in membrane is inhibited.
c. is equal to the equilibrium potential for K+
d. d. is equal to the equilibrium potential of Na+.

3. In some cases DM is due to


a. excessive receptors
b. antibodies against receptors
c. deficiency of receptors for extra cellular proteins
d. Deficiency of nucleotide regulatory G proteins.

4. Many substances are removed from the cell to outside by


a. Pinocytosis
b. Chemotaxis
c. Phagocytosis
d. Exocytosis

5. Excessive formation of a substance/ secretion in the body is controlled in order to maintain


homeostasis by
a. +ve feedback mechanism
b. -ve feedback mechanism
c. Osmosis
d. Haemodynamics.

6. An action potential in a nerve is


a. terminated by influx of Na+ excessive receptors
b. terminated by efflux of K+
c. initiated by efflux of Na+
d. is initiated by influx of K+

7. Milieu interior " is a term introduced by


a. Laplace
b. Boyle
c. Claud Bernard
d. Lansteiner
8. An example of co-transport is
a. Na+-K+ pump
b. Ca++ pump
c. Na+- H+ pump
d. Na+ glucose transport.

9. Which of the following has slowest conduction?


a. Alpha nerve fibres
b. Gamma-nerve fibres
c. B nerve fibres
d. C nerve fibres.

10. Saltatory conduction


a. is seen only in myelinated nerve fibres
b. is slower that non saltatory conduction
c. is not affected if a local anesthetic is applied to the node of Ranvier
d. none of the above

11. Myelin sheath is produced by


a. Axoplasm
b. Mitochondria
c. scawann cell
d. muscle cell

12. The action potential of skeletal muscle


a. has a prolonged plateau phase
b. spreads inwards to all parts of the muscle via T tubules
c. is longer than the action potential of cardiac muscle
d. is not essential for muscle conduction.

13. Smooth muscle need help of


a. calmodulin for contraction
b. acetyl choline for contraction
c. K+ for contraction
d. monoamine oxidase for contraction.

14. The likely mechanism through which neostigmine acts in improving muscular weakness is
a. It blocks action of acetylcholine
b. it interferes with action of mono-amine oxidase
c. it enhances the action of catecholamines
d. it blocks the action of acetylcholine esterase.

15. A skeletal muscle


a. obeys all or none phenomenon
b. becomes less excitable when its membrane becomes hyperpolarized
c. has a resting membrane potential positive inside
d. Contains excessive Na+ in intracellular compartment.
16. S.A. node is the pacemaker of heart because of
a. location in the right atrium
b. neural control
c. natural leakiness to Na+
d. Natural leakiness to k+

17. Absolute refractory period in the heart


a. corresponds to the duration of relaxation
b. lasts till half of cardiac contraction
c. shorter than refractory period in skeletal muscle
d. Lasts till cardiac contraction.

18. First heart sound occurs during the period of


a. isometric relaxation
b. isotonic relaxation
c. isovolumetric contraction
d. isovolumetric relaxation.

19. Which of these vessels does not have sympathetic control


a. Cerebral
b. Splanchnic
c. Cardiac
d. Cutaneous.

20. Blood brain barrier is made up of


a. Astrocytes
b. Oligodendrocytes
c. Oligodendroglia
d. Microglia.

21. Lesions of which of the following nuclei cause hypothalamic obesity


a. ventromedial nucleus
b. dorsomedial nucleus
c. suprachiasmatic nucleus
d. supraoptic nucleus.

22. The EPSP


a. is an all or none response to a presynaptic potential
b. can be temporarily summated during repetitive presynaptic stimulation
c. always initiates an action potential
d. lasts only for the duration of presynaptic action potential

23. Following statements are true for dopamine except


a. it is related to Parkinsonism
b. it is found in the cells uninhibited by Ach in basal ganglia
c. it is one of the endogenous opiates from CNS
d. it cannot be replaced in CNS from dietary dopamine.
24. Loss of fear and emotion is often observed in the lesion at
a. septal nucleus
b. thalamus
c. amygdaloidal nucleus
d. Sensory cortex.

25. The Renshaw cell


a. receive recurrent collaterals from motor neurons and inhibit other motor neurons in the
vicinity
b. is the inhibitory system of cerebellum
c. are a major component of muscle spindle
d. are present in retina

26. Premotor cortex refers to


a. some areas anterior to primary motor cortex causing complex co-ordinate movements
like speech, eye movements
b. an area of motor cortex responsible for voluntary movements
c. an area in temporal cortex
d. an area of cerebellum.

27. Functions of limbic system are all EXCEPT


a. Olfaction
b. Gestation
c. feeding behaviour
d. sexual behaviour

28. REM is
a. characterised by delta waves on ECG
b. a sound and dreamless sleep
c. characterised by total lack of muscular activity
d. referred to as paradoxical sleep

29. Sleep deprivation


a. can cause psychotic episodes
b. is associated with sluggishness of thoughts
c. makes a person more alert
d. has no effect on the individual

30. The sympathetic system


a. has short post ganglionic fibres
b. consists of vagus nerve
c. produces nicotine at its nerve endings
d. has a thoraco-lumbar outflow from the spinal cord

31. Visceral pain


a. shows relatively rapid adaptation
b. is mediated by beta fibres in dorsal root of spinal nerves
c. can sometimes be relieved by applying irritant to skin
d. can be produced by prolonged stimulation of touch receptors.

32. The naked nerve endings are responsible for the sensation of
a. Pain
b. Touch
c. hearing
d. vision

33. When a normally innervated skeletal muscle is stretched the initial response is contraction, with
increase in the stretch sudden relaxation occurs because of
a. decrease in gamma efferent discharge
b. inhibition of the discharge from annulospiral endings of afferent nerve fibres
c. decreased activity of afferent nerve fibres from golgi tendon organs
d. increased activity of afferent nerve fibres from golgi tendon organs

34. After anterolateral cordotomy relief of pain is due to interruption of


a. left dorsal column
b. left ventral spinothalamic tract
c. right lateral spinothalamic tract
d. left lateral spinothalamic tract

35. Parasympathetic system


a. has short preganglionic fibres
b. secretes dopamine
c. controls most of the movements and secretions of gut
d. brings increase in heart rate during exercise

36. In a health adult sitting with eyes closed the EEG rhythm observed with electrodes on occipital
lobes
a. Alpha
b. Theta
c. Delta
d. Beta

37. The basal ganglia are primarily concerned with


a. sensory integration
b. short term memory
c. control of movement
d. neuroendocrine control

38. Interruption of motor pathways in the internal capsule on one side causes
a. spastic paralysis on the same side
b. spastic paralysis on the opposite side
c. flaccid paralysis on the same side
d. flaccid paralysis on the opposite side

39. the extrapyramidal system is not concerned with


a. stretch reflex
b. righting reflex
c. spasticity
d. sensation of viscera

40. Thirst is stimulated by


a. increase in plasma osmolality and volume
b. increase in plasma osmolality and decrease in volume
c. decrease in osmolality and increase in volume
d. decrease in plasma osmolality and volume

41. Lesions of which of the following hypothalamic nuclei cause loss of circadian rhythm
a. Ventromedial
b. Dorsomedial
c. Suprachiasmatic
d. Supraoptic

42. Normal blood flow to the brain is


a. greatly modified by vasomotor control
b. about 150ml/min
c. about 750ml/min
d. greatly increased during exercise

43. Retrograde amnesia


a. is abolished by prefrontal lobectomy
b. responds to drugs that block dopamine receptors
c. is commonly precipitated by a blow on the head
d. is commonly precipitated by ageing

44. Non fluent aphasia is produced by lesion of


a. Brocas area
b. angular gyrus
c. parietal lobe
d. frontal lobe.

45. Choose one correct answer.


a. The resting membrane potential of most body cells is positive inside and negative outside.
b. If the resting membrane potential is made mode negative it is called hyopolarizartion.
c. If membrane potential moves in the positive direction it is called depolarization.
d. If you put both the measuring electrode (eg, microelectrode) and reference inside of a
cell, you will measure a negative voltage.

46. Choose one correct answer.


a. Phase 0 of a cardiac action potential is the rapid repolarization phase of the action
potential.
b. Phase 1 of a cardiac action potential is the upstroke of a cardiac action potential.
c. During phase 4 of a cardiac action corresponds to the plateau phase of the action
potential.
d. None are correct.
47. Choose one correct answer.
a. If efflux of an ion exceeds influx, the ion will accumulate in the cell.
b. The flux of an ion across a cell membrane is equal to the product of membrane
conductance to the ion and the driving forces acting on the ion.
c. Normally, the only driving force acting on ions is the membrane potential.
d. None are correct.

48. If the resting membrane potential is 100mV and the sodium equilibrium potential is +40mV then
what is the value of the electrochemical gradient (ie, total driving force) acting on sodium ions?
Choose one correct answer.
a. +60 mV
b. 60 mV
c. 140 mV
d. None are correct.

49. Choose one correct answer.


a. The lipid bilayer portion of the cell membrane acts like a resistor.
b. If the membrane conductance to an ion is very high and the total electrochemical gradient
is zero there will be no net movement of the ion through the membrane.
c. If the concentration gradient of an ion across the membrane is inwardly directed and the
electrical gradient is equal and opposite, there will be net movement of the ion into the
cell.
d. None are correct.

50. Choose one correct answer.


a. An increase in plasma potassium concentration will cause resting membrane potential to
become more negative.
b. Cardioplegia solutions contain a low concentration of potassium.
c. The upstroke of the nerve action potential is generated by Na influx via the Na-K pump.
d. Action potentials repolarize because the net ionic current is outward.

51. Choose one correct answer.


a. A reduction in sodium current will increase action potential conduction velocity in nerve.
b. An increase in cell diameter will increase action potential conduction velocity in nerve.
c. An increase in membrane capacitance will increase conduction velocity in heart cells.
d. None are correct.

52. Choose one correct answer.


a. Skeletal muscle cells are electrically connected via gap junctions.
b. Local circuit currents are not required for action potential propagation.
c. An increase in action potential amplitude will decrease conduction velocity.
d. None are correct.

53. Choose one correct answer.


a. The refractory period of nerve, heart and skeletal muscle is due to inactivation of sodium
channels.
b. The space constant is an index of how far local circuit currents can spread in nerve, heart
and skeletal muscle.
c. Closure of gap junctions will increase conduction velocity in heart.
d. None are correct.

54. Choose one correct answer.


a. Heart muscle cells receive motor neuron innervation.
b. Heart cells are held together at the intercalated disk.
c. Transverse tubules directly connect the interstitial fluid to the cytoplasm of the cell.
d. None are correct.

55. Which of the following is NOT an inward current?


a. Na
b. Ca
c. K
d. I Na-K (pump)

56. Which of the following is NOT true of the parasympathetic control of the heart?
a. It affects muscarinic receptors.
b. It decreases heart rate via the SA node.
c. It decreases conduction velocity via the AV node.
d. It can be blocked by beta blockers, e.g. propranolol.

57. Sodium enters the cell during the upstroke of the action potential. What is the major mechanism
for removing Na from the cell?
a. It passively diffuses out of the cell.
b. It is extruded via an ATP-dependent Na-K pump.
c. It is extruded via an ATP-dependent Na-Ca pump.
d. It is extruded via an ATP-independent Na-K pump.
e. It is extruded via an ATP-independent Na-Ca pump.

58. If sodium channels are open, membrane potential is +110 mV, and the equilibrium potential is
+68 mV, how will sodium ions respond?
a. They will move into the cell.
b. They will move out of the cell.
c. They will not move.
d. The conductance will drop to zero.

59. 9th Cranial nerve is


a. Vagus
b. Hypoglosal
c. Glassopharyngeal
d. Accessory

60. Largest Cranial nerve of the body is


a. Vagus
b. Hypoglosal
c. Trigeminal
d. Abducent

61. Longest Cranial nerve of the body is


a. Vagus
b. Hypoglosal
c. Trigeminal
d. Abducent

62. Trigeminal Cranial nerve is


a. Sensory
b. Motor
c. Mixed
d. None

63. Regulatory centre of Vomiting is


a. Cerebral
b. Cerebellum
c. Hypothalamus
d. Medulla oblongata

64. Regulatory center of Knowledge is


a. cerebral
b. cerebellum
c. Hypothalamus
d. medulla oblongata

65. Regulatory centre of Peristalsis is


a. Cerebral
b. Cerebellum
c. Hypothalamus
d. Medulla oblongata

66. 11th Cranial nerve is


a. Vagus
b. Trochlear
c. Glassopharyngeal
d. Accessory

67. Shortest Cranial nerve of the Body is


a. Vagus
b. Trochlear
c. Trigeminal
d. Abducent

68. Thinnest Cranial nerve of the Body is


a. Vagus
b. Trochlear
c. Trigeminal
d. Abducent

69. Accercelin is Blood Clotting factor no.


a. 4
b. 5
c. 6
d. 7

70. which cranial nerve is not responsible for eye ball muscles movement
a. Occulomotor
b. Optic
c. Trochlear
d. Abducen

71. Loss of tongue movement is due to defect in which cranial nerve


a. Trigeminal
b. Glosso pharyngeal
c. Hypoglossal
d. All the above

72. Accessory Cranial nerve is


a. Sensory
b. Motor
c. Mixed
d. None

73. Specific gravity of CSF is -


a. 1005
b. 1010
c. 1025
d. 1017

74. Inability to understand, speak or read is:


a. Dyslexia
b. Aphasia
c. Disconnection syndrome
d. Ataxia

75. Action potential is:


a. Decremental phenomenon
b. Doesnt obey all or none phenomenon
c. K+ goes from ECF to ICF
d. Threshold stimulus is required

76. Which of the following is INCORRECT concerning the voltage clamp technique?
a. The voltage clamp technique is used to record action potentials from heart cells
b. It is useful to measure effects of antiarrhythmic agents on specific ion channels.
c. The voltage clamp can be used to set the Vm either for a segment of the cell membrane
or the entire cell.
d. It allows the investigator to measure both the magnitude and time course of the ionic
current generated by ions moving through channels.

77. Which of the following is NOT true of an inward ionic current?


a. It is negative.
b. It reduces the polarity of the cell.
c. It makes the membrane potential more negative.
d. It can be caused by Na ions moving into the cell
e. It can be caused by anions moving out of the cell.

78. If you reduced the outward current in a heart cell, what would be the result?
a. Increase the rate of repolarization.
b. Prolong the action potential.
c. Increase the rate of depolarization.
d. decrease the rate of depolarization

79. Which of the following is NOT a factor affecting local circuit current in the heart?
a. Ri
b. Rmyo
c. Ro
d. Cm
e. Rk
80. Synaptic transmission in auto nomic ganglia is due to
a. a.Cholinergic,
b. b. Nicotinic,
c. c. Muscarine,
d. Dopaminergic

81. Transmitter at neuromuscular functions


a. a.Nicotine
b. Atropine
c. Acetylcholine
d. Noradrenaline
82. Frontal lobe lesion causes are except
a. Acalculia,
b. Emotional alteration,
c. Personality changes,
d. B+C

83. EEG with eyes open, mind wandering shows


a. Alpha waves,
b. beta waves,
c. Delta waves,
d. Theta waves

84. In the C.S.F. protein level is elevated and sugar reduced in


a. T.B.M
b. AIDS
c. Hepatitis
d. C.R.F.

85. Parasympathetic stimulation of tissues in walls of bronchioles causes:


a. Bronchoconstriction
b. Bronchodilation
c. Increased blood flow
d. Dilatation of alveoli

86. A cell is put in ECF which is highly permeable to Na+, false is


a. Na+ Moves Along the Concentration Gradient,
b. The Potential Across the Cell Membrane Drops to 70mv,
c. Na+ Comes out of the Cell,
d. The Electrogenic Potential is 0 at Isoelectric pH.

87. The Nernst equation represents the potential at which:


a. Electrical neutrality exists
b. Concentration of ions on each side of membrane equal
c. Potential at which there is no net movement of ions
d. Balance of chemical & electrical forces
e. Both sides are equiosmolar

88. Shivering that is ?mediated by the hypothalamus:


a. muscle spindle to increase tone
b. via red nucleus
c. rhythmic stimulation of anterior horn cells
d. Activation of shivering centre in brainstem

89. Transection of a motor nerve leads to:


a. Muscle fibre hypertrophy
b. Increased/decreased RMP
c. Increased/decreased receptors
d. Increased spontaneous muscle activity

90. The setpoint in temperature regulation control the body's response to changes in temperature.
Location of sensory receptor which regulates the setpoint?
a. Anterior hypothalamus
b. Posterior hypothalamus
c. Spinal cord
d. Skin
e. Great veins

91. Cerebrospinal fluid (CSF):


a. Production is 150 ml / day
b. Volume is 50 ml
c. Produced by choroidal blood vessels and ependymal cells
d. All the above

92. Which ONE of the following is characteristic of type A nerve fibres:


a. Nociception
b. Slower conduction than C fibres
c. Myelinated
d. Substance P
e. Sensory only

93. Assertion: The imbalance in concentration of Na+, K+ and proteins generates resting potential.
Reason: To maintain the unequal distribution of Na+ and K+, the neurons use electrical energy.
a. Both Assertion and Reason are true and Reason is the correct explanation of the
Assertion.
b. Both Assertion and Reason are true but the Reason is not the correct explanations of
Assertion.
c. Assertion is true, but Reason is false.
d. Both Assertion and Reason are false.

94. If vagus nerve in man is damaged, which of the following will not be affected?
a. pancreatic secretion
b. gastrointestinal movements
c. tongue movements
d. cardiac movements

95. The function of vagus nerve innervating the heart is to


a. Initiate the heart beat
b. Reduce the heart beat
c. Accelerate the heart beat
d. Maintain constant heart beat

96. When you have a toothache, you feel pain because _______________.
a. there is a cavity in your tooth
b. tiny bits of food are left between your teeth
c. bacteria digest the food left between your teeth and produce an acid
d. the cavity reaches the nerves and the nerves send a message to the brain

97. Cornea transplant in humans is almost never rejected. This is because:


a. It is a non-living layer
b. Its cells are least penetrable by bacteria
c. It has no blood supply
d. It is composed of enucleated cells

98. Any quick response to stimuli that by passes the brain.


a. reflex action
b. voluntary action
c. knee jerk
d. involuntary action

99. The cerebrum is the largest part of your brain and is composed of two hemispheres (right and
left). It controls your _______________.
a. voluntary movement, speech, hearing, vision, memory, reasoning
b. breathing, heart rate, blood pressure
c. balance and coordination
d. involuntary control of respiration

100. The brain stem is the base of the brain that connects the brain to the spinal cord. It
controls your _______________.
a. voluntary movement, speech, hearing, vision, memory, reasoning
b. balance and coordination
c. breathing, heart rate, blood pressure
d. state of consciousness

101. Which of the following best describes a fast EPSP synaptic event?
a. The depolarization persists several seconds to minutes after the termination of the
presynaptic transmitter release.
b. Acetylcholine activation of nicotinic cholinergic post-synaptic receptors results in a
depolarization lasting less than 50 msec.
c. reguires K+ channel closure
d. requires K+ channel opening

102. A researcher wanted to study the nerve impulse conduction in cat. He takes the neuron
from cat and incubates in a Ca2+ free culture medium. To his surprise he did not find any nerve
impulse that is conducting. What could be the reason for this observation?
a. The neurotransmitters are not released.
b. The calcium pump is not expressed.
c. The secretary vesicles are not formed in the axon terminal.
d. The cat requires calcium for synthesis of neuron

PART-C

103. Which of the following statement is Correct?


A. Sympathetic nervous system stimulates secretion and parasympathetic nervous system
inhibits secretion by salivary amylase
B. Sympathetic nervous system dialates and parasympathetic nervous system constricts the
pupil of eye
C. Sympathetic nervous system decreases and parasympathetic nervous system increases
heart rate
D. Sympathetic nervous system stimulates and parasympathetic nervous system inhibits
intestinal peristalsis.
a. A and B,
b. B only
c. C only
d. A and D

104. A man falls into deep sleep with one arm under his head. After awakening the arm is
paralyzed but tingling sensation and pain sensation persists. This loss of motor function without
the loss of sensory function is due to
a. A fibres are more susceptible to hypoxia that B
b. A fibres are more sensitive to pressure than C
c. C fibres are more sensitive to pressure than A
d. Sensory nerves are nearer bone and hence affected by pressure.
105. After falling down from a staircase a young woman is found to have partial loss of
voluntary movement on the right side of the body and loss of pain and temperature sensation on
the left side below the mid-thoracic region. The probable site of lesion is
a. transection of the right half of the spinal cord in the upper thoracic region
b. transection of the left side of the spinal cord in the upper thoracic region
c. transection of sensory and motor pathways on the right side of the pons
d. transection of the left half of the spinal cord in the lumbar region

106. Which of the following statements are true?


(1) The sensory function of the nervous system involves sensory receptors sensing certain changes
in the internal and external environments.
(2) Sensory neurons receive electrical signals from sensory receptors.
(3) The integrative function of the nervous system involves analyzing sensory information, storing
some of it, and making decisions regarding appropriate responses.
(4) Interneurons are located primarily in the PNS.
(5) Motor function involves the activation of effectors (muscles and glands).
a. 1, 2, 3, and 4
b. 2, 4, and 5
c. 1, 2, 3, and 5
d. 1, 2, and 4
e. 2, 3, 4, and 5

107. A neurons resting membrane potential is established and maintained by


(1) a high concentration of K_ in the extracellular fluid and a high concentration of Na_ in the
cytosol,
(2) the plasma membranes higher permeability to Na+ because of the presence of numerous Na+
leakage channels,
(3) differences in both ion concentrations and electrical gradients, (4) the fact that there are
numerous large, nondiffusible anions in the cytosol,
(5) sodiumpotassium pumps that help to maintain the proper distribution of sodium and
potassium.
a. 1, 2, and 5
b. 1, 2, and 3
c. 2, 3, and 4
d. 3, 4, and 5
e. 1, 2, 3, 4, and 5

108. Place the following events in a chemical synapse in the correct order:
(1) release of neurotransmitters into the synaptic cleft,
(2) arrival of nerve impulse at the presynaptic neurons synaptic end bulb (or varicosity),
(3) either depolarization or hyperpolarization of postsynaptic membrane,
(4) inward flow of Ca2+ through activated voltage-gated Ca2+ channels in the synaptic end bulb
membrane,
(5) exocytosis of synaptic vesicles,
(6) opening of ligand-gated channels on the postsynaptic plasma membrane,
(7) binding of neurotransmitters to receptors in the postsynaptic neurons plasma membrane.
a. 2, 1, 5, 4, 7, 6, 3
b. 1, 2, 4, 5, 7, 6, 3
c. 2, 4, 5, 1, 7, 6, 3
d. 4, 5, 1, 7, 6, 3, 2
e. 2, 5, 1, 4, 6, 7, 3

109. Match the following:


A. the recovery of the resting membrane 1. Excitatory postsynaptic potential
potential

B. a neurotransmitter-caused depolarization 2. repolarization


of the
C. postsynaptic membrane

D. a neurotransmitter-caused 3. Absolute refractory period


hyperpolarization of the
E. postsynaptic membrane

F. time during which a neuron cannot 4. Inhibitory postsynaptic potential


produce an action potential even with a
very strong stimulus

a. A-4; B-3; C-1; D-2


b. A-2; B-1; C-4; D-3
c. A-3; B-1; C-4; D-2
d. A-2; B-3; C-4; D-1

110. Which of the following statements are true?


1. If the excitatory effect is greater than the inhibitory effect but less than the threshold
of stimulation, the result is a subthreshold EPSP.
2. If the excitatory effect is greater than the inhibitory effect and reaches or surpasses
the threshold level of stimulation, the result is a threshold or suprathreshold EPSP
and one or more nerve impulses.
3. If the inhibitory effect is greater than the excitatory effect, the membrane
hyperpolarizes, resulting in inhibition of the postsynaptic neuron and the inability of
the neuron to generate a nerve impulse.
4. The greater the summation of hyperpolarizations, the more likely a nerve impulse will
be initiated.
a. 1 and 4
b. 2 and 4
c. 1, 3, and 4
d. 2, 3, and 4
e. 1, 2, and 3

111. Which of the following statements are true?


(1) The basic types of ion channels are gated, leakage, and electrical.
(2) Ion channels allow for the development of graded potentials and action potentials.
(3) Voltage-gated channels open in response to changes in membrane potential.
(4) Ligand-gated channels open due to the presence of specific chemicals.
(5) A graded potential is useful for communication over long distances.
a. 1, 2, and 3
b. 2, 3, and 4
c. 2, 3, and 5
d. 2, 3, 4, and 5
e. 1, 3, and 5

112. Match the following

1. temporal
A. composed primarily of myelinated axons
summation

B. bundles of axons and associated connective


tissue and blood vessels in the PNS 2. white matter

C. extensive neuronal networks that help


3. nerve
regulate the digestive system
D. results from the buildup of neurotransmitter
from the rapid, successive release by a single
4. enteric plexus
presynaptic end bulb

a. A-4; B-3; C-1; D-2


b. A-2; B-1; C-4; D-3
c. A-3; B-1; C-4; D-2
d. A-2; B-3; C-4; D-1

113. Which of the following statements are true?


(1) The frequency of impulses and number of activated sensory neurons encodes differences in
stimuli intensity.
(2) Larger-diameter axons conduct nerve impulses faster than smaller-diameter ones.
(3) Continuous conduction is faster than saltatory conduction.
(4) The presence or absence of a myelin sheath is an important factor that determines the speed
of nerve impulse propagation.
(5) Action potentials are localized, but graded potentials are propagated.
a. 1, 3, and 5
b. 3 and 4
c. 2, 4, and 5
d. 2 and 4
e. 1, 2, and 4.

114. Which one of the following is an example of negative feedback loop in humans?
A. Constriction of skin blood vessels and contraction of skeletal muscles when it is too cold
B. Secretion of tears after falling of sand particles into the eyes
C. Salivation of mouth at the sight of delicious food
D. Secretion of sweat glands and constriction of skin blood vessels when it is too hot
a. A only
b. A and B
c. C only
d. B and D

115. Neurotransmitters are removed from the synaptic cleft by


(1) axonal transport,
(2) diffusion away from the cleft,
(3) neurosecretory cells,
(4) enzymatic breakdown,
(5) cellular uptake.
a. 1, 2, 3, and 4
b. 2, 4, and 5
c. 2, 3, and 4
d. 1, 4, and 5
e. 1, 2, 3, 4, and 5

116. Match the following


A. neurons with just one process extending 1. microglia
from the cell body; are always sensory
neurons
B. small phagocytic neuroglia 2. astrocytes

C. small phagocytic neuroglia 3. oligodendrocytes


D. provide myelin sheath 4. unipolar neurons
E. for CNS axons
a. A-4; B-1; C-2; D-3;
b. A-4; B-3; C-2; D-1;
c. A-2; B-1; C-4; D-3;
d. A-4; B-3; C-1; D-2;

117. An experimenter was wanted to study the role of Ion channels in nerve impulse
conduction. The induce a mutation in the Voltage-gated K+ channels of a neuron so that they
opened at the same time as the Na+ channels. What effect could an experiment find?
a. The nerve would not depolarize at all.
b. The nerve will depolarize highly
c. The nerve will hyperpolarize
d. The nerve will slightly depolarize

118. Match the following:


A. contains neuronal cell bodies, dendrites, axon
1. nucleus
terminals, unmyelinated axons and neuroglia
B. a cluster of cell bodies within the CNS
2. gray matter
C. form CSF and assist in its circulation; form
blood cerebrospinal barrier 3. multipolar neurons
D. neurons having several dendrites and one axon;
most common neuronal type 4. ependymal cells

a. A-4; B-3; C-1; D-2


b. A-2; B-1; C-4; D-3
c. A-3; B-1; C-4; D-2
d. A-2; B-3; C-4; D-1

119. Unidirectional transmission of a nerve impulse through nerve fibre is due to the fact that:
A. nerve fibre is insulated by a medullary sheath
B. sodium pump starts operating only at the cyton and then continues into the nerve fibre
C. neurotransmitters are released by dendrites and not by axon endings
D. neurotransmitters are released by the axon endings and not by dendrites
a. A and B
b. B only
c. D only
d. B and C

120. Match the following

A. neurons with one main dendrite and one axon;


1. Schwann cells
found in the retina of the eye
B. provide myelin sheath for PNS axons 2. ganglia
C. support neurons in PNS ganglia
3. bipolar neurons

D. a cluster of neuronal cell bodies located outside


4. satellite cells
the brain and spinal cord
a. A-4; B-3; C-1; D-2
b. A-2; B-1; C-4; D-3
c. A-3; B-1; C-4; D-2
d. A-2; B-3; C-4; D-1

121. Consider a normal neuron at rest. Suppose there is slightly more Cl- outside than inside.
Under what circumstances, if any, would Cl- not diffuse across the membrane, even if many Cl-
channels were open and there was a gradient?
a. If the electrical potential was negative inside and equal in energy to the concentration
gradient driving Cl- in.
b. If the electrical potential was positive inside and equal in energy to the concentration gradient
driving Cl- in.
c. If the electrical potential was negative inside and the energy is less than the concentration
gradient driving Cl- in.
d. If the electrical potential was positive inside and the energy is less than the concentration
gradient driving Cl- in.

122. Match the following:


A. a sequence of rapidly occurring events 1. graded potential
that decreases and eventually reverses
the membrane potential and then
restores it to the resting state; a nerve
impulse
B. a small deviation from the resting 2. Relative refractory period
membrane potential that makes the
membrane either more or less polarized

C. period of time when a second action 3. threshold


potential can be initiated with a very
strong stimulus
D. the minimum level of depolarization 4. action potential
required for a nerve impulse to be
generated

a. A-4; B-1; C-2; D-3


b. A-2; B-1; C-4; D-3
c. A-3; B-1; C-4; D-2
d. A-2; B-3; C-4; D-1

123. If you have ever been driving and nod for a brief moment or two and then pull your
head back up, you can probably thank your ______ for its role in arousal and preparing the
forebrain for sensory input.
a. brainstem
b. occipital lobe
c. hypothalamus
d. reticular formation

124. Match the following:


A. polarization that is less negative than the 1. spatial summation
resting level
B. results from the buildup of 2. Hyperpolarizing graded potential
neurotransmitter released simultaneously
by several presynaptic end bulbs
C. the hyperpolarization that occurs after 3. depolarizing graded potential
the repolarizing phase of an action
potential
D. polarization that is more negative than 4. after-hyperpolarizing phase
the resting level
a. A-4; B-3; C-1; D-2
b. A-2; B-1; C-4; D-3
c. A-3; B-1; C-4; D-2
d. A-2; B-3; C-4; D-1

125. Match the following:


A. the part of the neuron that contains the 1. Nissl bodies
nucleus
B. and organelles
C. rough endoplasmic reticulum in neurons; 2. synaptic vesicles
site of protein synthesis

D. store neurotransmitter 3. axon


E. the process that propagates nerve 4. cell body
impulses toward another neuron, muscle
fiber, or gland cell

a. A-4; B-1; C-2; D-3;


b. A-4; B-3; C-2; D-1;
c. A-2; B-1; C-4; D-3;
d. A-4; B-3; C-1; D-2;

126. Match the following


A. the highly branched receiving or input 1. myelin sheath
portions of a neuron

B. a multilayered lipid and protein covering 2. dendrites


for axons produced by neuroglia

C. the outer nucleated cytoplasmic layer of 3. initial segment


the Schwann cell
D. first portion of the axon, closest to the 4. neurolemma
axon hillock

a. A-4; B-3; C-1; D-2


b. A-2; B-1; C-4; D-3
c. A-3; B-1; C-4; D-2
d. A-2; B-3; C-4; D-1

127. Match the following


A. site of communication between two 1. neurofibrils
neurons or
B. between a neuron and an effector cell

C. form the cytoskeleton of a neuron 2. nerve fiber

D. gaps in the myelin sheath of an axon 3. synapse

E. general term for any neuronal process 4. Node of Ranvier

a. A-4; B-3; C-1; D-2


b. A-2; B-1; C-4; D-3
c. A-3; B-1; C-4; D-2
d. A-2; B-3; C-4; D-1

128. Match the following


area where the axon joins synaptic cleft
the cell body

area where nerve impulses arise axon hillock


the numerous fine processes trigger zone
at the ends of an axon and
its collaterals

interstitial fluid-filled space axon terminals


separating two neurons

a. A-4; B-3; C-1; D-2


b. A-2; B-1; C-4; D-3
c. A-3; B-1; C-4; D-2
d. A-2; B-3; C-4; D-1

129. The buzzing of the alarm clock woke Carrie. She stretched, yawned, and started to
salivate as she smelled the brewing coffee. She could feel her stomach rumble. List the divisions
of the nervous system that are involved in each of these actions.
a. Smelling the coffee and hearing the alarm are somatic sensory, stretching and yawning
are somatic motor, salivating is autonomic (parasympathetic) motor, stomach rumble is
enteric motor.
b. Smelling the coffee and hearing the alarm are enteric motor, stretching and yawning are
somatic motor, salivating is autonomic (parasympathetic) motor, stomach rumble is
enteric motor
c. Smelling the coffee and hearing the alarm are autonomic (parasympathetic) motor,
stretching and yawning are somatic motor, salivating is somatic sensory, stomach
rumble is enteric motor
d. Smelling the coffee and hearing the alarm are relay neurons, stretching and yawning are
somatic motor, salivating is autonomic (sympathetic) motor, stomach rumble is enteric
motor.

130. Baby Ming is learning to crawl. He also likes to pull himself onto window sills, gnawing on
the painted wood of his century-old home as he looks out the windows. Lately his mother, an
anatomy and physiology student, has noticed some odd behavior and took Ming to the
pediatrician. Blood work determined that Ming had a high level of lead in his blood, ingested from
the old leaded paint on the window sill. The doctor indicated that lead poisoning is a type of
demyelination disorder. What could be the reason for odd behaviour?
a. loss of motor control and coordinated body movement.
b. Hyperactivation of neurons which cause restlessness.
c. Lead poisoning interferes with neurotransmitters.
d. Motor neuron failure.

131. As a torture procedure for his enemies, mad scientist Dr. Moro is trying to develop a drug
that will enhance the effects of substance P. What cellular mechanism is assigned to substance P?
a. It is bodys natural pain killer.
b. Substance P causes the sensory neurons numb.
c. Motor neurons are affected by substance P
d. It increases the pain perception

132. A nerve impulse or action potential is generated from transient changes in the
permeability of the axon membrane to Na+ and K+ ions. The depolarization of the membrane
beyond the threshold level leads to Na+ flowing into the cell and a change in membrane potential
to a positive value. The K+ channel then opens allowing K+ to flow outwards ultimately restoring
membrane potential to the resting value. The Na+ and K+ channels operate in opposite directions
because
a. There is an electro chemical gradient growth generated by proton transfer.
b. There is a difference in Na+ and K+ concentrations on either side of the membrane
c. Na+ is a voltage-gated channel, whereas K+ is ligand-gated
d. Na+ is dependent on ATP whereas K+ is not.

133. A monkey undergoes cerebellectomy. After the post-operative recovery, the monkey was
given a task to press a bar. The possible observations are:
A. Its hand would overshoot the target while reaching the bar.
B. It would be unable to move forelimbs.
C. It would show intention tremor while trying to press the bar.
D. It would press the bar with mouth instead of hand.
Which one of the following is correct?
a. A and C
b. B only
c. D only
d. B and D

134. Spinal cord of an animal was transected at C1/C2 level. The respiration of the animal
stopped and it needed artificial respiration. However, the heart continued to beat although at a
slower rate. Some of the explanations given were:
A) respiration regulatory centre is located in the medulla.
B) respiration regulatory centre is located above the C1/C2 cut.
C) heart regulatory centre is above the C1/C2 cut.
D) heart has autoregulation.
Which one of the following is most appropriate?
a. A only.
b. B and C only.
c. A, B and D only
d. B, C and D only.
ANSWER KEYS:

1.c

2.a

3.b

4.d

5.b

6.b

7.c

8.d

9.d

10.a

11.c

12.b

13.a

14.d

15.a

16.c

17.d

18.c

19.a

20.a

21.a

22.b
23.c

24.

25.a

26.a

27.b

28.d

29.a

30.d

31.c

32.a

33.d

34.d

35.c

36.a

37.c

38.b

39.d

40.d

41.c

42.c

43.c

44.b

45.c

46.d

47.b
48.c

49.b

50.d

51.b

52.d

53.a

54.b

55.d

56.d

57.b

58.b

59.c

60.c

61.a

62.c

63.d

64.a

65.d

66.d

67.d

68.b

69.c

70.c

71.c

72.b
73.a

74.a

75.d

76.a

77.c

78.b

79.e

80.b

81.c

82.a

83.a

84.a

85.a

86.c

87.c

88.a

89.d

90.b

91.d

92.c

93.c

94.c

95.b

96.d

97.c
98.a

99.a

100.c

101.b

102.a

PART C

103.b

104.b

105.a

106.c

107.d

108.c

109.b

110.e

111.d

112.d

113.e

114.a

115.c

116.a

117.d
118.b

119.c

120.c

121.a

122.a

123.d

124.c

125.a

126.b

127.c

128.d

129.a

130.a

131.d

132.b

133.a

134.d
SENSE ORGAN

1. Function of cochlea is:


a. Auditory function
b. Nerve deafness
c. Basilar membrane is tuned
d. Equilibruin is disturbed

2. Bitots spot is found in


a. Xeropthalmia
b. Rickets
c. Osteomalacia
d. Typhoid

3. In order to perceive a sensation, the following conditions are necessary, with the exception of
_______________.
a. a strong enough stimulus to initiate a response
b. a receptor to convert the stimulus to an impulse
c. the interpretation of the impulse in the brain
d. a motor impulse to an effector organ or muscle

4. Which one of these is not an exteroceptor?


a. chemoreceptors in tongue and nose
b. photoreceptors of the eye
c. proprioceptors
d. tactile receptors

5. Of this list of cutaneous receptors, the _____________ are located in the lower dermis and detect
deep pressure and stretch.
a. organs of Ruffini
b. bulbs of Krause
c. free nerve endings
d. root hair plexuses

6. Phantom pain occurs because ________________.


a. the patient has received a fright, which initiates pain impulses in certain people
b. it is actually referred pain that has been interpreted incorrectly by the brain
c. the severed neurons of an amputee still function and send pain impulses
d. certain diseases cause chronic pain in many areas of the body, even those not affected
by the disease

7. Taste buds are found only in the _______________ and _____________ papillae.
a. vallate; fungiform
b. vallate; filiform
c. fungiform; filiform
d. posterior; filiform
8. Within the eye, the _________________ contains sebaceous and sudoriferous glands that produce
the "sleepers" or "sleep dust" at night.
a. palpebral fissure
b. medial commissure
c. lateral commissure
d. lacrimal caruncle

9. Modified sweat glands, also called ___________________, are located within the eyelids.
a. sebaceous glands
b. tarsal glands
c. ciliary glands
d. conjunctiva

10. Of the intrinsic (smooth) muscles within the eye, the ________________ causes the pupil to become
larger.
a. ciliary muscle
b. pupillary dilator muscle
c. pupillary constrictor muscle
d. medial rectus muscle

11. The choroid of the eye carries out each of these functions, with the exception of
________________.
a. houses blood vessels to carry nutrients and remove wastes
b. having a dark pigment to keep light rays from bouncing within the eye
c. housing the photoreceptors
d. All of these are functions of the choroid.

12. The area of keenest vision, where there is a high concentration of cone cells, is called the
_______________.
a. fovea centralis
b. macula lutea
c. blind spot
d. optic disc

13. The tectal system, which is responsible for body-eye coordination, is comprised of the
_________________ and the motor pathways they activate.
a. optic chiasma
b. geniculostriate system
c. striate cortex
d. superior colliculi

14. Between the scala vestibuli and scala tympani of the cochlea is the ______________, a triangular
middle chamber filled with endolymph and housing the spiral organ.
a. membranous ampulla
b. cochlear duct
c. semicircular canal
d. cupula
15. ______________ afflicts the inner ear and may cause both hearing loss and equilibrium disturbance
as the autonomic nervous system constricts vessels in the inner ear.
a. acute purulent otitis media
b. otosclerosis
c. Meniere's disease
d. Strabismus

16. Of the following processes, which occurs in dendrites?


a. interpretation
b. transmission
c. stimulation
d. transduction

17. Nociceptors sense


a. pressure
b. pain
c. heat
d. touch

18. The taste that most people sense on the back of the tongue is
a. sweet
b. bitter
c. salty
d. sour

19. Whether a hair cell depolarizes or hyperpolarizes depends on whether it bends toward or away from
a. kinocilium
b. ampullae
c. statocysts
d. otolith

20. In the cochlea, the hair cells are contained by the


a. tectorial membrane
b. tympanic membrane
c. basilar membrane
d. vestibular membrane

21. The amount of light entering the eye is determined by the size of the
a. retina
b. cornea
c. fovea
d. optic nerve

22. The cells responsible for color vision in vertebrates are called
a. rod cells
b. cone cells
c. bipolar cells
d. cupula cells
e. ampullae

23. Which of the following substances is involved in the electromagnetic and chemical events of vision?
a. rhodopsin
b. G proteins
c. phosphodiesterase
d. all of the above
e. none of the above

24. Which of the following is not a mechanical stimulus?


a. gravity
b. sound
c. touch
d. vibration
e. smell

25. Nerve impulses are carried from the eye to the brain by the
a. optic nerve
b. cornea
c. bipolar cells
d. rod and cone cells
e. vestibular apparatus

26. The membrane that separates the outer ear from the middle ear of mammals is called the
a. foveal membrane
b. basilar membrane
c. oval window
d. tectorial membrane
e. tympanic membrane

27. Which of the following is not one of the four taste classes that taste buds respond to?
a. bitter
b. salty
c. spicy
d. sweet

28. Rod cells and cone cells are located in the


a. organ of Corti
b. retina
c. iris
d. cornea
e. pupil

29. Which of the following sensory systems is not found in fish?


a. ampullae of Lorenzini
b. lateral line system
c. pit organs
d. taste buds
e. all of the above are found in fish

30. The simplest sensory receptors in a nervous system are


a. found only in invertebrates
b. free nerve endings that depolarize in response to direct physical stimulation
c. mechanical receptors that employ a lever device
d. auditory receptors

31. Which of the following sensory systems of vertebrates did not evolve first in an aquatic
environment?
a. infrared vision
b. regular vision
c. electric organs
d. taste

32. The most sensitive vertebrate chemoreceptors known are the


a. rod and cone cells of mammals
b. taste receptors of fishes
c. olfactory receptors of mammals
d. organs of Corti of bats

33. Compared with chemical stimuli, auditory stimuli


a. travel farther
b. travel more quickly
c. provide better directional information
d. all of the above
e. none of the above

34. The Eustachian tube connects the


a. outer ear and middle ear
b. middle ear and inner ear
c. inner ear and throat
d. cochlea and semicircular canals
e. middle ear and throat

35. Which of the following sensory receptors function via hyperpolarization rather than depolarization?
a. rod and cone cells
b. organs of Corti
c. proprioceptors
d. all of the above
e. none of the above

36. The fovea is part of the


a. cornea
b. iris
c. papillae
d. retina
e. organ of Corti
37. Which of the following is not one of the ossicles in the middle ear of humans?
a. stirrup
b. anvil
c. saddle
d. hammer

38. Which of the following stimuli can be detected by a proprioceptor?


a. taste
b. pain
c. color
d. gravity
e. all of the above

39. Information transmitted from the pit organs of a snake is processed by


a. modified muscles
b. the auditory center of the snake's brain
c. electroplates
d. the visual center of the snake's brain
e. magnetic receptors

40. Receptors that report your arm position and tension are:
a. electroreceptors.
b. exteroceptors.
c. interoceptors.
d. proprioceptors.
e. thermoreceptors.

41. Which is a functional mismatch?


a. Pacinian corpuscles -- detects warmth
b. Meissner's corpuscles -- detects light touch of the skin
c. semicircular canals -- detects rotational movements in the three dimensions of space
d. statocyst -- a gravity receptor
e. vestibular apparatus -- detects changes in speed or direction of movement

42. In which of the following animals would you expect to find the protein opsin?
a. fruit fly
b. alligator
c. bird
d. human
e. all of the above

43. Which of the following statements is correct?


a. In order to produce a clear image, the aqueous humor is involved in the process of
accommodation.
b. The space anterior to the lens is filled with the vitreous humor.
c. The function of the crystalline lens is to bend light rays and focus them on the optic
nerve.
d. The amount of light entering your eye is regulated by smooth muscle in the ciliary body.
e. Nearsightedness is a condition resulting from loss of lens elasticity.

44. Which of the following statements is incorrect?


a. The human eye contains both rhabdomeric receptors, the rods, and ciliary receptors,
the cones.
b. Rhabdomeres are found in the compound eyes of insects, but not in the eyes of octopi.
c. In mammals, color vision depends upon three types of cones that are maximally
sensitive to three different wavelengths of light.
d. Suffering from a Vitamin A deficiency leads to night blindness.
e. Accommodation involves the ability to change the width of the lens by contracting or
relaxing certain muscles.

45. Muscle spindles are stretch receptors.


a. True
b. False

46. The sclera of the eye contains a dark pigment that absorbs light, preventing internally reflected light
from blurring the visual image.
a. True
b. False

47. The olfactory nerve carries impulses associated with taste.


a. True
b. False

48. The transparent cornea is an extension of the sclera.


a. True
b. False

49. The human inner ear is where sound waves are amplified by means of the vibrations of tiny bones.
a. True
b. False

PART-C
50. A 1 meter tall object was placed 10meter in front of a normal eye. The size of the image on the
retina will be (consider distance between lens and retina 1.7cms)
a. 0.17mm
b. 1.7mm
c. 3.4mm
d. 170mm

51. The stereocilia of auditory hair cells are arranged in rows but the heights of stereocilia are not the
same in all the rows. Though the height of stereocilia is the same within a particular row, the heights
increase in subsequent rows. When the stereocilia of shorter rows are mechanically pushed towards
the taller rows, the hair cells are depolarized but a push on opposite direction hyperpolarizes them.
The significance of this graded height of stereocilia is:
A. Each row of stereocilia may be displaced independent of other rows in physiological
conditions.
B. The tip of the taller stereocilia will show greater displacement as compared to shorter ones
when all the rows are moving in the same axis.
C. The hair cells will be depolarized or hyperpolarized in different grades when the axis of
displacement is changed.
D. The taller stereocilia are involved with depolarization and shorter ones are responsible for
hyperpolarization.
Which one of the following is correct?
a. A only
b. B only
c. B and C only
d. A and D

52. A person has been suffering from night blindness. On consultation, the doctor advised the person to
eat carrots and /or cod fish oil. After some time having seen no improvement, the doctor gave the
person Vitamin A injection. Still no marked improvement was seen. The doctor mooted several
suggestions indicating lack of the following enzymes for the failure of treatment:
A. Retinol dehydrogenase
B. Retinal reductase
C. Retinal isomerase
D. Retinal synthase
Which one of the following is correct?
a. A only
b. B only
c. B and C
d. C and D

53. Which of the following are true?


(a) The sites of olfactory transduction are the olfactory hairs.
(b) The olfactory bulbs transmit impulses to the temporal lobe of the brain.
(c) The axons of olfactory receptors pass through the olfactory foramina in the cribriform
plate of the ethmoid bone.
(d) The olfactory nerves are bundles of axons that terminate in the olfactory tracts.
(e) Within the olfactory bulbs, the first-order neurons synapse with the second order
neurons.
a. 1, 2, and 4
b. 2, 3, 4, and 5
c. 1, 2, 3, 4, and 5
d. 1, 3, and 5
e. 1, 2, 3, and 5

54. Which of the following statements is incorrect?


1. Olfactory receptors respond to the chemical stimulation of an odorant molecule by
producing a receptor potential.
2. Basal stem cells continually produce new olfactory receptors.
3. Adaptation to odors is rapid and occurs in both olfactory receptors and the CNS.
4. Production of nasal mucus by olfactory glands serves to moisten the olfactory
epithelium and dissolve odorants.
5. The orbitofrontal area is an important region for odor identification and discrimination.
a. A only
b. B and D
c. D and E
d. C only

55. When viewing an object close to your eyes, which of the following are required for proper image
formation on the retina?
1. increased curvature of the lens,
2. contraction of the ciliary muscle,
3. divergence of the eyeballs,
4. refraction of light at the anterior and posterior surfaces of the cornea,
5. constriction of the pupil by contraction of the extrinsic eye muscles.
a. 1, 2, 3, 4, and 5
b. 1, 2, and 4
c. 1, 2, 3, and 4
d. 2, 4, and 5
e. 2, 3, and 4

56. Which of the following statements is incorrect?


1. Retinal is the light-absorbing portion of all visual photopigments.
2. The only photopigment in rods is rhodopsin, but three different cone
photopigments are present in the retina.
3. Retinal is a derivative of vitamin C.
4. Color vision results from different colors of light selectively activating different cone
photopigments.
5. Bleaching and regeneration of the photopigments account for much but not all of
the sensitivity changes during light and dark adaptation.
a. A and B
b. E and C
c. D only
d. C only

57. Which of the following is the correct sequence for the auditory pathway?
1. external auditory canal, tympanic membrane, auditory ossicles, oval window,
cochlea and spiral organ
2. tympanic membrane, external auditory canal, auditory ossicles, cochlea and spiral
organ, round window
3. auditory ossicles, tympanic membrane, cochlea and spiral organ, round window,
oval window, external auditory canal
4. auricle, tympanic membrane, round window, cochlea and spiral organ, oval window
5. external auditory canal, tympanic membrane, auditory ossicles, internal auditory
canal, spiral organ, oval window
a. A and E
b. B and C
c. D only
d. A only

58. Which one of the following is the correct sequence for the passage of a nerve
impulse?
1. Primary visual cortex
2. Optic tract
3. Optic nerve
4. Optic chiasma
5. Visual association area
a. 3, 4, 2, 1, 5
b. 1, 2, 3, 4, 5
c. 2, 3, 4, 1, 5
d. 5, 1, 2, 4, 3
e. 4, 3, 2, 5, 1

59. The four refracting media of the eye, listed in the sequence through which light
passes, are:
1. Vitreous body
2. Lens
3. Aqueous humor
4. Cornea
a. 1,2, 3, 4
b. 4, 1, 2, 3
c. 4, 3, 2, 1
d. 2, 3, 4, 1
e. 3, 2, 1, 4

60. A 5-year-old boy who recently emigrated from West Africa is brought to the physician by his
mother for treatment of recurrent follicular conjunctivitis. His father and uncle have had similar
symptoms and are now blind. Which of the following is the most likely causal organism?
a. Adenovirus
b. Chlamydia trachomatis
c. Herpes simplex virus
d. Neisseria gonorrhoeae
e. Streptococcus pneumonia.
ANSWER KEYS

1.A

2.A

3.D

4.C

5.A

6.C

7.A

8.D

9.C

10.B

11.C

12.A

13.D

14.B

15.C

16.D

17.B

18.C

19.A

20.C

21.C

22.B

23.D
24.E

25.A

26.E

27.C

28.B

29.C

30.C

31.A

32.B

33.D

34.E

35.A

36.D

37.C

38.D

39.D

40.D

41.A

42.E

43.E

44.A

45.A

46.B

47.B

48.A
49.B

PART C

50.B

51.A

52.A

53.D

54.A

55.B

56.D

57.D

58.A

59.C

60.B
EXCRETORY SYSTEM

1. A toxicant transported to the kidney via the blood will have the highest delivery to the renal
a. Medulla
b. Cortex
c. Papilla
d. None of the above

2. Which of the following is least likely to pass through the glomerulus?


a. Polycationic macromolecules
b. Inulin
c. Polyanionic macromolecule
d. Neutral macromolecule

3. Analgesic nephropathy is due to the long-term use of the combination of


a. NSAIDs and Acetaminophen
b. Acetaminophen and opiates
c. Opiates and NSAIDs
d. Tramadol and acetaminophen

4. Specific Gravity of urine is


a. 1005
b. 1010
c. 1025
d. 1017-34

5. Which cells of a nephron secrete rennin


a. juxta glomerular cell
b. Lacis
c. Mesangial
d. All

6. The amount of urine passed by a man in 24 hours is


a. 500 ml
b. 1000 ml
c. 1500 ml
d. 2000 ml

7. Water handling by the kidney (% reabsorption):


a. 93%
b. 94%
c. 99%
d. 99.4%
e. 99.9%

8. Responses to increased osmolarity


a. Thirst and ADH from stimulation of osmoreceptors in posterior hypothalamus
b. Thirst via stimulation of SFO and OVLT via Angiotensin II in hypovolaemia
c. Baroreceptors afferents to the Posterior Pituitary
d. Increased ADH levels
e. Aldosterone

9. Sweat in patients acclimatised to hot weather (as compared to patients in a temperate


climate)contains less Na+ because:
a. Takes longer for Na+ to be transported through sweat ducts
b. Aldosterone effect causing a reduction in Na+ in sweat
c. Increased intake of water causing a reduction in Na concentration

10. Renal blood flow is dependent on:


a. Juxtaglomerular apparatus
b. [Na+] at macula densa
c. Afferent vasodilatation
d. Arterial pressure
e. Efferent vasoconstriction

11. Renal blood flow:


a. Is 600-650ml/min per kidney
b. Is directly measured by infusing PAH (PARA-AMINO HIPPURATE)
c. Is increased by sympathetic tone.
d. Mesangial cells do not have any role in renal blood flow.

12. Renal blood flow:


a. Greater per unit mass than cerebral blood flow
b. Is greater in the medulla compared to the cortex
c. Is closely related to tubular sodium reabsorption
d. Only sympathetically mediated
e. Some noradrenergic endings on JG complex and tubules
f. Parasympathetic via hypogastric plexus

13. Which has the greatest renal clearance?


a. PAH -
b. Glucose
c. Urea
d. Water
e. Inulin

14. The ascending limb of the Loop of Henle is:


a. Impermeable to Na+
b. Involved in active transport of K+ into the lumen
c. Involved in active transport of Cl- out of lumen
d. Involved in active transport of Na+ into lumen
e. Hypotonic at the top

15. Regarding glucose handling in the kidney


a. Reuptake is passive
b. Tm is the same for all nephrons
c. D-glucose more rapidly absorbed than L-glucose
d. Reabsorption is inversely proportional to lipid solubility

16. Secretion/absorption of urea takes place in:


a. Proximal convoluted tubule
b. Distal convoluted tubule

17. Which ONE of the following is not involved in the regulation of glomerular filtration rate (GFR)?
a. Juxtaglomerular apparatus
b. Arterial pressure
c. Efferent arteriolar tone
d. Na content in distal tubule
e. Afferent arteriolar tone

18. With regard to glomerular filtration:


a. Autoregulation maintains flow
b. Afferent arteriole driving force
c. Is equal for cationic & anionic molecules
d. All cross if >/< 8 nm in diameter

19. Significant tubular reabsorption occurs with:


a. Phosphate
b. Creatinine
c. Urea
d. Sulphate
e. All of the above

20. Increased GFR caused by


a. Increased cardiac output
b. Afferent arteriolar vasoconstriction
c. Efferent arteriolar vasodilatation
d. Increased chloride delivery to the macula densa

21. The formula for GFR is:


a. GFR = Kf (HPG - HPB + OPG - OPB)
b. GFR = Kf (HPG - HPB - OPG + OPB)
c. GFR = Kf (HPG + HPB - OPG + OPB)
d. GFR = Kf (HPG + HPB - OPG - OPB)
e. GFR = Kf (HPG - HPB - OPG - OPB)

22. The effect of PTH on the kidney is to:


a. Increase Ca excretion and increase phosphate excretion
b. Increase Ca excretion and decrease phosphate excretion
c. Decrease Ca excretion and increase phosphate excretion
d. Decrease Ca excretion and decrease phosphate excretion
e. None of the above
23. Water handling by kidney (% reabsorption)
a. 93%
b. 94%
c. 99%
d. 99.4%
e. 99.9%

24. Resistance to renal blood flow is chiefly determined by:


a. Renal artery
b. Afferent & efferent arterioles
c. Interlobular & arcuate arteries
d. Peritubular capillaries

25. For renal clearance of a substance to exceed Inulin,


a. Increase in GFR
b. Must be secreted by either the proximal or distal tubules
c. Must have a lower molecular weight than Inulin

26. Water excretion by the kidney is due to:


a. A Osmosis
b. Active transport into the lumen
c. Passive secretion in the collecting tubules
d. Solvent drag
e. Facilitated diffusion
f. Paracellular movement

27. Angiotensin II causes:


a. Increases proximal tubular reabsorption of Na & H2O & increases secretion of K+
b. Increases distal tubular reabsorption of Na & H2O & decreases secretion of K+
c. Decreases distal tubular reabsorption of Na & H2O
d. Increases excretion of Na & H2O

28. Glomerulotubular balance


a. A Involves afferent arteriole feedback loop
b. B Involves efferent arteriole feedback loop
c. C Juxtaglomerular complex
d. D Ability to increase tubular absorption in response to an increase in filtered load
e. E None of the above

29. Kidneys produce:


a. Erythropoietin
b. ADH
c. Angiotensin II
d. ANP
e. Cholecalciferol

30. Renal nerve sympathetic stimulation


a. Causes increased sodium reabsorption from the PCT
b. Inhibits renin release
c. Increased GFR

31. Water reabsorption by the kidney:


a. 90% in proximal tubule
b. 60% in distal tubule
c. By active transport
d. none of the above

32. Glomerular filtration rate (GFR):


a. Is independent of the size of the capillary bed
b. Depends only on the hydrostatic and osmotic pressure differences across the capillary
c. Is determined by the same forces governing filtration across all other capillaries
d. Depends only on the permeability of the capillary
e. Requires active transport

33. Pressure diuresis:


a. Due to decreased reabsorption of Na+ & water in peritubular capillaries
b. Regulated by macula densa
c. Increase ADH
d. Increase angiotensin
e. Control by JGA

34. What is the minimum amount of urine required to excrete 600mOsm


a. 100ml
b. 500ml
c. 1 litre
d. 2 litre
e. 4 litre

35. Increase in GFR occurs with


a. Increased sympathetic stimulation
b. Decreased renal blood flow
c. Hypoproteinaemia
d. Ureteric obstruction
e. None of the above

36. Regarding renal clearance:


a. Inulin clearance measures renal blood flow
b. Creatine clearance correlates with GFR
c. Filtration fraction measured as inulin clearance/ PAH clearance

37. The clearance (or 'renal regulation') of which ONE of the following is NOT regulated by a hormone:
a. Sodium
b. Potassium
c. Calcium
d. Phosphate
e. Sulphate

38. Biggest contribution to urine concentration by:


a. Na+ absorption in thick ascending limb
b. Passive diffusion of urea in collecting ducts
c. Chloride absorption in distal convoluted tubule

PART-C

39. As the renal artery approaches the kidney, it branches to supply the renal glomeruli. Place the
following in the correct sequence starting from the renal artery.
1. arcuate artery
2. interlobular vein
3. interlobar artery
4. afferent arteriole
a. 2,1,3,4
b. 2,3, 1,4
c. 1,3,4,2
d. 4,3,1,2
e. 3,1,2,4

40. A substance is freely filtered and actively secreted. Which of the following represent the changes
in concentration of the substance along the nephron?
{A graph of clearance vs plasma concentration with various labelled curves labelled A, B, C, D}

a. Curve A-B
b. Curve A-C
c. Curve A-D
d. Curve A-E
e. None of the above

41. Which of the following statements are correct?


1. Glomerular filtration rate (GFR) is directly related to the pressures that determine net
filtration pressure.
2. Angiotensin II and atrial natriuretic peptide help regulate GFR.
3. Mechanisms that regulate GFR work by adjusting blood flow into and out of the
glomerulus and by altering the glomerular capillary surface area available for filtration.
4. GFR increases when blood flow into glomerular capillaries decreases.
5. Normally, GFR increases very little when systemic blood pressure rises.
a. 1, 2, and 3
b. 2, 3, and 4
c. 3, 4, and 5
d. 1, 2, 3, and 5
e. 2, 3, 4, and 5

42. Which of the following hormones affect Na_, Cl_, Ca2_, and water reabsorption and K_ secretion
by the renal tubules?
1. Angiotensin II
2. Aldosterone
3. ADH
4. atrial natriuretic peptide
5. thyroid hormone
6. parathyroid hormone.
a. 1, 3, and 5
b. 2, 3, and 6
c. 2, 4, and 5
d. 1, 2, 4, and 5
e. 1, 2, 3, 4, and 6

43. Which of the following are features of the renal corpuscle that enhance its filtering capacity?
1. large glomerular capillary surface area
2. thick, selectively permeable filtration membrane
3. High capsular hydrostatic pressure
4. high glomerular capillary pressure,
5. mesangial cells regulating the filtering surface area.
a. 1, 2, and 3
b. 2, 4, and 5
c. 1, 4, and 5
d. 2, 3, and 4
e. 2, 3, and 5

44. The micturition reflex


1. is initiated by stretch receptors in the ureters
2. relies on parasympathetic impulses from the micturition center in S2 and S3
3. results in contraction of the detrusor muscle
4. results in contraction of the internal urethral sphincter muscle
5. inhibits motor neurons in the external urethral sphincter.
a. 1, 2, 3, 4, and 5
b. 1, 3, and 4
c. 2, 3, 4, and 5
d. 2 and 5
e. 2, 3, and 5
45. Creatinine is a 113 dalton amino acid derivative that is generated from the breakdown of creatine
in muscle, distributed throughout total body water, and excreted by the kidneys primarily by
glomerular filtration. Although the serum level is affected primarily by the level of GFR, it is also
affected by other physiological processes, such as tubular secretion, generation, and extrarenal
excretion of creatinine. Due to variation in these processes amongst individuals and over time
within individuals, particularly the variation in creatinine generation, the cutoff for normal versus
abnormal serum creatinine concentration differs among groups. Some medications inhibit tubular
secretion of creatinine, thereby decreasing creatinine clearance and increasing serum creatinine
without a change in GFR. Which among the following is doing this function?
A. cephalosporin and aminoglycoside
B. trimethoprim
C. flucytocin, cisplatin
D. penicillin
a. A, B and C
b. B , C and D
c. A and D
d. A and B only

46. Cystatin C is a 13 kD, non-glycosylated, basic protein that is produced by all nucleated cells. It is
freely filtered by the glomerulus and then reabsorbed and catabolized by the tubular epithelial
cells, with only small amounts excreted in the urine. Its urinary clearance cannot be measured,
which makes it difficult to study factors affecting its clearance and generation. The generation of
cystatin C appears to be less variable and less affected by age and sex than serum creatinine;
however, some studies have reported increased cystatin level. Which among the following is the
cause of increased cystatin level?
A. Increased level of C-reactive protein or BMI
B. Steroid use
C. Hyperthyroidism
D. Hypothyroidism
a. A, B and C
b. A and B only
c. B and D
d. B and C only

47. Renal ultrasound examination. all are true except


a. its disadvantage is that it is highly operator dependent
b. quick, rapid, cheap and non-invasive and often the only required method of renal imaging
c. it can show the renal size, position, dilatation of the collecting system and other
abdominal pathologies like cystic liver.
d. in chronic renal failure, the density of the renal cortex is unfortunately decreased and
there is loss of cortico-medullary differentiation.
e. by utilizing the Doppler techniques, much information can be gained like the resistivity
index

48. All are true for Renal angiography and venography, except
a. the main indication of renal angiography is the diagnosis of renal artery stenosis and renal
hemorrhage
b. therapeutic intervention may be undertaken at the same time of doing renal angiography
like dilatation and stenting of renal artery stenosis and occluding and AV fistula
c. unlike IVU, there is a risk of cholesterol athero-embolisation
d. when compared to IVU, the risk is contrast nephropathy is lower
e. renal venography mainly used in the diagnosis of renal vein thrombosis and renal cell
carcinoma extension
ANSWER KEYS:

1.b

2.c

3.a

4.b

5.d

6.c

7.d

8.d

9.b

10.d

11.b

12.a

13.a

14.c

15.c

16.a

17.b

18.a

19.a

20.a

21.b

22.c
23.d

24.b

25.b

26.a

27.a

28.d

29.a

30.a

31.d

32.c

33.a

34.b

35.c

36.c

37.e

38.a

Part C

39.d

40.d

41.d

42.e

43.c
44.e

45.a

46.a

47.d

48.d
STRESS ADAPTATION & THERMOREGULATION

1. Heat regulating center are located in


a. cerebral
b. cerebellum
c. Hypothalamus
d. Medulla oblongata

2. For every % of rise of temperature B.M.R. increases by


a. 7 %
b. 21 %
c. 14 %
d. 28 %

3. By increase 1F temperature of body the pulse increases up to


a. 5
b. 10
c. 15
d. 20

4. Normal G.F.R. is
a. 100-110 ml/mt.
b. 120-125 ml/mt.
c. 140-160 ml/mt.
d. 160-180 ml/mt.

5. Exercise hyperthermia and fever can be distinguished by:


a. core temperature is always higher in fever than with exercise
b. the hypothalamic set-point is normal with exercise and is higher in fever
c. heat-dissipation mechanisms are lower or absent in fever
d. all of the above
e. only (b) and (c)

6. Aspirin and Tylenol decrease fever because:


a. they block cyclooxygenases
b. they block external pyrogens
c. they block internal pyrogens
d. they decrease prostaglandin E2 levels in the hypothalamus
e. only (a) and (d)

7. Which of the following is true about thermoregulation?


a. The sympathetic activates effectors for both heat generation and heat dissipation
b. The sympathetic activates heat generation mechanisms only
c. The parasympathetic activates heat dissipation mechanisms
d. In normal individuals the hypothalamic set point is constant around 37C always
e. All of the above

8. Heat exhaustion is caused by:


a. Failure of the hypothalamic thermostat to regulate temperature
b. Fall in blood pressure due to dehydration and decrease in systemic peripheral resistance
c. Excessive heat production during exercise
d. Protein denaturation and cell death due to high temperature
e. All of the above

9. Increased endurance due to aerobic exercise training is caused mainly by increased:


a. muscle phosphocreatine stores
b. muscle ATP concentrations
c. oxidative phosphorylation capacity
d. glycolytic capacity
e. glycogen stores

10. The phosphocreatine stores are able to sustain a maximum contraction of a muscle for:
a. 10 seconds
b. 30 seconds
c. 1 minute
d. 30 minutes
e. hours

11. Core body temperature is closest to


a. Oral temperature,
b. rectal,
c. Surface,
d. Axillary.

12. Which among the following is INCORRECT?


a. Humans can survive at a temperature that would cook a piece of steak, because the steak
cannot dissipate core heat energy.
b. Heat conductance of air at high pressure exceeds that at low pressure, whereby more heat
energy is lost from the divers body by conduction through air inside a diving bell deep under
water than at one atmosphere of pressure.
c. Temperature homeostasis is present when heat energy production equals heat loss.
d. Basal metabolic rate (BMR) is lower before than after a meal.
e. The neutral environmental temperature defines the level, where the resting metabolic rate is
minimal.

13. Acute onset (4 hours) diabetes insipidus in an otherwise healthy person produces these biochemical
changes?
a. Na 130, K 3.0, Osm 260
b. Na 130, K 4.0, Osm 300
c. Na 150, K 3.0, Osm 260
d. Na 150, K 3.5, Osm 320
e. Na 160, K 3.0, Osm 320

14. Gibbs-Donnan effect leads to:


a. Non-diffusible ions between 2 sides will be equal
b. Diffusible ions between 2 sides will be equal
c. Equal concentrations of ions on both sides
d. Equal passive diffusion
e. Osmotic gradient

15. Decreased osmolality and hypovolaemia. Would you see:


a. Decreased urine output
b. Increased urine output
c. Decreased ADH secretion
d. all

16. The ion with lowest intracellular concentration is:


a. Na+
b. HCO3
c. Ca2+
d. Mg2+
e. K+

17. Organic ion necessary in Na-K ATPase


a. Ca+2
b. Mg+2
c. PO4
d. SO4-2

18. The fight-or-flight reaction occurs at which stage of the general adaptation syndrome?
a. alarm
b. resistance
c. recovery
d. exhaustion

19. Which of the following describes a state in which blood pressure, heart rate and other vital
functions are maintained within a narrow range of normal?
a. general adaptation syndrome
b. resistance
c. homeostasis
d. stress response

20. During a stress response, which of the following physiological responses occur?
a. heart rate increases
b. blood pressure decreases
c. digestive system activity increases
d. bronchi contract

21. The action of the endocrine system is activated by which of the following systems?
a. central nervous system
b. parasympathetic nervous system
c. sympathetic nervous system
d. none of the above

22. The long-term wear and tear of the stress response is referred to as:
a. fight-or-flight reaction
b. allostatic load
c. general adaptation syndrome
d. psychoneuroimmunology

23. Long term overexposure to which of the following hormones, released during stress, has been linked
to health problems such as increased risk of heart disease and depression?
a. cortisol
b. epinephrine
c. norepinephrine
d. none of the above

24. Which of the following influence an individuals reaction to a stressful situation?


a. personality
b. gender
c. past experiences
d. all of the above

25. Psychoneuroimmunology is defined as the study of:


a. the ability to seek mental health assistance
b. the relationship between stress and disease
c. the general adaptation syndrome
d. the causes of stress

26. Which of the following health problems is associated with stress?


a. insomnia
b. digestive problems
c. increased risk of cardiovascular disease
d. all of the above

27. Stress triggered by a positive experience is known as:


a. eustress
b. distress
c. homeostasis
d. Stress cannot be triggered by a positive experience.

28. In what way/ways may stress contribute to coronary heart disease?


a. Via the release of fatty acids into the blood stream
b. Via hypertension
c. Via elevated smoking behaviour
d. All of the above

29. stress as a response can be measured using physical and physiological indices, such as:
a. Galvanic skin response
b. Height and weight
c. Body mass index
d. Anxiety and depression levels

30. Declines or alterations in immune function have frequently been associated with the experience of
stressful events. Which cell types have been linked to the subjective experience of stress?
a. B cells and O cells
b. NK cells, B cells, and T cells
c. T cells and B cells
d. NK cells, B cells, T cells, and O cells

31. Which physiological process/processes/systems are involved in heightening the arousal that
facilitates the 'fight or flight' response?
a. Sympathetic nervous system
b. Noradrenaline and adrenaline
c. Catecholamines
d. All of the above

32. Why it is that high level of stress during examinations is detrimental to performance?
a. It is not detrimental, stress is good in exam conditions
b. Because stress can impair memory and attention
c. Because too much arousal impairs performance
d. Both B and C

33. Which of the following adaptations will be apt to increase the work capacity at high altitude?
a. Increasing workload, decreasing duration of exercise
b. Increasing workload, increasing duration off exercise
c. Decreasing workload, increasing duration of exercise
d. Decreasing workload, decreasing duration of exercise

34. Hypothermia is
a. Is determined as a core body temperature below 32 degree Celcius.
b. Is characterized by shivering
c. May be caused by benzodiazepines
d. Is due to hypothyroidism in 30% of cases.
e. Causes a delta wave on the ECG.

35. The ECG changes of hypothermia include all, except


a. Delta Wave
b. Ventricular fibrillation
c. Prolonged PR interval
d. Prolonged QT interval
e. J wave

36. In acclimatization to altitude:


a. P50 is reduced, improving O2 uptake in the lungs
b. P50 is increased, improving O2 offloading in the tissues
c. 2,3 DPG levels are reduced, improving O2 offloading in the tissues
d. Alkalaemia reduces the affinity for O2, increasing p50
e. Increase in 2,3 DPG and a decrease in P50

37. Suffocation due to defective oxygen supply is called


a. Anoxia
b. Asphyxiation
c. Yawning sneezing
d. Noxia

38. As a compensatory process, uninjured renal cells following nephrotoxic exposure can undergo all of
the following except
a. Hypertrophy
b. Apoptosis
c. Adaptation
d. Proliferation

39. Cellular adaptation responses to toxicant-induced renal injury include the induction of all the
following except:
a. Metallothionein
b. Heat shock proteins
c. Glucose-regulated proteins
d. BRCA-1

40. Following toxic damage to the kidney, casts obstructing the tubular lumen are composed primarily
of
a. Neutrophils and macrophages
b. Red cells and white cells
c. Basophils and plasma cells
d. Necrotic and viable tubular cells

PART-C

41. Which among the following is CORRECT?


a. Water-induced hyponatraemia is called miners cramps.
b. Substantial influx of Ca2+ to the cells is probably involved in some cases of malignant
hyperthermia.
c. Hypothermia is a fall in core temperature below 32oC.
d. One degradation per second from radioactive material equals one bequerels (1 Bq), which is
also equal to one curie.
42. Acute radiation reduces leucocyte and thrombocyte production.
a. A, B, E
b. C and D
c. A, C, D, E
d. All the above

43. When a person dives deep under water and resurfaces rapidly, which one of the following sequencs
of events causes decompression sickness?
a. Increase in pressure -> N2 dissolves in blood -> rapid fall in pressure -> N2 bubbles in blood
b. Increase in pressure -> pressure on ear ossicles -> Loss of orientation -> imbalance
c. Fall in pressure -> Loss of O2 from blood -> deoxygenation -> fatigue
d. Increased pressure enhanced blood pressure -> headache -> Loss of memory

44. Recognized cause of hyperthermia includes


A. Salicylate poisoning
B. Spinal cord injury
C. Seizures
D. Barbiturates
E. Exercise
a. A , C and E
b. A nd B
c. A, B, C, D and E
d. B and D

45. The clinical features of hypothermia include all, except


A. Diuresis
B. Metabolic acidosis
C. Clinical improvement with dantrolene therapy
D. Acute pancreatitis
E. Polycythemia rubra vera
a. C and E
b. A and B
c. D only
d. E only

46. In thermoregulation
A. Respiratory heat loss is insignificant under normal conditions
B. Brown fat is an important source of heat production in neonates
C. Shivering is due to impulses conducted via autonomic efferents
D. Peripheral vasoconstriction increases heat production
E. Sweating is mediated by sympathetic cholinergic neurons
Which of the above statement are correct
a. A, B, C
b. A, B and E
c. B and D
d. C and E

47. Which of the following is false?


a. Convection only takes place in gases and fluids.
b. Heat radiation can take place across a vacuum.
c. A perfect radiating surface has an emissivity (e) = 1.
d. A perfect heat insulator has an emissivity (e) = 0.
e. Water is a better heat insulator than air.

48. Which of the following statements is false? Heat loss mechanism is done
a. By increasing conduction and radiation from the skin
b. By increasing sweating rate
c. Through increasing blood flow near the skin surface
d. Through counter current exchange.
e. By Reducing the metabolism

49. For an astronaut working outside a spaceship, the greatest loss of heat would occur by means of
Choose one answer.
a. conduction.
b. convection
c. conduction and convection.
d. radiation.
e. None of the above

50. If core body temperature rises above normal, which of the following would occur to cool the body?
(1) dilation of vessels in the skin, (2) increased radiation and conduction of heat to the environment,
(3) increased metabolic rate, (4) evaporation of perspiration, (5) increased secretion of thyroid
hormones.
a. 3, 4, and 5
b. 1, 2, and 4
c. 1, 2, and 5
d. 1, 2, 3, 4 and 5
e. 1, 2, 4, and 5
51. Which among the following hormone is involved in thermoregulation?
A. TRH and TSH
B. GHRH and TSH
C. CRH and ACTH
D. CRH and GHRH
a. A only
b. B only
c. And B
d. B and C
ANSWER KEYS:

1.c

2.c

3.c

4.b

5.e

6.e

7.a

8.b

9.c

10.a

11.b

12.d

13.e

14.e

15.a

16.c

17.b

18.a

19.c

20.a

21.c

22.b

23.a

24.d
25.b

26.d

27.a

28.d

29.a

30.b

31.d

32.d

33.c

34.c

35.a

36.e

37.b

38.c

39.d

40.d

PART C

41.b

42.a

43.a

44.a

45.a
46.b

47.e

48.d

49.d

50.b

51.a
DIGESTIVE SYSTEM

1. During infusion of an acidic solution (HCl infusion) , which contributes most to buffering?
a. Phosphate buffer
b. Bicarbonate buffer
c. Intracellular buffers
d. Proteins (intracellular proteins)
e. None of the above

2. In a patient with diabetic ketoacidosis, the following are true except:


a. There is decreased PaCO2
b. There is decreased concentration of H+ intracellularly
c. Renal excretion of titratable acids will be increased
d. There is increased synthesis of bicarbonate
3. Pancreatic fistula draining 1L/day, normal volume status maintained:
a. Hyperchloraemic metabolic acidosis
b. Hypochloraemic metabolic acidosis
c. Metabolic acidosis with normal chloride
d. Hyperchloraemic metabolic alkalosis
e. Hypochloraemic metabolic alkalosis

4. What is the main role of the large intestine?


a. The main role of the large intestine is to churn food around with enzymes.
b. The main role of the large intestine is to break down solid food.
c. The main role of the large intestine is to absorb water.
d. The main role of the large intestine is to mash and cut foods.

5. Products of the liver include


a. pepsin, gastrin and bile.
b. bile, proteases and urea.
c. bile, urea and blood proteins.
d. proteases, amylases and lipase.

6. If a persons liver fails, which process listed below would stop?


a. Digestion of proteins.
b. Destruction of red blood cells.
c. Storage of starch between meals.
d. Reabsorption of water from the digestive tract.

7. The emulsification of fats is a result of the release of secretions from the


a. pancreas.
b. gall bladder.
c. small intestine.
d. salivary glands.

8. The muscularis layer of the gastrointestinal tract contains only circular and longitudinal fibers,
except in the :
a. stomach
b. duodenum
c. jejunum
d. ileum
e. colon

9. The basic electrical rhythm of the small intestine is generated by the :


a. myenteric plexus
b. submucosal plexus
c. vagus nerve
d. parasympathetic
e. sympathetic

10. When Adipocytes increase in size, due to increased fat storage, they increase the production of:
a. leptin
b. resistin
c. free fatty acids
d. all of the above
e. none of the above

11. The cystic fibrosis transmembrane regulator allows Cl- diffusion across:
a. The basolateral membrane of gastric parietal cells.
b. The basolateral membrane of pancreatic duct cells.
c. The luminal membrane of pancreatic duct cells.
d. The luminal membrane of gastric parietal cells.
e. (c) and (d).

12. Which of the following reflex(es) is(are) likely to occur during the gastric phase of digestion:
a. ileogastric
b. gastrocolic
c. gastroileal
d. gastroesophageal
e. only (b) and (c)

13. Which of the following is not true about bilirubin:


a. is a waste product of hemoglobin metabolism
b. acts as an extracellular antioxidant
c. contributes to the color of urine and feces
d. functions to emulsify fats in the intestine
e. none of the above: all are true

14. Aminoacids, di and tripeptides enter the intestinal brush border via
a. facilitated diffusion
b. Na+-cotransport
c. H+-cotransport
d. Endocytosis
e. Both (b) and (c)

15. Which of the following is(are) true about insulin effects in hepatocytes:
a. stimulates exocytosis of vesicles containing glucose transporters (glut4)
b. increases glucose-6-phosphatase activity
c. increases glycogen phosphorylase activity
d. increases hexokinase activity
e. all of the above

16. Secretion of somatostatin in the stomach is highest:


a. during the gastric phase
b. during the intestinal phase
c. when the gastric pH < 2.0
d. all of the above
e. only (b) and (c)

17. Glucose stimulation of insulin secretion by pancreatic beta cells is mediated by:
a. ATP-gated K+ - channels that close when ATP increases
b. ATP-gated K+ - channels that open when ATP increases
c. ATP-gated Na+ - channels that open when ATP increases
d. ATP-gated Na+ - channels that close when ATP increases
e. none of the above

18. Intestinal crypts:


a. increase the surface area of the mucosa of the small intestine
b. produce new cells of the mucosa
c. function in the absorption of nutrients
d. contain the immune cells that protect the intestine (Peyers patches)
e. all of the above

19. The surface area of the mucosa of the small intestine is increased by:
a. plicae circularis
b. intestinal crypts
c. villi
d. microvilli
e. only (a), (c), and (d)

20. lacteals:
a. are in the villi
b. carry lymph
c. secrete milk
d. transport chylomicrons
e. only (a), (b), and (d)

21. Which cells secrete acid in the stomach?


a. G
b. D
c. ECL
d. Parietal
e. Chief
22. Which hormone is INCORRECTLY paired with its stimulus?
a. Secretin acid in the duodenum
b. CCK fatty foods
c. GIP glucose in the small intestine
d. Motilin acid in the stomach
e. Gastrin peptides in the stomach

23. Accumulation of fluid in the peritoneal cavity (ascites) is caused by:


a. increased pressure in the hepatic portal vein
b. increased pressure in the hepatic artery
c. decreased plasma protein (albumin) concentration
d. all of the above
e. only (a) and (c)

24. The following are true about bile salts, except:


a. they are found at high concentration in the hepatic portal vein
b. they give feces their characteristic color
c. they are made from cholesterol
d. they are absorbed in the ileum via a Na-cotransporter
e. they form mixed micelles with lipids

25. The first step in the conversion of pancreatic zymogens to active enzymes is:
a. hydrolysis of trypsinogen to trypsin by acid pH
b. hydrolysis of trypsinogen to trypsin by pepsin
c. hydrolysis of trypsinogen to trypsin by enteropeptidase
d. hydrolysis of trypsinogen to trypsin by trypsin
e. none of the above

26. Defecation is:


a. a short reflex triggered by distension of the rectum
b. a long reflex mediated by the vagus nerve
c. a long reflex mediated by sacral parasympathetic nerves
d. unavoidable if the external anal sphincter relaxes
e. both (c) and (d)

27. Which of the following types of GI motility only occurs during fasting periods?
a. Mass movements
b. Moving motor complexes
c. Segmentation
d. Peristalsis
e. Haustration

28. Which of the following is not true?


a. Glucagon inhibits insulin secretion
b. Insulin inhibits glucagon secretion
c. The sympathetic inhibits insulin secretion
d. The parasympathetic inhibits glucagon secretion
e. None of the above: all are true
29. The largest metabolic reserves for the average adult are stored as:
a. glycogen
b. proteins
c. aminoacids
d. triglycerides
e. fatty acids

30. Use the following information to answer 2 questions below:

I. Leptin. II. Grhrelin. III. Insulin. IV. CCK. V. Neuropeptide Y. VI. CART

Which of the peptides above inhibit(s) appetite:


a. I only
b. IV only
c. I and IV only
d. I, III and IV only
e. I, III, IV and VI only

31. Which of the peptides above stimulate(s) appetite:


a. II only
b. II and V only
c. II, V and VI only
d. II, III, V and VI only
e. I, II, III, V and VI only

32. All are GIT hormones except


a. Cholecystokinin
b. Gastrin
c. Secretin
d. Erythropoietin.

33. Iron is absorbed in


a. Stomach
b. Duodenum
c. Jejunum
d. Ileum.

34. In infants, defecation often follows a meal. The cause of colonic contractions in this situation is
a. gastro-ileal reflex
b. increased circulating levels of CCK
c. gastrocolic reflex
d. enterogastric reflex

35. Which of the following has highest pH


a. gastric juice
b. pancreatic juice
c. bile in GB
d. secretions of intestinal glands.

36. Man is unable to digest


a. dextrin
b. glucose
c. cellulose
d. glycogen.

37. Steatorrhoea may be caused by all factors except


a. Pancreatectomy
b. gastrin secreting hormone
c. resection of distal ileum
d. hemolytic jaundice

38. Normal swallowing is dependent on the integrity of the


a. 9th and 10th cranial nerves
b. pyramidal tract
c. trigeminal nerve
d. appetite center of hypothalamus.

39. Secretion of intrinsic factor occurs in


a. parietal cells of stomach
b. chief cells of stomach
c. upper abdomen
d. alpha cells of pancreas

40. In which of the following is absorption of water greatest


a. Colon
b. Jejunum
c. Duodenum
d. Stomach

41. Secretin is released by


a. acid in duodenum
b. acid in stomach
c. cells in the liver
d. distention of colon.

42. Which of the following would not be produced by total pancreatectomy


a. Hyperglycaemia
b. metabolic acidosis
c. weight gain
d. decreased absorption of amino acids

43. Vitamin D is essential for normal


a. fat absorption
b. Calcium absorption
c. ADH secretion
d. protein absorption

44. Gastrin secretion is increased by


a. acid in the lumen of stomach
b. distension of stomach
c. increased circulating levels of secretin
d. vagotomy

45. Saliva is responsible for all EXCEPT


a. helps in deglutition
b. prevents dental caries
c. is essential for complete digestion of starch
d. Prevents decalcification of the teeth.

46. Which of the following is NOT a characteristic of the myenteric ganglia of the enteric nervous
system?
a. Also known as Auerbach's plexus
b. Contains most of the motor neurons to circular and longitudinal muscles of the intestinal
tract
c. It is smaller than the submucousal ganglia and most prominent in the small and large
intestine
d. It is interconnected with the submucosal ganglia.
e. Contains both excitatory and inhibitory motor nerves to the smooth muscle fibers.

47. The enteric or intrinsic nervous system includes all of the following except:
a. The same number of neurons as the spinal cord.
b. Motor programs and reflex circuits.
c. Sensory neurons capable of detecting chemical, mechanical and thermal conditions in the
GI tract.
d. Vagus innervation to the esophagus, stomach, and pancreas.

48. Which of the following statements correctly describes a function of the parasympathetic
innervation of the GI tract?
a. Norepinephrine is the primary excitatory neurotransmitter.
b. Excessive parasympathetic activity can lead to a pathological state called paralytic ileas.
c. Inhibitory efferent Vagal innervation causes sphincters to contract.
d. Afferent sensory input is received in the nucleus of the solitary tract and processed in the
dorsal motor nucleus of the medulla oblongata.
e. Glucose concentration, pH, osmolality, movement of material past mechanoreceptors,
and level of contractile strength or stretch are monitored by efferent parasympathetic
fibers.

49. Muscles in the propulsive and receiving segments of the GI tract respond differently to food
movement through the gut. Which of the following statements correctly describes activity in the
propulsive segment?
a. The circular and longitudinal muscles are contracted.
b. The longitudinal muscles are contracted and the circular muscles are relaxed.
c. Both the longitudinal muscles and circular muscles are relaxed.
d. The circular muscles are contracted and the longitudinal muscles are relaxed.

50. Circular smooth muscle in the the GI tract:


a. Accounts for the majority of smooth muscle in the stomach and intestine.
b. Are coupled via gap junctions which increases the resistance and slows electrical signals
through the fibers.
c. Generates less propulsive force than longitudinal smooth muscle.

51. Which of the following is characteristic of the segmenting movements in the small intestine?
a. It decreases particle size, which increases the surface area for digestion.
b. It brings the products of digestion to the mucosal surface for absorption.
c. It results in the mixing of the luminal contents in front of the propulsive segments (i.e. in
the receiving segments)
d. All of the above.

52. Which of the following sphincters does NOT prevent reflux of material?
a. Lower esophageal sphincter
b. Gastroduodenal sphincter.
c. Ileocolonic sphincter
d. Internal anal sphincter.

53. Which of the following is NOT involved in swallowing?


a. Contracture of the upper esophageal sphincter.
b. Coordination by the swallowing center in the medulla oblongata.
c. The approximation of the vocal cords to close the glottis.
d. The raising of the larynx to close its entrance.
e. The elevation of the soft palate to close the nasopharynx.

54. Which of the following swallowing disorders results from the failure of organized peristaltic
behavior, or the simultaneous contraction all along the smooth muscle?
a. Diffuse spasm.
b. Dysphasia
c. Achalasia.
d. Myelphasia.
e. Trachiasia.

55. Choose the correct listing of the anatomical divisions of the stomach in the direction food
normally passes.
a. Corpus, fundus, antrum.
b. Fundus, antrum, corpus.
c. Antrum, corpus, fundus.
d. Fundus, corpus, antrum.
e. Corpus, antrum, fundus.

56. Which of the following is true of smooth muscle in the proximal stomach?
a. It has no action potentials.
b. It contracts phasically.
c. Its main purpose is to grind and mix food prior to propelling it to the gastroduodenal
junction.
d. Receptive relaxation is controlled by decreasing activity of excitatory vagal neurons.

57. Many factors influence the gastric action potential of the stomach. Which of the following is false?
a. Acetylcholine, gastrin, and cholecystokinin increase amplitude of the plateau phase.
b. Gastrin increases AP frequency.
c. The initial contraction coincides with the plateau phase while the trailing contraction
coincides with the rising phase of the AP.
d. Epinephrine and VIP decrease the amplitude of the plateau phase.
e. It is a single action potential originating in the orad boundary of the distal stomach.

58. Emptying gastric contents into the small intestine is strictly controlled. Which of the following is
most acurately represents this concept?
a. High caloric meals empty faster.
b. Liquid and solid meals empty at the same rate.
c. Higher acidity in the stomach results in a slower rate of emptying.
d. Tonicity of the gastric solution doesn't affect the rate of emptying.

59. How is the MMC controlled?


a. Ingestion of food ends MMC at all points in the intestine.
b. IV feeding ends MMC.
c. Gastrin and CCK terminate MMC in stomach and entire small intestine.
d. Vagal innervation completely controls MMC.
e. The MMC increases in speed as it reaches the ileum.

60. What is the difference between normal peristalsis of the SI and giant migrating contractions?
a. Normal peristalsis only propagates a short distance.
b. Because of the distance giant migrating contractions travel, they contribute significantly
to mixing chyme.
c. Normal peristalsis can be initiated by vinegar on the mucosa, parasites, enterotoxins, and
ionizing radiation.
d. Normal peristalsis helps strip lumen clean as it travels.
e. Normal peristalsis is sometimes associated with abdominal cramping and diarrhea.

61. Which best describes the motility in the large intestine?


a. Food clears the LI in about the same time as it takes to clear the small intestine.
b. Of the parts of the large intestine, chyme spends the least time in the ascending colon.
c. The ascending colon consists of haustra that aid in storage and dehydration of feces.
d. Remnants of meals stay in the LI for approximately 6-8 days.
e. Receptive relaxation allows the transverse colon to accept chyme without significantly
increasing the pressure.

62. Power propulsion is controlled by many factors in the large intestine. Which of the following is
NOT one of them?
a. Castor oil acts on mucosal receptors to initiate it.
b. May be triggered by ileal chyme delivery to the ascending colon.
c. Can be triggered by parasites, enterotoxins, and food antigens.
d. Normally starts in middle of transverse colon after the relaxation of circular muscles and
downstream disappearance of haustral contractions.
e. The influx of material will stimulate a mass movement 6-7 times daily.

63. What is Hirshsprung's disease?


a. A disease developed in adulthood.
b. Loss of the intrinsic nervous plexus resulting in continuous contraction of circular muscle.
c. Incompetence of internal or external anal sphincters.
d. Inappropriate leakage of feces or flatus.
e. A sensory malfunction where patients can't detect filling of the rectum.

64. Which of the following is NOT a component of saliva?


a. Alpha-amylase and ligual lipase.
b. Mucin and muramidase.
i. Bicarbonate and magnesium.
c. Protease and peptidase.
d. Lactoferrin and ABO blood group substances.

65. If you increase the rate of salivary secretion which of the following changes in ionic composition
results?
a. Sodium concentration increases.
b. Bicarbonate concentration decreases.
c. Chloride decreases.
d. Potassium increases.
e. Osmolarity decreases.

66. Which of the following salivary components continues to be active in the acidic environment of
the stomach?
a. Alpha-amylase
b. Lingual lipase.
c. Parotid-amylase
d. Beta-amylase

67. Which of the following best describes type of saliva produced when parasympathetic stimulation
increases the rate of salivary secretion?
a. Copius, protein-poor, electrolyte-rich.
b. Scant, transient protein-rich (mucin), electrolyte poor.
c. Scant, protein-poor, electrolyte-poor.
d. Copius, protein-rich (mucin), electrolyte-rich.

68. Which pair is correct concerning secretory glands/cells in the stomach and their products?
a. Cardiac glands -- hydrochloric acid and intrinsic factor.
b. Parietal cells -- pepsinogen
c. Chief cells -- mucus
d. G cells -- protease
e. F cells -- somatostatin.

69. Which of the following does NOT contribute to hydrochloric acid secretion in the stomach?
a. A basolateral bicarbonate/chloride exchanger to maintain intracellular chloride ions also
resulting in an alkaline tide in the bloodstream.
b. Passive flow of chloride and potassium ions through apical channels.
c. Hydrogen ions supplied by a basolateral hydrogen/sodium pump.
d. Water passively follows acid into gut to maintain osmotic balance.
e. The primary contributor to hydrochloric acid secretion is a hydrogen/potassium ATPase
that pumps out hydrogen in exchange for extracellular potassium.

70. Which of the following does NOT regulate gastric acid secretion?
a. Vagus nerve stimulation.
b. Cholecystokinin.
c. Histamine
d. Gastrin
e. Acetylcholine.

71. Which of the following combinations will result in the most acid secretion through potentiation?
a. Vagal input, histamine, gastrin
b. Vagal input, histamine, secretin
c. Histamine, gastrin, secretin.
d. Gastrin, vagal input, secretin
e. Gastrin, vagal input, cholecystokinin.

72. A condition that results in too much acid in the stomach could be attributed to which of the
following?
a. Lack of taste buds or damage to olfactory nerves.
b. Defect resulting in release of too much secretin.
c. Defective G cells.
d. Defect in D cells in the antrum.
e. Loss of elasticity of small intestine making distension more difficult.

73. If you follow the path that food normally takes through the digestive system, which of the
following is in the correct order?
a. Upper esophageal sphincter, epiglottis, lower esophageal sphincter, pyloric canal, duct of
Santorini, duct of Wirsung.
b. Pharynx, epiglottis, lower esophageal sphincter, pyloric canal, duct of Wirsung, duct of
Santorini.
c. Pharynx, epiglottis, lower esophageal sphincter, pyloric canal, duct of Santorini, duct of
Wirsung.
d. Pharynx, upper esophageal sphincter, pyloric canal, lower esophageal sphincter, duct of
Wirsung, duct of Santorini.

74. Pancreatic secretions have many of the same ions that plasma does, only the concentrations and
their rate of secretion are different. Which of the following is true?
a. A higher rate of pancreatic secretion results in increased bicarbonate and sodium ion.
Plasma has less bicarbonate and more sodium.
b. A higher rate of pancreatic secretion results in increased bicarbonate and decreased
chloride ion. Plasma has less bicarbonate and more chloride.
c. A higher rate of pancreatic secretion increases its osmolarity and pH. Plasma has a lower
osmolarity as pancreatic secretions.
d. A higher rate of pancreatic secretion results in increased bicarbonate and potassium ion.
Plasma has lower levels of both ions.

75. If duodenal mucosal release of CCK was not working properly, which phase of pancreatic secretion
would be interfered with?
a. Cephalic phase.
b. Gastric phase.
c. Intestinal phase.
d. Duodenal phase.

76. Which of the following has little effect on pancreatic secretion or it's components?
a. Acetylcholine.
b. Sympathetic stimulation.
c. Secretin
d. Cholecystokinin
e. VIP

77. Which of the following is NOT a component of bile secretions?


a. Stercobilin.
b. Bile pigments
c. Cholesterol
d. Lecithin
e. Bilirubin-glucuronide

78. Since bile acids are highly lipophilic, they would be rapidly absorbed through the small intestine
wall before they could facilitate lipid digestion. Which of the following prevents that from
happening?
a. Addition of hydroxyl and carboxyl acid groups to steroid nucleus of cholesterol.
b. Bile acids combine with phospholipids to become less easily absorbed.
c. Bile acids are conjugated to glycine or taurine so they ionize readily.
d. Cholesterol binds to the bile acids to keep them from being absorbed.

79. If a patient has inadequate bile secretion, which of the following could contribute to the
condition?
a. Excessive release of cholecystokinin
b. Excessive release of motilin
c. Excessive release of secretin
d. Excessive steroid hormones.
e. Excessive parasympathetic stimulation.

80. Components of the intestinal mucosa combine to increase the surface area to 600 times that of a
cylinder. Which component makes the greatest contribution to increasing the surface area?
a. The folds
b. The villi
c. The microvilli
d. The cilia
81. Sodium ions play an important role in carbohydrate absorption. All of the following processes
require it EXCEPT:
a. Sugars leaving the intestinal cell to enter the portal blood system.
b. Glucose absorption from gut lumen by second active transport.
c. Process that brings potassium into the instestinal cell.
d. Galactose absorption from the gut lumen.

82. Which of the following is NOT a polysaccharide?


a. Dextrin
b. Glycogen
c. Sorbitol
d. Amylose
e. Amylopectin.

83. Choose the FALSE statement regarding dietary fiber?


a. Examples are cellulose, hemicellulose, pectin, and gums.
b. Insoluble in water, poorly digested, excreted in feces.
c. Pectin is readily absorbed.
d. Binds to bile acids and promotes their excretion.
e. Reduces colon transit time and may reduce the production of carcinogenic secondary bile
acids.

84. A deficiency of colipase would result in which of the following?


a. Lipase would not be able to bind to the oil-water interface of the lipid.
b. An inability to digest phospholipids.
c. An inability to digest cholesterol.
d. An inability to digest lipids in the stomach.

85. Which of the following pass through the thoracic duct before passing through the liver?
a. Lecithin.
b. Micelles
c. VLDLs
d. Chylomicrons.
e. Small and medium chain fatty acids.

86. Theoretically, which of the following deficiencies would make pepsin's role of protein digestion in
the stomach more crucial?
a. Lipase deficiency.
b. A defect in the amino acid intestinal transport mechanism.
c. Enterokinase deficiency.
d. Cholecystokinin deficiency.

87. Of the following water soluble vitamins, which one CANNOT be absorbed by passive diffusion if
the concentration is high enough?
a. Vitamin B1
b. Niacin.
c. Vitamin B12
d. Biotin.

88. Defective parietal cells would result in malabsorption of which vitamin?


a. Vitamin B1
b. Vitamin B2
c. Niacin
d. Vitamin B12
e. Folic acid.

89. Which of the following is absorbed primarily by passive diffusion powered by a concentration
gradient?
a. Sodium
b. Potassium
c. Calcium
d. Zinc
e. Iron.

90. Of the approximately 9 liters of water that go through the GI tract daily, how much makes it out
in the feces?
a. 100 ml
b. 1000 ml
c. 2000 ml
d. 1500 ml
e. 500 ml.
91. Transport of fatty acids across the inner mitochondrial membrane is carried out by
a. Carnitine,
b. Epinephrine,
c. Pyruvate Kinase,
d. NADPH

92. PCO2 is less due to:


a. Metabolic Acidosis with Low Bicarbonate,
b. Hyperventilation with Low pH,
c. Hyperkalemia,
d. All of the Above.

93. Why there is hyperkalemia in above situation


a. Lack of Insulin Causes K+ to Move out of Cell,
b. Lack of Insulin Causes K+ to move inside the cell,
c. Low pH causes hyperkalemia,
d. Due to hyperventilation.

94. True about BMR


a. starvation decreases BMR by 50%
b. starvation increases BMR
c. independent of hormonal influence
d. independent of energy expenditure
95. Salivary secretion does all except:
a. Lubrication
b. Taste sensation
c. Digestion of carbohydrates
d. Digestion of proteins

96. Absorption of vitamin D in intestine is helped by:


a. Calcium
b. Pottassium
c. Magnesin
d. Iron

97. pH of the pancreatic secretion is


a. 6.8
b. 7.7
c. 7.5-8.0
d. 7.5 8.3

98. BMR rate in Male is


a. 24.2 K call/sq meter/hr
b. 36.1 K call/sq meter/hr
c. 32.1 K call/sq meter/hr
d. 38.2 K call/sq meter/hr

99. Daily dose of pottasium is


a. 1 gm
b. 1.5 gm
c. 4 gm
d. 2 gm
100. Iron absorption taken place in
a. Stomach
b. Duodenum
c. Jejunum
d. Ileum

101. Daily requirement of vitamine k is


a. 5000 I.U.
b. 300 I.U.
c. 400 I.U.
d. 3000 I.U.

102. BMR rate in Female is


a. 24.2 K call/sq meter/hr
b. 36.1 K call/sq meter/hr
c. 32.1 K call/sq meter/hr
d. 38.2 K call/sq meter/hr

103. The Golden yellow color of faeces is due to all expect


a. Stercobilinogen
b. Urobilin
c. Stercobilin
d. None
104. The Succus entricus is produced by
a. Brunners glands
b. Villi
c. Crypts of lieberkuhn
d. None
105. A BMR below ------- is almost diagnostic for Hypothyroidism
a. 50 %
b. 20 %
c. 30 %
d. 40 %

106. The poor mans meat in India


a. Wheat
b. Rice
c. Pulses
d. Milk

107. Higher levels of HbA1c are found in people is more prone to


a. Diabetes mellitus
b. Anemia
c. Bleeding disorders
d. Haemophilia

108. Serum amylase rise in


a. Pancreatitis
b. Endocardaitis
c. Liver Cirosis
d. Myocardial infaction

109. Vitamin k is formed in


a. kidney
b. Liver
c. Stomach
d. Large intestine

110. SGOT & SGPT get increased in


a. Viral hepatitis
b. Liver damage
c. Both
d. None

111. ORS solution dose not contant


a. Sodium cloride
b. Calcium cloride
c. Bicarbonate
d. Glucose

112. Anorexia, Nausia, Vomitting, Swelling, bleeding ete. are the symptoms of
hypervitaminosis of
a. Vit A
b. Vit D
c. Vit E
d. Vit B6

113. Ketone bodies are formed in


a. Liver
b. Spleen
c. Kidney
d. Blood

114. What is the daily requirement of vitamin A is adult


a. 5000 IU/Kg body weight
b. 3000 IU/Kg body weight
c. 400 IU/Kg body weight
d. 80 IU/Kg body weight

115. The Normal amount of faeces is


a. 100-200 gm/day
b. 200-300 gm/day
c. 200-250 gm/day
d. 300-400 gm/day
116. Nacl present in a composition of ORS solution according to Who in 2011 is
a. 10 gm
b. 3.5 gm
c. 5 gm
d. 2.6 gm

117. Intrinsic factor is present in


a. Liver
b. kidney
c. Gastric mucosa
d. Saliva

118. BMI (Body Mass Index) range 28.5 Kg/m is indicates


a. Underweight
b. Normal
c. Overweight
d. Obese

119. Best source for vitamin k is


a. Leafy vegetables
b. Pulses
c. Oil seeds
d. Fruits

120. Burning feet syndrome is caused by the deficiency of


a. Niacin
b. Vit. B12
c. Folic acid
d. Pantothenic acid
121. Vitamin D promotes the absorption of
a. Calcium
b. Phosphorous
c. Both a & b
d. None
122. Recommended daily dose of retinol for an adult is
a. 500 microgram
b. 600 microgram
c. 650 microgram
d. 750 microgram

123. Oesophagus at rest is:


a. Open at the top
b. Open at the bottom
c. Open at the top and the bottom
d. Closed at the top and the bottom
e. Contracted throughout its length

124. Na+ absorption in small bowel


a. Occurs by active transport
b. Occurs with H+
c. Decreases with glucose ( OR: Is facilitated by glucose)
d. Is by active transport at the brush border membrane
e. ? passive across basilateral membrane (?diffusion)
f. Occurs with Cl- through tight junctions

125. After a fatty meal, most of the fat would be:


a. Absorbed in the portal circulation & transported to the liver
b. Absorbed in the portal vein & transported in the hepatic artery
c. Absorbed into chylomicrons in the lymphatics
d. Absorbed as triglycerides into the portal vein & bypass the liver

126. Fat digestion:


a. Bile salts are the most efficient emulsifiers
b. Gastric lipase is the most important
c. Pancreatic lipase in the duodenum is the most important
d. Digestion takes place in micelles
e. Micelles attach to enterocyte receptor
127. Vitamin B12 deficiency:
a. Due to decreased ingestion
b. Due to decreased absorption by ileum
c. Causes a deficiency in haemoglobin
d. Causes a decrease in decrease in red cell production
e. all of the above

128. Iron absorption:


a. Passive
b. Binds to apoferritin in small intestine lumen
c. Decreased with increased pH
d. Requires acidic gastric pH

129. Findings in iron deficiency:


a. Increased apoferritin synthesis
b. Decreased transferrin saturation
c. Transferrin synthesis is reduced
d. Increased amounts of ferritin
e. Haemosiderin is produced

130. The major route of iron excretion is:


a. Excretion of transferrin in the gut
b. Shedding of intestinal mucosal cells
c. Increased renal excretion

131. Gastric acid secretion is decreased by:


a. Vagal inhibition
b. Luminal peptides & amino acids (OR: Ingestion of protein)
c. Noradrenaline
d. M1 cholinergic antagonist same efficacy at reducing gastric acid secretion
e. Distension of bowel wall

132. Release of which ONE of the following increases the pH of duodenal contents?
a. Secretin
b. Gastrin
c. Intrinsic factor
d. Cholecystokinin
e. Gastrin releasing peptide

133. Speed of delivery of nutrients from stomach to small intestine:


a. CHO>fat>protein
b. CHO>protein>fat
c. D.Fat>protein>CHO

134. Gastric emptying is slowest after consuming:


a. High protein meal
b. High fat meal
c. Alcohol
d. Metoclopamide/calcium
e. Carbohydrates

135. In the small intestine, glucose is absorbed


a. Passively
b. In combination with Sodium
c. By facillitated difussion
d. By cotransport with Chloride
e. Actively by insulin dependent uptake

136. After ingestion of a meal:


a. Digestion of fat and carbohydrate begins in the mouth while protein digestion begins in
the stomach
b. Carbohydrate in the mouth and protein in the stomach
c. Protein in mouth and fats and carbohydrate in stomach
d. Most fluid and electrolytes are absorbed in the large bowel
e. Composition of the food has no effect on transit time through the bowel

137. Calcium uptake in the intestine:


a. Is passive
b. Requires a carrier protein on the mucosal side
c. Is by facilitated diffusion
d. Is less than 10% than dietary intake
e. Is facilitated by phosphate

138. Bacteria in the intestines:


a. Reduced by the continuous movement of contents through GIT
b. Small intestine is sterile
c. Bacteria in small intestine and large intestine same in number but different species
d. Required for the absorption?/ breakdown of?
e. Reduced in small intestine due to gastric acid & fast motility

PART-C

139. Which of the following statements regarding the regulation of


gastric secretion and motility are true?
(1) The sight, smell, taste, or thought of food can initiate the cephalic phase of gastric activity.
(2) The gastric phase begins when food enters the small intestine.
(3) Once activated, stretch receptors and chemoreceptors in the stomach trigger the flow of
gastric juice and peristalsis.
(4) The intestinal phase reflexes inhibit gastric activity.
(5) The enterogastric reflex stimulates gastric emptying.
a. 1, 3, and 4
b. 2, 4, and 5
c. 1, 3, 4, and 5
d. 1, 2, and 5
e. 1, 2, 3, and 4

140. Which of the following is NOT a characteristic of the myenteric ganglia of the enteric
nervous system?
a. Also known as Auerbach's plexus
b. Contains most of the motor neurons to circular and longitudinal muscles of the
intestinal tract
c. It is smaller than the submucousal ganglia and most prominent in the small and large
intestine
d. It is interconnected with the submucosal ganglia.
e. Contains both excitatory and inhibitory motor nerves to the smooth muscle fibers.
141. Many factors influence the gastric action potential of the stomach. Which of the
following is false?

a. Acetylcholine, gastrin, and cholecystokinin increase amplitude of the plateau phase.


b. Gastrin increases AP frequency.
c. The initial contraction coincides with the plateau phase while the trailing contraction
coincides with the rising phase of the AP.
d. Epinephrine and VIP decrease the amplitude of the plateau phase.
e. It is a single action potential originating in the orad boundary of the distal stomach.

142. If you increase the rate of salivary secretion which of the following changes in ionic
composition results?
a. Sodium concentration increases.
b. Bicarbonate concentration decreases.
c. Chloride decreases.
d. Potassium increases.
e. Osmolarity decreases.

143. Which of the following does NOT regulate gastric acid secretion?
A. Vagus nerve stimulation.
B. Cholecystokinin.
C. Histamine
D. Gastrin
E. Acetylcholine
a. Only A
b. Only B
c. C and D only
d. E only

144. Pancreatic secretions have many of the same ions that plasma does, only the
concentrations and their rate of secretion are different. Which of the following is true?
a. A higher rate of pancreatic secretion results in increased bicarbonate and sodium ion.
Plasma has less bicarbonate and more sodium.
b. A higher rate of pancreatic secretion results in increased bicarbonate and decreased
chloride ion. Plasma has less bicarbonate and more chloride.
c. A higher rate of pancreatic secretion increases its osmolarity and pH. Plasma has a
lower osmolarity as pancreatic secretions.
d. A higher rate of pancreatic secretion results in increased bicarbonate and potassium
ion. Plasma has lower levels of both ions.

145. If duodenal mucosal release of CCK was not working properly, which phase of pancreatic
secretion would be interfered with?
a. Cephalic phase.
b. Gastric phase.
c. Intestinal phase.
d. Duodenal phase.

146. Since bile acids are highly lipophilic, they would be rapidly absorbed through the small
intestine wall before they could facilitate lipid digestion. Which of the following prevents that
from happening?
a. Addition of hydroxyl and carboxyl acid groups to steroid nucleus of cholesterol.
b. Bile acids combine with phospholipids to become less easily absorbed.
c. Bile acids are conjugated to glycine or taurine so they ionize readily.
d. Cholesterol binds to the bile acids to keep them from being absorbed.

147. If a patient has inadequate bile secretion, which of the following could contribute to the
condition?
a. Excessive release of cholecystokinin
b. Excessive release of motilin
c. Excessive release of secretin
d. Excessive steroid hormones.
e. Excessive parasympathetic stimulation.

148. Theoretically, which of the following deficiencies would make pepsin's role of protein
digestion in the stomach more crucial?
a. Lipase deficiency.
b. A defect in the amino acid intestinal transport mechanism.
c. Enterokinase deficiency.
d. Cholecystokinin deficiency.

149. Sodium ions play an important role in carbohydrate absorption. All of the following
processes require it EXCEPT:
a. Sugars leaving the intestinal cell to enter the portal blood system.
b. Glucose absorption from gut lumen by second active transport.
c. Process that brings potassium into the instestinal cell.
d. Galactose absorption from the gut lumen.
ANSWER KEYS:

1.b

2.b

3.b

4.c

5.c

6.b

7.b

8.a

9.a

10.d

11.c

12.e

13.d

14.e

15.d

16.c

17.a

18.b

19.e

20.e

21.d

22.d
23.e

24.b

25.c

26.c

27.b

28.a

29.d

30.e

31.b

32.d

33.b

34.c

35.b

36.c

37.d

38.a

39.b

40.b

41.a

42.c

43.b

44.b

45.c

46.c

47.d
48.d

49.d

50.a

51.d

52.d

53.a

54.a

55.d

56.a

57.c

58.c

59.a

60.a

61.b

62.e

63.b

64.c

65.a

66.b

67.a

68.e

69.c

70.b

71.a

72.d
73.c

74.b

75.c

76.b

77.a

78.c

79.d

80.c

81.a

82.c

83.c

84.a

85.d

86.c

87.c

88.d

89.b

90.a

91.a

92.d

93.d

94.b

95.d

96.a

97.d
98.d

99.a

100.b

101.d

102.c

103.a

104.c

105.c

106.c

107.a

108.a

109.d

110.c

111.c

112.a

113.a

114.d

115.a

116.d

117.c

118.c

119.a

120.d

121.c

122.d
123.d

124.a

125.c

126.c

127.e

128.c

129.b

130.b

131.a

132.a

133.b

134.b

135.b

136.a

137.b

138.e

PART C

139.a

140.c

141.c

142.a
143.b

144.b

145.c

146.c

147.d

148.c

149.a
ENDOCRINOLOGY AND REPRODUCTION

Use the following information to answer questions 1 and 2 below:

Cortisol. II. Glucagon. III. Insulin. IV. Growth hormone. V. Epinephrine.

1. Which of the hormones above stimulate(s) lipolysis?


a. II only
b. I and II only
c. II and IV only
d. I, II and IV only
e. I, II, III, and IV only

2. Which of the hormones above stimulate(s) protein synthesis?


a. I and III only
b. I only
c. III and IV only
d. I, III and IV only
e. I, III, IV, and V only

3. Thyroid hormones increase all of the following, except:


a. shivering thermogenesis
b. non-shivering thermogenesis
c. uncoupling of oxidative phosphorylation
d. futile transport cycles (ion pumps and leaks)
e. oxidative metabolism

4. The primary follicles in ovaries of a menstruating woman contain:


a. oogonia
b. primary oocytes
c. secondary oocytes
d. corpora lutea
e. all of the above

5. Crossing-over and random segregation of maternal and paternal chromosomes in gametogenesis


occur:
a. during mitosis of oogonia or spermatogonia
b. before the first meiotic division
c. during the first meiotic division
d. between the first and the second meiotic division
e. after the second meiotic division

6. The seminiferous tubules of a boy before the onset of puberty contain:


a. spermatogonia
b. primary spermatocytes
c. secondary spermatocytes
d. spermatids
e. all of the above.
7. In an embryo with karyotype XX:
a. Wolffian ducts will regress and Mullerian ducts will develop
b. Mullerian ducts will regress and Wolffian ducts will develop
c. Estrogens are needed for proper development of female genitalia
d. Increased androgens can cause masculinization of external genitalia
e. Both (a) and (d)

8. In an embryo with karyotype XY and defective androgen receptors:


a. Testes will develop and secrete testosterone
b. External genitalia will be female
c. Mullerian ducts will regress
d. There will be no vas deferens, epididymis, prostate, nor seminal vesicles
e. All of the above

9. Which of the following cells require the enzyme aromatase for proper function:
a. Granulosa
b. Theca
c. Sertoli
d. Leydig
e. Both (a) and (c)

10. Which of the following cells has(ve) FSH receptors and secrete(s) inhibin?
a. Granulosa
b. Theca
c. Sertoli
d. Leydig
e. Both (a) and (c)

11. Elderly men compared with young adult men have:


a. Decreased testosterone levels
b. Decreased gonadotropin levels
c. Increased gonadotropin levels
d. Increased testosterone levels and (c) only.

12. The highest pituitary gonadotropin levels in menstruating women occur:


a. Just prior to ovulation
b. During the menstrual phase
c. During the mid-follicular phase
d. During the mid-luteal phase
e. A few days before menstruation

13. Which of the phases of intercourse will be most affected by sympathetic impairment:
a. Arousal
b. Plateau
c. Orgasmic
d. Resolution
e. All of the above
12. Which of the following does not contain erectile tissue?
a. clitoris
b. glans penis
c. nipples
d. penile shaft
e. prepuce

13. A deficiency of ACTH secretion would greatly diminish


a. The synthesis of aldosterone
b. The synthesis of testosterone in a man
c. The secretion of cortisol
d. The secretion of estradiol
e. All of the above

14. Cortisol synthesis would be diminished


a. A defect in 11-hydoxysteroid dehydrogenase
b. A defect in 21-hydoxylase
c. A defect in 18-hydoxylation
d. A defect in 5-alpha-reductase
e. A defect in aromatase

15. A meal rich in proteins but low in carbohydrates does not cause hypoglycaemia because
a. glucagon secretion is stimulated by meals
b. the meal causes compensatory increase in T4 secretion
c. cortisol in circulation prevents glucose from entering the muscles
d. the amino acids in the meal are converted to glucose

16. Which of the following is incorrectly paired


a. beta cells-insulin
b. F cells- gastrin
c. delta cells- somatostatin
d. alpha cells- glucagon

17. After intravenous administration of a large dose of insulin, the return of a low blood sugar level
to normal is delayed by
a. thyrotoxicosis
b. glucagon deficiency
c. diabetes
d. parathormone deficiency

18. Insulin increases entry of glucose into


a. renal tubule
b. the mucosa of the small intestine
c. neurons of motor cortex
d. skeletal muscle cells

19. Glucagon is not normally found in the


a. Brain
b. Pancreas
c. Git
d. adrenal glands

20. Which of the following is NOT produced by physiological amounts of glucocorticoids


a. maintenance of normal vascular reactivity
b. inhibition of inflammatory response
c. increased excretion of a water load
d. inhibition of ACTH secretion

21. Cortisol increases blood glucose level by


a. increasing lipolysis
b. increasing protein synthesis in muscles
c. increasing gluconeogenesis
d. increasing growth hormone secretion

22. Epinephrine and norepinephrine


a. are amino acids
b. are both secreted by neurons in the autonomic nervous system
c. are polypeptides
d. both activate alpha and beta adrenergic receptors

23. A decrease in extracellular volume is expected to cause increased secretion of all except
a. Vasopressin
b. Rennin
c. Thyroxin
d. ACTH

24. A patient with parathyroid deficiency 10 days after thyroidectomy will show
a. a low plasma phosphate and Ca++ levels and tetanus
b. a low plasma Ca++ levels, increased muscular excitability and Trousseaus sign
c. high plasma phosphate and Ca++ and bone demineralization
d. increased muscular excitability, high plasma Ca++ and bone demineralization

25. Which of the following is not involved in regulation of plasma Ca++ levels
a. Kidneys
b. Skin
c. lungs
d. intestine

26. Ca++ plays an important role in following biological processes except


a. oxygen utilization
b. contraction of cardiac muscle
c. contraction of skeletal muscle
d. blood coagulation

27. Epiphyseal closure is regulated by


a. calcitonin
b. somatomedins
c. 1,25 dihydroxy cholecalciferol
d. Thyroxine

28. Which of the following pituitary hormones is a polypeptide


a. MSH
b. ACTh
c. beta - endorphin
d. growth hormone

29. Growth hormone acts directly on


a. stimulation of protein synthesis
b. stimulation of cartilage formation
c. elevation of BSL
d. stimulation of bone formation

30. Hypopituitarism is characterized by


a. Infertility
b. intolerance to heat
c. weight gain
d. excessive growth of the soft tissue

31. Excessive growth hormone secretion in adults causes


a. Acromegaly
b. Gigantism
c. increased entry of glucose in muscles
d. hypothyroidism

32. Angiotensin increases blood pressure by acting on the following EXCEPT


a. aldosteron secretion
b. vascular smooth muscle
c. parasympathetic nervous system
d. sympathetic nervous system

33. Erythropoietin
a. contains iron
b. has no effect on WBC count
c. stimulates renin secretion
d. increases half life of RBC

34. Somatostatin
a. inhibits insulin and glucagoon release
b. stimulates insulin and glucagon release
c. stimulator of glucagon release
d. acts as obesity hormone

35. Thyroid hormone stored in the lumen of follicles is in the form of


a. free T3
b. free T4
c. attached to thyroglobulin in the gland
d. attached to thyroid binding globulin

36. Secretion of growth hormone


a. increases during REM sleep
b. increases during exercise
c. increases during starvation
d. increases during NREM sleep

37. Atrial natriuretic peptide brings


a. afferent arteriolar constriction in kidney
b. efferent arteriolar consrtiction in kidney
c. increases renin secretion
d. constriction of mesangial cells

38. Thyroid binding globulins are normal in


a. Hyperthyroidism
b. Pregnancy
c. parents treated with glucocorticoids
d. parents treated with estrogens

39. In starvation which of the following is reduced


a. plasma T4
b. plasma T3
c. reverse tri-iodothyroxine
d. thyroxine

40. Hypothyroidism is associated with increased levels of


a. Cholesterol
b. Albumin
c. TBG
d. Iodine

41. The metabolic rate is least affected by an increase in the plasma levels of
a. TSH
b. TRH
c. TBG
d. none of the above

42. The coupling of mono iodotyrosine and di-iodotyrosine and the iodination of thyroglobin is
blocked by
a. TSH
b. TRH
c. Iodine
d. thiocarbamides such as propylthiouracil
43. Parathyroid hormone
a. decreases Ca++ mobilization of bone
b. increases Ca++ mobilization from bone
c. decreases circulating levels of free Ca++
d. increases urinary excretion of Ca++

44. Thyrocalcitonin
a. is secreted by thyroid.
b. is secreted by hypothalamus
c. is secreted by parathyroid
d. increases Ca++ absorption by stomach.

45. Testosterone is secreted by


a. sertoli cells of testis
b. cells of adrenal medulla
c. cells of hypothalamus
d. leydig cells of testis

46. Temporary methods of birth control which are best suited to prevent transmission of disease are
a. IUD
b. Spermatocides
c. Condom
d. Ru 486

47. Cryptorchidism means


a. descent of testis
b. hypogonadism
c. hyperfunction of the testis
d. undescended testis

48. Androgen binding protein is produced by


a. Adrenals
b. Hypothalamus
c. sertoli cells
d. leydig cells

49. All of the following are produced by the corpus leuteum except
a. Estrogens
b. Progesterone
c. relaxin
d. F.S.H.

50. The testis is kept at a temperature of 2-3 degrees C below core temperature due to
a. contraction of cremasteric muscle
b. contraction of dartos muscle
c. contraction of internal oblique muscle
d. relaxation of cremasteric muscle and due to position of the testis outside the pelvic cavity
51. Early detection of pregnancy depends on detection of
a. FSH
b. Progesterone
c. LH
d. hCG

52. Secondary amenorrhoea can most commonly be caused in the following conditions EXCEPT
a. age above 60 years
b. stress
c. pregnancy
d. competitive athletes

53. In the first 20 weeks of pregnancy, placental function is best assessed by urinary
a. Pregnanediol
b. Pregnanetriol
c. chorionic gonadotropin
d. estriol

54. Best method for diagnosing fetal lung maturity is


a. clinical examination
b. ultrasonography
c. amniocentesis
d. fetal kick counts

55. Full lung maturity is indicated by L/S ratio


a. 2:1
b. 3:1
c. 4:1
d. 5:1

56. The best method to diagnose Rh sensitization in the mother is


a. direct coombs test
b. indirect coombs test
c. 'c' antigen
d. 'a' antigen

57. Inhibin is secreted by


a. graffian follicle
b. corpus leuteum
c. endometrium
d. placenta

58. Maximal rise in the levels of prolactin are seen in


a. at term
b. after delivery
c. during lactation only
d. none of the above
59. Estrogens are given in high doses to the mother to suppress lactation in
a. cleft palate in child
b. highly obese mother
c. to prevent transmission of communicable disease like HIV
d. Inverted nipples.

60. thyroxine is carried by


a. globulin
b. pre albumin
c. transferrin
d. ceruloplasmin
e. albumin

61. Posterior pituitary hormones are:


a. ADH and Oxytocin
b. TRH and FSH
c. Vasopressin and CRH
d. Oxyticin and ACTH

62. Cerebellar Ataxia is characterized by all except:


a. Resting tremor
b. Dysdiadochokinesis
c. Ataxia
d. Hypotonia

63. Anabolic hormones are all except:


a. Cortisol
b. Testosterone
c. Growth Hormone
d. Insulin

64. Avoiding response is due to:


a. Ispilateral frontal lobe lesions
b. Contra lateral frontal lobe lesions
c. Ipsilateral parietal lobe lesions
d. Contralateral parietal lobe lesions

65. Frequency of Alpha waves is:


a. 4-6
b. 13
c. 8-13
d. 2-4

66. Absorption of H2O in PCT is characteristic by all except:


a. Vasopressin dependent
b. 60% of water is absorbed
c. Depends on solutes
d. Vasopressin independent

67. Which one of the hormone is not secreated by adenohypophysis part of pitutary gland?
a. MSH
b. STH
c. ADH
d. TSH

68. Which one of the hormone is not secreated by pitutary gland


a. MSH
b. STH
c. TH
d. TSH

69. Which of the following hormone hyposecretion is caused Conns disease


a. GH
b. Aldosterone
c. Corticosterone
d. TH

70. In menstrual cycle Follicular phase occures between


a. 1 - 4 days
b. 4- 14 days
c. 11 - 18 days
d. 14- 28 days

71. In the menstrual cycle Luteal phase occures between


a. 1 4 days
b. 4 14 days
c. 11 18 days
d. 14 28 days

72. Which one of the hormone is not secreted by thyroid gland -


a. MSH
b. STH
c. TH
d. TSH

73. Normal Sperm count in male is


a. 50 million
b. 100 million
c. 50-100 million
d. 60-150 million

74. From which month onwards intrauterine life, the erythropoiesis starts in red bone marrow
a. 4th
b. 5th
c. 3th
d. 7th

75. The pregnant women are especially susceptible to deficiency of which Vitamin
a. B6
b. B12
c. Folic acid
d. B12 & Folic acid

76. Which cells are the basis of blood testis barrier.


a. Interstitial cells
b. Leyding cells
c. Sertoli cells
d. All

77. Which cells of the testis are a source of estrogen in the adult healthy male
a. Interstitial cells
b. Leyding cells
c. Sertoli cells
d. All

78. The semen is in reaction


a. Acidic
b. Alkaline
c. Neutral
d. None

79. The hormone secreted by theca internal of a graffian follicle


a. Estrogen
b. Progestrone
c. LH
d. All

80. Normal testicular development requires:


a. Xy
b. XX
c. Y chromosome
d. X chromosome

81. Which endocrine gland is attributed with fight or flight functions


a. Pituitary gland
b. Adrenal
c. Thyroid
d. Pancreas

82. Spermatogonia matures into spermatozoa in


a. 40 days,
b. 72 days,
c. 96 days,
d. 120 days.

83. Insulin like growth factor is synthesized in


a. Placenta,
b. Liver,
c. islet of Langerhans,
d. Pitutary

84. Milk producing hormone is the


a. Relaxin
b. Progestrone
c. Prolactin
d. Estrogen

85. Pregnany hormone is


a. estrogen
b. Oxytocin
c. Progesterone
d. Chorionic gonadotropic hormone

86. After Ovulation, the ovum remains alive for about


a. 12 24 hours
b. 24 36 hours
c. 24 48 hours
d. 48 72 hours

87. Viable period of Spermatozoa within the female genital tract is


a. 24 hours
b. 36 hours
c. 48 hours
d. 72 hours

88. Life expectancy of Spermatozoa after ejaculation in the female reproductive tract is
a. 24 hours
b. 36 hours
c. 48 hours
d. 72 hours

89. Atrial natriuretic factor acts by:


a. Membrane bound Guanyl cyclase,
b. Cytosolic Guanyl cyclase,
c. Soluble CGMP,
d. Lipooxygenase.

90. There is increase in secretion through anterior pituitary by


a. Hyperosmolarity of plasma,
b. Somatostatin,
c. Somatomedin,
d. Suckling of breast

91. Effects of a 24 hour fast:


a. Glycogenolysis (?gluconeogenesis)
b. Protein catabolism
c. Acidosis
d. Ketone production from protein
e. All of the above

92. Which hormone causes increased BSL, increased protein anabolism & increased plasma FFA?
a. Cortisol
b. Parathyroid hormone
c. Growth hormone
d. Insulin

93. Which hormone causes increased BSL, increased protein catabolism & increased plasma FFA?
a. Cortisol
b. Parathyroid hormone
c. Growth hormone
d. Insulin

94. The hypothalamus inhibits the release of:


a. TSH
b. ACTH
c. FSH
d. GH
e. Oxytocin

95. Secretion of renin is stimulated by:


a. Increased left atrial pressure
b. Increased angiotensin II
c. Decreased right atrial pressure.
d. Increased angiotensin I

96. Secretion of renin is stimulated by:


a. Increased left atrial pressure
b. Increased angiotensin II
c. Decreased right atrial pressure
d. erythropoietin

97. Regarding hyperglycaemia: Which of the following is untrue? It causes:


a. Increased H+
b. Increased Na+ (K+)
c. Increased urine output
d. Increased ECF (or blood volume)
e. Increased glucagon
98. Mechanism of action of ADH:
a. Insertion of water channels (pores) into basolateral membrane
b. Increase in GFR
c. Insertion of water channels into luminal (apical) membrane
d. Increased Na+ uptake in DCT
e. Removal of water pores from apical membrane

99. How many hours after a meal is Basal Metabolic Rate (BMR) measured?
a. 1 hour
b. 2 hours
c. 6 hours
d. 12 hours
e. 18 hours

100. Which ONE of the following is a water soluble vitamin?


a. Vitamin A
b. Vitamin B
c. Vitamin D
d. Vitamin E
e. Vitamin K

101. Insulin ( OR: Insulin receptor):


a. Receptor site intracellular
b. Inactivates tyrosine kinase
c. Activates membrane glucose transport
d. Acts via activation of transport protein to increase glucose transport into cells

102. How does insulin act?


a. Voltage gated ion channels
b. Tyrosine kinase membrane receptor
c. Nuclear receptor
d. G protein

103. Heat production at rest is mostly due to:


a. Skeletal muscle activity
b. Na,K ATPase pump
c. Dynamic action of food.
d. digestion

104. Decreased heat production under general anaesthesia is due to:


a. Decreased skeletal muscle tone
b. Decreased anterior pituitary function
c. Vasodilatation
d. Starvation
e. Decreased Na+/K+ ATPase activity
105. Angiotensinogen secretion is increased by:
a. ACTH
b. Beta-endorphin
c. Growth hormone
d. Antidiuretic hormone
e. Prolactin

106. The energy value of 1g of carbohydrate is:


a. 3 kcal
b. 4 kcal
c. 5 kcal
d. 7 kcal
e. 9 kcal

107. Oxytocin causes:


a. Decrease in systolic blood pressure
b. Water intoxication
c. Increase in cardiac output
d. Increase in systolic blood pressure
e. All of the above

108. G protein coupled receptors. All true EXCEPT:


a. Seven transmembrane components
b. Hydrophobic links
c. Extracellular portion for phosphorylation
d. G protein has intrinsic GTPase activity
e. The receptor is a heterotrimeric protein

109. Endothelins:
a. Produced by damaged vascular endothelium
b. Vasoactive
c. Found in brain & intestine
d. All of the above

110. A low respiratory quotient in a septic patient is due to:


a. Increased lactic acid
b. Fat metabolism
c. Increased ventilation
d. Fever

111. Phosphorylase:
a. Is found in all human cells
b. Present in liver & muscle
c. Something about glucagon & the liver
d. Something about cAMP/adrenergic transmission

112. Creatine phosphate:


a. Is a source of creatinine for protein synthesis
b. Is a source of cyclic AMP for second messenger systems
c. Is a high energy phosphate source for muscle contraction.

113. Metabolic rate is increased least with:


a. Exercise
b. Specific dynamic action of food
c. Hot climate
d. Cold climate
e. Increased CNS activity

114. Glucocorticoids
a. Increases RBC
b. Increases lymphocytes
c. They also increase neutrophils

115. ADH secretion is decreased by:


a. Morphine
b. Nicotine
c. Nausea ( giddy and vomiting)
d. Hypoxia (or ACTH)
e. Alcohol

116. Calcitriol: Main actions on calcium by


a. Increased absorption of Ca++ and PO4 from gut
b. Negative feedback on PTH
c. Increased absorption of vit D from gut.
d. Decreased calcium absorption.

117. The hyperventilation of pregnancy is due to:


a. Progesterone
b. Decreased resistance.
c. Increased resistance.
d. Congestion

118. Normal maternal ABG at term:


a. pH 7.36, pCO2 36 mmHg
b. pH 7.42, pCO2 36 mmHg
c. pH 7.44, pCO2 30 mmHg
d. pH 7.20, pCO2 36 mmHg

119. Closure of the ductus arteriosus occurs due to:


a. Prostaglandins
b. Oxygen
c. Aortic pressure exceeds pulmonary artery pressure
d. Vascular smooth muscle contraction in the presence of oxygen
120. With regard to the foetal circulation:
a. Goes into the left atrium
b. Ductus venosus drains into the IVC directly
c. Oxygen saturation is 40% in umbilical vein.
d. Goes into the right atrium

121. Brown fat:


a. Produces ATP and Heat
b. Insulates the great vessels of the neck
c. Is autonomically mediated
d. Extramitochondrial uncoupling of oxidative phosphorylation

122. Highest O2 saturation in the foetal circulation is in:


a. Thoracic IVC
b. Right atrium
c. Ascending aorta
d. Pulmonary vein
e. Ductus arteriosus

123. Which of the following is immediately due to onset of ventilation in the newly born?
a. Increased left atrial pressure
b. Closure of ductus venosus
c. Decreased RV pressure.
d. Opening of ductus venosus.

124. Which effect is due to spontaneous ventilation in neonate? (Or: The first breath in a
neonate has a predominant role in:)
a. Decreasing RV outflow pressure
b. Closure of ductus venosus
c. Closure of foramen ovale
d. Increased systemic vascular resistance
e. Increased LV pressure

125. FRC in the neonate:


a. 1 ml/kg
b. 15 ml/kg
c. 30 ml/kg
d. 70 ml/kg

126. The reason for increased aortic pressure after birth:


a. Removal of placental circulation
b. Duct closure
c. Increased pulmonary flow

127. Tidal volume of a neonate:


a. 1 ml/kg
b. 3 mls/kg
c. 7 mls/kg
d. 15 mls/kg
e. 30 mls/kg

128. Tidal volume in a 2.3kg neonate:


a. 12ml
b. 10ml
c. 15ml
d. 30ml

PART-C

129. When a pheochromocytoma suddenly discharges a large amount of epinephrine into the
circulation the patients heart rate would be expected to
a. increase because epinephrine has a direct chronotropic effect on the heart
b. increase because of increased parasympathetic discharge to the heart
c. decrease because the increase in blood pressure stimulates the carotid and aortic
baroreceptors
d. decrease because of increased tonic parasympathetic discharge to heart.

130. A person comes to you with an elevated serum free calcium but the parathyroid hormone
level (PTH) is in the normal range . What is the best conclusion?
a. The PTH is normal, therefore the problem does not lie in the parathyroid gland.
b. The person must have excessive sensitivity to PTH, since normal levels are stimulating
excessive calcium mobilization from bone.
c. The PTH should be low if the parathyroid were functioning normally, thus the problem
does lie in the parathyroid gland.
d. You cannot be sure what is going on; you need to perform a parathyroid scan.

131. A mother comes to see you because two of her seven children are morbidly obese. She
and her husband think they may have some relatives in common, and both parents are dark-
haired. She brings one of her children with her. The boy is 13 years old, and weighs 230 lb; his BMI
is 52. At birth, he was found to have adrenal insufficiency and has been treated with appropriate
doses of glucocorticoids. On exam he is an obese, but otherwise normal red-haired kid. What
would be the best treatment, assuming that the phenotype is due to a single genetic lesion, and
that the treatment options listed were available?
a. Leptin injections.
b. A melanocortin 4 receptor (one of the MSH receptors) agonist.
c. An NPY antagonist.
d. Four weeks in a summer camp for obese kids.
e. Thyroid hormone replacement.

132. You have a brother who has type 1 diabetes. He has had severe hypoglycemia in the past,
without warning. Therefore, you have 1 mg of glucagon available for injection in this situation.
When you give the glucagon, which of the following will happen?Glycogenolysis and
gluconeogenesis will promptly increase.
a. The glucose transporter GLUT4 will translocate to the plasma membrane, causing brain
glucose uptake to increase.
b. The tyrosine kinase activity of the glucagon receptor will be turned on.
c. Lipolysis will be suppressed.
d. Nothing, because you have to give some form of glucose along with the glucagon in order
for it to work.

133. Which of the following statements is true regarding thyroid hormone receptors (TRs):
a. The three beta isoforms of the thyroid hormone receptor (TR-b1, b2, b3) are the products
of three distinct genes
b. The predominant TR isoform in the liver is TR-a1
c. TR-b2 is the major mediator of the negative regulation of the TSH and TRH genes in the
pituitary and paraventricular hypothalamus respectively.
d. Uncoupling protein-1 (UCP-1) is an important thyroid hormone target in white adipose
tissue that is involved in thyroid hormone induced thermogenesis
e. Increased cholesterol levels that occur in hypothyroidism are the result of upregulation
of LDL receptor expression in hepatocytes.

134. Which of the following statements regarding thyroid hormone synthesis is correct?
a. Thyroid hormone is actively transported from the thyrocyte (thyroid follicular epithelial
cell) to the circulation by a specific transport protein.
b. The predominant hormone made by thyrocytes is tri-iodothyronine (T3)
c. The synthesis of thyroxine (T4) occurs within the thyrocyte.
d. Thyroid hormone is proteolytically cleaved from thyroglobulin by enzymes acting within
the lumen of the thyroid follicle. The released hormone then diffuses from through the
thyrocyte and into the circulation.
e. Iodine that is released from proteolytic cleavage of thyroglobulin can be recycled from
mono and diiodotyrosines by the action of type I deiodinase enzyme residing in thyroid
follicular cells.

135. A 51-year old woman seeks attention because she has not had a menstrual period for 4
months. She is not pregnant, and her follicle stimulating hormone level returns at 112 uIU/dL
(normal &lt; 14). She is menopausal and is deficient in estrogen and:
a. Activin
b. Inhibin
c. GnRH
d. Corticosterone

136. In parathyroid cells, calcium regulates expression and release of parathyroid hormone
(PTH) by binding to:
a. A nuclear receptor in the supergene family that includes steroid hormone receptors, the
thyroid hormone receptor, and a number of "orphan" receptors.
b. A transmembrane protein receptor that activates a G-protein-mediated signal cascade.
c. The golgi bodies where PTH is stored, and stimulating their fusion with exocytotic vesicles.
d. A calcium-binding domain in the extracellular matrix, inducing a structural change in the
matrix.
e. A glycoprotein in the gap junction between parathyroid cells, mediating transcytotic
calcium flux.
137. Which of the following would cause the change from curve A to curve B in heart
contractility?

a. Norepinephrine
b. Acetylcholine
c. Intracellular acidosis
d. Ca channel blockers

138. With respect to the extra embryonic structures formed in the mammals, the possible
functional attributes have been designated:
A. A allantoin stores urinary waste and helps mediate gas exchange. It is derived from
splanchnopleure at caudal end of the primitive streak.
B. Amnion is a water sac and protects the embryo and its surrounding amniotic fluid. This
epithelium is derived from somatopleure.
C. Chorion is essential for gas exchange in amniotic embryos. It is generated from the
splanchnopleure.
D. Yolk sac is the last embryonic membrane to form and is derived from somatopleure.
Which of the above statements are correct?
a. A and B
b. A and C
c. B and C
d. A and D

139. Insulin and other growth factors stimulate a pathway involving a protein kinase mTOR,
which in turn augments protein synthesis. mTOR essentially modifies protein(s) which in their
unmodified form act as inhibitors of protein synthesis. The following proteins are possible
candidates:
A. eEF-1
B. eEIF-4E-BP1
C. eIF-4E
D. PHAS-1
Which of the following set is correct:
a. A and B
b. B and D
c. A and C
d. B and C

140. The graph represents relative plasma concentration of hormones (A and B) during
reproductive cycle in a normal female. Which one of the following combinations is correct?

a. Is FSH and (B) is estrogen


b. is estrogen and (B) is LH
c. is FSH and (B) is LH
d. is LH and (B) is FSH

141. In an animal experiment; Electrical stimulation of an area in the brain (A) increased a
function (F) which was prevented by systemic injection of adrenergic antagonistic, prazosin.
Injection of carbachol (Cholinerigc agonist) into A also increased function F which was, however,
not prevented by systemic injection of adrenergic antagonistic, prazoin. The results are likely to
be due to the stimulation of
a. Nonadrenergic and cholinoceptive neurons
b. Cholinergic and Non-adrenoceptive neurons
c. Adrenergic terminals in A
d. Both neurons and fibres passing through A.

142. Glucose is mobilized in muscle when epinephrine activates Gs In an experiment in which


muscle cells were stimulated with epinephrine, glucose mobilization was observed even after
withdrawal of epinephrine. This could be
a. Due to the presence of a cAMP phosphodiesterase inhibitor.
b. Very low rates of cyclic AMP formation.
c. Due to the presence of a cAMP phosphodiesterase activator.
d. Due to the absence of protein kinase A.

143. In a stressful condition, ACTH secretion was increased and as a result glucocorticoid
concentration was elevated in blood. One or a combination of the following changes most likely
taking place in this condition:
A. Decreased circulating eosinophils and basophils.
B. Reduced IL2 release.
C. Potentiated inflammatory response to tissue injury.
D. Increased mitotic activity of lymphocytes in lymph nodes.
The correct answer is
a. B and C.
b. A and B.
c. B and D.
d. C and D.

144. In one study, a group of 5 day rat pups were fed for 3 weeks a diet A and the pups gained
weight by 300%. In a second study, when the same diet fed for 3 weeks to rats of 350 gms, they
did not gain weight significantly. In a third study, a diet B was fed to 250-350 gms rats and it was
observed that they delivered normal pups after five weeks. Based on these observations which of
the following statements are correct?
a. Diet A facilitates weight gain than diet B.
b. Diet B facilitates pregnancy and child-bearing.
c. More control experiments are to be conducted for definitive conclusion.
d. Diet A is more energy containing that diet 'B'. Hence, its quantity should be reduced.

145. Which of the following graph represents normal sexual cycle in a normal human female?

a. 1
b. 2
c. 3
d. 4

146. Level of FSH during infancy and adulthood is the same but spermatogenesis is seen only
during adulthood. mRNA levels coding for FSH receptor are also found to be the same in testis of
both age groups. Which of the following investigations will clarify this paradox a little more?
a. Culture testicular cells and add LH to see testosterone production.
b. Culture testicular cells and add testosterone to see comparative rise in FSH mRNA from
both age groups.
c. Culture testicular cells and add FSH to see comparative rise in cAMP production by both
age groups
d. Add both LH and FSH to testicular cells and evaluate cAMP production.

147. In an experiment, sperm removed from epidydimis of a male mouse was added in a dish
containing appropriate media and oocyte. No fertilization was seen. However, when sperm from
epididymis were directly placed in uterus of an ovulated female, she became pregnant. These
observations suggest that:
a. The sperm needs to travel some distance to attain fertilizing ability.
b. The oocyte secretes some biochemicals or factors which help sperm to fertilize.
c. The hormone in body helps sperm to attain fertilizing ability.
d. The contents of female reproductive tract interact with sperm and activate it for
fertilization.

148. GnRH is secreted during infancy (0-6 months) and puberty onwards (4 years and above)
in monkeys. However, i. v injection of GnRH during pre-pubertal period (about 2years of age) led
to elevated LH and FSH in blood compared to untreated 2years old monkey. This suggests that:
A. Hypothalamus is active during pre-pubertal period.
B. GnRH action on pituitary is age dependent.
C. Pituitary matures during adulthood.
D. Pituitary is active in all the stages of development in monkey.
Which one of the following is true?
a. A and B
b. B and C
c. C only
d. D only

149. A person suffering from thryotoxicosis has extremely high level of thyroid hormone in
blood. There is a failure of feed back regulation in hypothalamic-pituitary-thyroid axis. The
detailed blood investigation exhibited high level of the following
a. Thyroid stimulation hormone
b. Thyroid stimulation immunoglobulin
c. Thyrotropin releasing hormone
d. Parathyroid hormone.
ANSWER KEYS

1.d

2.c

3.a

4.b

5.b

6.a

7.e

8.e

9.a

10.e

11.d

12.a

13.c

14.b

15.a

16.b

17.b

18.d

19.d

20.b

21.c

22.d

23.c
24.b

25.c

26.a

27.b

28.c

29.c

30.a

31.a

32.c

33.b

34.a

35.c

36.d

37.b

38.a

39.b

40.a

41.c

42.d

43.b

44.a

45.d

46.c

47.d

48.c
49.d

50.d

51.d

52.a

53.c

54.c

55.a

56.b

57.a

58.a

59.c

60.a

61.a

62.a

63.a

64.b

65.c

66.a

67.c

68.c

69.b

70.c

71.d

72.c

73.d
74.c

75.d

76.c

77.c

78.b

79.a

80.a

81.b

82.b

83.b

84.c

85.c

86.d

87.c

88.c

89.a

90.b

91.a

92.c

93.a

94.d

95.c

96.c

97.e

98.c
99.d

100.b

101.c

102.b

103.b

104.a

105.a

106.b

107.a

108.a

109.d

110.b

111.b

112.c

113.e

114.a

115.e

116.b

117.a

118.c

119.d

120.b

121.c

122.a

123.c
124.a

125.c

126.a

127.c

128.c

PART C

129.a

130.c

131.b

132.a

133.c

134.e

135.a

136.b

137.a

138.a

139.b

140.b

141.a

142.a

143.b

144.c

145.c

146.b

147.d
148.b

149.b

You might also like